Download as pdf or txt
Download as pdf or txt
You are on page 1of 130

SUBJECT CODE : 210241 Discrete Mathematics

(For IN SEM Exam - 30 Marks)


Strictly as per Revised Syllabus of
Savitribai Phule Pune University Subject Code : 210241
Choice Based Credit System (CBCS)
S.E. (Computer) Semester - I S.E. (Computer) Semester - I

First Edition : August 2020

Discrete Mathematics
ã Copyright with Dr. H. R. Bhapkar
(For IN SEM Exam - 30 Marks) All publishing rights (printed and ebook version) reserved with Technical Publications. No part of this book
should be reproduced in any form, Electronic, Mechanical, Photocopy or any information storage and
Dr. H. R. Bhapkar retrieval system without prior permission in writing, from Technical Publications, Pune.

M.Sc. SET, Ph.D. (Mathematics)


MIT Art, Design and Technology University’s
MIT School of Engineering, Lonikalbhor, Pune
Email : drhrbhapkar@gmail.com Published by :
® ®
Amit Residency, Office No.1, 412, Shaniwar Peth,
Mobile : 9011227141 TECHNICAL Pune - 411030, M.S. INDIA, Ph.: +91-020-24495496/97
PUBLICATIONS
SINCE 1993 An Up-Thrust for Knowledge Email : sales@technicalpublications.org Website : www.technicalpublications.org
Dr. Rajesh Nandkumar Phursule
Ph.D. (Computer Science and Engineering)
Associate Professor,
Pimpri Chinchwad College of Engineering Printer :
Nigdi, Pune. Yogiraj Printers & Binders
Sr.No. 10/1A,
Ghule Industrial Estate, Nanded Village Road,
Tal. - Haveli, Dist. - Pune - 411041.

ISBN 978-93-332-0276-3
® ®
TECHNICAL
PUBLICATIONS
SINCE 1993 An Up-Thrust for Knowledge

9 789333 202763 SPPU 19

(i) 9789333202763 [1] (ii)

Preface Syllabus
The importance of Discrete Mathematics is well known in various engineering fields.
Overwhelming response to our books on various subjects inspired us to write this book. Discrete Mathematics - (210241)
The book is structured to cover the key aspects of the subject Discrete Mathematics.
Credit Scheme Examination Scheme and Marks
The book uses plain, lucid language to explain fundamentals of this subject. The 03 Mid_Semester (TH) : 30 Marks
book provides logical method of explaining various complicated concepts and stepwise
methods to explain the important topics. Each chapter is well supported with necessary
illustrations, practical examples and solved problems. All the chapters in the book are Unit I Set Theory and Logic
arranged in a proper sequence that permits each topic to build upon earlier studies. All
care has been taken to make students comfortable in understanding the basic concepts Introduction and significance of Discrete Mathematics, Sets – Naïve Set Theory
of the subject.
(Cantorian Set Theory), Axiomatic Set Theory, Set Operations, Cardinality of set,
The book not only covers the entire scope of the subject but explains the philosophy Principle of incl usion and exclusion, Types of Sets - Bounded and Unbounded Sets,
of the subject. This makes the understanding of this subject more clear and makes it
more interesting. The book will be very useful not only to the students but also to the Diagonalization Argument, Countable and Uncountable Sets, Finite and Infinite Sets,
subject teachers. The students have to omit nothing and possibly have to cover nothing Countably Infinite and Uncountably Infinite Sets, Power set, Propositional Logic -
more. logic, Propositional Equivalences, Application of Propositional Logic - Translating
We wish to express our profound thanks to all those who helped in making this English Sentences, Proof by Mathematical Induction and Strong Mathematical
book a reality. Much needed moral support and encouragement is provided on
Induction. (Chapters - 1, 2, 3)
numerous occasions by our whole family. We wish to thank the Publisher and the
entire team of Technical Publications who have taken immense pain to get this book
in time with quality printing.
Unit II Relations and Functions
Any suggestion for the improvement of the book will be acknowledged and well
appreciated. Relations and their Properties, n-ary relations and their applications, Representing
relations, Closures of relations, Equivalence relations, Partial orderings, Partitions,
Hasse diagram, Lattices, Chains and Anti-Chains, Transitive closure and Warshall‘s
Authors algorithm. Functions - Surjective, Injective and Bijective functions, Identity function,
Dr. H. R. Bhapkar
Partial function, Invertible function, Constant function, Inverse functions and
Dr. Rajesh N. Phursule
Compositions of functions, The Pigeonhole Principle. (Chapters - 4, 5)

Dedicated to the Readers of the Book

(iii) (iv)
1.13 Multiset.........................................................................................................1 - 45

Table of Contents 1.13.1 Multiplicity of an Element . . . . . . . . . . . . . . . . . . . . . . . . . . . . . . . . . . . . . . . . . 1 - 46


1.13.2 Equality of Multisets. . . . . . . . . . . . . . . . . . . . . . . . . . . . . . . . . . . . . . . . . . . . . . 1 - 46
1.13.3 Union and Intersection of Multisets . . . . . . . . . . . . . . . . . . . . . . . . . . . . . . . . . 1 - 46
Unit - I
1.13.4 Difference of Multisets. . . . . . . . . . . . . . . . . . . . . . . . . . . . . . . . . . . . . . . . . . . . 1 - 47
Chapter - 1 Theory of Sets (1 - 1) to (1 - 48) 1.13.5 Sum of Multisets. . . . . . . . . . . . . . . . . . . . . . . . . . . . . . . . . . . . . . . . . . . . . . . . . 1 - 47
1.1 Introduction ......................................................................................................1 - 2 Chapter - 2 Propositional Calculus (2 - 1) to (2 - 38)
1.2 Sets ...................................................................................................................1 - 2 2.1 Introduction ......................................................................................................2 - 2
1.3 Methods of Describing Sets ..............................................................................1 - 2 2.2 Statements or Propositions ..............................................................................2 - 2
1.3.1 Roster Method (Listing Method) . . . . . . . . . . . . . . . . . . . . . . . . . . . . . . . . . . . . . . 1 - 2 2.3 Laws of Formal Logic.........................................................................................2 - 3
1.3.2 Statement Form . . . . . . . . . . . . . . . . . . . . . . . . . . . . . . . . . . . . . . . . . . . . . . . . . . . 1 - 2 2.3.1 Law of Contradiction. . . . . . . . . . . . . . . . . . . . . . . . . . . . . . . . . . . . . . . . . . . . . . . . 2 - 3
1.3.3 Set Builder Notation . . . . . . . . . . . . . . . . . . . . . . . . . . . . . . . . . . . . . . . . . . . . . . . . 1 - 3 2.3.2 Law of Excluded Middle . . . . . . . . . . . . . . . . . . . . . . . . . . . . . . . . . . . . . . . . . . . . . 2 - 3
1.4 Some Special Sets .............................................................................................1 - 3 2.4 Connectives and Compound Statements .........................................................2 - 3
1.5 Subsets..............................................................................................................1 - 3 2.4.1 Compound Statement . . . . . . . . . . . . . . . . . . . . . . . . . . . . . . . . . . . . . . . . . . . . . . 2 - 3
1.5.1 Proper Subset . . . . . . . . . . . . . . . . . . . . . . . . . . . . . . . . . . . . . . . . . . . . . . . . . . . . . 1 - 3 2.4.2 Truth Table . . . . . . . . . . . . . . . . . . . . . . . . . . . . . . . . . . . . . . . . . . . . . . . . . . . . . . . 2 - 4
1.5.2 Improper Subsets . . . . . . . . . . . . . . . . . . . . . . . . . . . . . . . . . . . . . . . . . . . . . . . . . . 1 - 4 2.4.3 Negation . . . . . . . . . . . . . . . . . . . . . . . . . . . . . . . . . . . . . . . . . . . . . . . . . . . . . . . . . 2 - 4
1.5.3 Equal sets. . . . . . . . . . . . . . . . . . . . . . . . . . . . . . . . . . . . . . . . . . . . . . . . . . . . . . . . . 1 - 4 2.4.4 Conjunction . . . . . . . . . . . . . . . . . . . . . . . . . . . . . . . . . . . . . . . . . . . . . . . . . . . . . . . 2 - 4
1.6 Types of Sets .....................................................................................................1 - 4 2.4.5 Disjunction. . . . . . . . . . . . . . . . . . . . . . . . . . . . . . . . . . . . . . . . . . . . . . . . . . . . . . . . 2 - 5
1.7 Venn Diagrams..................................................................................................1 - 6 2.4.6 Conditional Statement (If ..... then ....) . . . . . . . . . . . . . . . . . . . . . . . . . . . . . . . . . 2 - 6

1.8 Operations on Sets............................................................................................1 - 6 2.4.7 Biconditional (If and only if) . . . . . . . . . . . . . . . . . . . . . . . . . . . . . . . . . . . . . . . . . . 2 - 7

1.8.1 Union of Two Sets . . . . . . . . . . . . . . . . . . . . . . . . . . . . . . . . . . . . . . . . . . . . . . . . . . 1 - 6 2.4.8 Special Propositions . . . . . . . . . . . . . . . . . . . . . . . . . . . . . . . . . . . . . . . . . . . . . . . . 2 - 8

1.8.2 Intersection of Two Sets. . . . . . . . . . . . . . . . . . . . . . . . . . . . . . . . . . . . . . . . . . . . . 1 - 7 2.5 Propositional or Statement Formula ..............................................................2 - 12


1.8.3 Disjoint Sets. . . . . . . . . . . . . . . . . . . . . . . . . . . . . . . . . . . . . . . . . . . . . . . . . . . . . . . 1 - 8 2.6 Tautology ........................................................................................................2 - 12
1.8.4 Complement of a Set . . . . . . . . . . . . . . . . . . . . . . . . . . . . . . . . . . . . . . . . . . . . . . . 1 - 8 2.7 Contradiction ..................................................................................................2 - 12
1.8.5 Difference of Sets . . . . . . . . . . . . . . . . . . . . . . . . . . . . . . . . . . . . . . . . . . . . . . . . . . 1 - 8 2.8 Contingency ....................................................................................................2 - 12
1.8.6 Symmetric Difference of Sets . . . . . . . . . . . . . . . . . . . . . . . . . . . . . . . . . . . . . . . 1 - 10
2.9 Precedence Rule .............................................................................................2 - 16
1.9 Algebra of Set Operations...............................................................................1 - 11
2.10 Logical Equivalence.......................................................................................2 - 16
1.10 Principle of Duality........................................................................................1 - 12
2.11 Logical Identities ...........................................................................................2 - 18
1.11 Cardinality of Sets ........................................................................................ 1 - 23
2.12 The Duality Principle .....................................................................................2 - 20
1.12 The Principle of Inclusion and Exclusion for Sets..........................................1 - 24
2.13 Logical Implication ........................................................................................2 - 20

(v) (vi)

2.14 Important Connectives .................................................................................2 - 20 4.6.2 Complement of a Relation . . . . . . . . . . . . . . . . . . . . . . . . . . . . . . . . . . . . . . . . . . 4 - 10

2.15 Normal Forms ...............................................................................................2 - 21 4.6.3 Composite Relation . . . . . . . . . . . . . . . . . . . . . . . . . . . . . . . . . . . . . . . . . . . . . . . 4 - 11

2.15.1 Disjunctive Normal Form (dnf) . . . . . . . . . . . . . . . . . . . . . . . . . . . . . . . . . . . . . 2 - 22 4.7 Types of Relations on Set................................................................................4 - 12


2.15.2 Conjunctive Normal Form (cnf) . . . . . . . . . . . . . . . . . . . . . . . . . . . . . . . . . . . . . 2 - 22 4.7.1 Identity Relation . . . . . . . . . . . . . . . . . . . . . . . . . . . . . . . . . . . . . . . . . . . . . . . . . . 4 - 12

2.15.3 Principal Normal Form . . . . . . . . . . . . . . . . . . . . . . . . . . . . . . . . . . . . . . . . . . . . 2 - 22 4.7.2 Reflexive Relation . . . . . . . . . . . . . . . . . . . . . . . . . . . . . . . . . . . . . . . . . . . . . . . . 4 - 13

2.16 Methods of Proof..........................................................................................2 - 27 4.7.3 Symmetric Relation . . . . . . . . . . . . . . . . . . . . . . . . . . . . . . . . . . . . . . . . . . . . . . . 4 - 14

2.16.1 Law of Detachment (or Modus Ponens) . . . . . . . . . . . . . . . . . . . . . . . . . . . . . . 2 - 28 4.7.4 Compatible Relation . . . . . . . . . . . . . . . . . . . . . . . . . . . . . . . . . . . . . . . . . . . . . . . 4 - 14


4.7.4.1 Asymmetric Relation . . . . . . . . . . . . . . . . . . . . . . . . 4 - 15
2.16.2 Modus Tollen (Law of Contrapositive) . . . . . . . . . . . . . . . . . . . . . . . . . . . . . . . 2 - 29
4.7.5 Antisymmetric Relation . . . . . . . . . . . . . . . . . . . . . . . . . . . . . . . . . . . . . . . . . . . . 4 - 15
2.16.3 Disjunctive Syllogism . . . . . . . . . . . . . . . . . . . . . . . . . . . . . . . . . . . . . . . . . . . . . 2 - 29
4.7.6 Transitive Relation . . . . . . . . . . . . . . . . . . . . . . . . . . . . . . . . . . . . . . . . . . . . . . . . 4 - 15
2.16.4 Hypothetical Syllogism . . . . . . . . . . . . . . . . . . . . . . . . . . . . . . . . . . . . . . . . . . . . 2 - 29
4.7.7 Equivalence Relation . . . . . . . . . . . . . . . . . . . . . . . . . . . . . . . . . . . . . . . . . . . . . . 4 - 16
2.17 Quantifiers ....................................................................................................2 - 32
4.7.8 Properties of Equivalence Relations . . . . . . . . . . . . . . . . . . . . . . . . . . . . . . . . . 4 - 16
2.17.1 Universal Quantifiers . . . . . . . . . . . . . . . . . . . . . . . . . . . . . . . . . . . . . . . . . . . . . 2 - 33
4.7.9 Equivalence Classes . . . . . . . . . . . . . . . . . . . . . . . . . . . . . . . . . . . . . . . . . . . . . . . 4 - 17
2.17.2 Existential Quantifier . . . . . . . . . . . . . . . . . . . . . . . . . . . . . . . . . . . . . . . . . . . . . 2 - 33
4.8 Partitions of a Set............................................................................................4 - 36
2.17.3 Negation of Quantified Statement . . . . . . . . . . . . . . . . . . . . . . . . . . . . . . . . . . 2 - 33
4.8.1 Relation Induced by a Partition . . . . . . . . . . . . . . . . . . . . . . . . . . . . . . . . . . . . . . 4 - 37
Chapter - 3 Mathematical Induction (3 - 1) to (3 - 18) 4.8.2 Refinement of Partitions . . . . . . . . . . . . . . . . . . . . . . . . . . . . . . . . . . . . . . . . . . . 4 - 39
3.1 Introduction ..................................................................................................... 3 - 2 4.8.3 Product and Sum of Partitions. . . . . . . . . . . . . . . . . . . . . . . . . . . . . . . . . . . . . . . 4 - 40
3.2 First Principle of Mathematical Induction Statement.......................................3 - 2 4.8.4 Quotient Set . . . . . . . . . . . . . . . . . . . . . . . . . . . . . . . . . . . . . . . . . . . . . . . . . . . . . 4 - 40
3.3 Second Principle of Mathematical Induction Statement 4.9 Closure of a Relation.......................................................................................4 - 40
(Strong Mathematical Induction) ......................................................................3 - 2
4.9.1 Reflexive Closure. . . . . . . . . . . . . . . . . . . . . . . . . . . . . . . . . . . . . . . . . . . . . . . . . . 4 - 40

Unit - II 4.9.2 Symmetric Closure . . . . . . . . . . . . . . . . . . . . . . . . . . . . . . . . . . . . . . . . . . . . . . . . 4 - 41


4.9.3 Transitive Closure . . . . . . . . . . . . . . . . . . . . . . . . . . . . . . . . . . . . . . . . . . . . . . . . . 4 - 42
Chapter - 4 Relations (4 - 1) to (4 - 64) 4.9.4 Warshall's Algorithm to Find Transitive Closure . . . . . . . . . . . . . . . . . . . . . . . . 4 - 42
4.1 Introduction ......................................................................................................4 - 2 4.10 Partially Ordered Set.....................................................................................4 - 52
4.2 Cartesian Product ............................................................................................4 - 2 4.10.1 Hasse Diagram . . . . . . . . . . . . . . . . . . . . . . . . . . . . . . . . . . . . . . . . . . . . . . . . . 4 - 53
4.3 Relation ............................................................................................................4 - 3 4.10.2 Chains and Antichains . . . . . . . . . . . . . . . . . . . . . . . . . . . . . . . . . . . . . . . . . . . . 4 - 54
4.4 Matrix Representation of a Relation ................................................................4 - 4 4.10.3 Elements of Poset . . . . . . . . . . . . . . . . . . . . . . . . . . . . . . . . . . . . . . . . . . . . . . . . 4 - 54
4.4.1 Relation Matrix Operations . . . . . . . . . . . . . . . . . . . . . . . . . . . . . . . . . . . . . . . . . . 4 - 5 4.10.4 Types of Lattices . . . . . . . . . . . . . . . . . . . . . . . . . . . . . . . . . . . . . . . . . . . . . . . . . 4 - 56
4.4.2 Properties of Relation Matrix . . . . . . . . . . . . . . . . . . . . . . . . . . . . . . . . . . . . . . . . 4 - 6 4.10.5 Properties of a Lattice . . . . . . . . . . . . . . . . . . . . . . . . . . . . . . . . . . . . . . . . . . . . 4 - 58
4.5 Diagraphs .........................................................................................................4 - 6 4.10.6 Types of Lattices . . . . . . . . . . . . . . . . . . . . . . . . . . . . . . . . . . . . . . . . . . . . . . . . . 4 - 58
4.6 Special Types of Relations ................................................................................4 - 9 4.11 Principle of Duality........................................................................................4 - 59
4.6.1 Inverse Relation (OR Converse Relation) . . . . . . . . . . . . . . . . . . . . . . . . . . . . . . . 4 - 9

(vii) (viii)
Chapter - 5 Functions (5 - 1) to (5 - 28)
5.1 Introduction ......................................................................................................5 - 2
5.2 Function ............................................................................................................5 - 2
5.2.1 Important Definitions . . . . . . . . . . . . . . . . . . . . . . . . . . . . . . . . . . . . . . . . . . . . . . . 5 - 2
5.2.2 Partial Functions . . . . . . . . . . . . . . . . . . . . . . . . . . . . . . . . . . . . . . . . . . . . . . . . . . . 5 - 3
5.2.3 Equality of Two Functions . . . . . . . . . . . . . . . . . . . . . . . . . . . . . . . . . . . . . . . . . . . 5 - 4
5.2.4 Identity Function. . . . . . . . . . . . . . . . . . . . . . . . . . . . . . . . . . . . . . . . . . . . . . . . . . . 5 - 4
5.2.5 Constant Function. . . . . . . . . . . . . . . . . . . . . . . . . . . . . . . . . . . . . . . . . . . . . . . . . . 5 - 4
5.2.6 Composite Function . . . . . . . . . . . . . . . . . . . . . . . . . . . . . . . . . . . . . . . . . . . . . . . . 5 - 4
5.3 Special Types of Functions................................................................................5 - 6
5.4 Infinite Sets and Countability..........................................................................5 - 17
5.4.1 Infinite Set . . . . . . . . . . . . . . . . . . . . . . . . . . . . . . . . . . . . . . . . . . . . . . . . . . . . . . . 5 - 17
5.4.2 Countable Sets . . . . . . . . . . . . . . . . . . . . . . . . . . . . . . . . . . . . . . . . . . . . . . . . . . . 5 - 18
5.5 Pigeon Hole Principle......................................................................................5 - 20
5.6 Discrete Numeric Functions............................................................................5 - 22
5.6.1 Basic Operations on Numeric Functions . . . . . . . . . . . . . . . . . . . . . . . . . . . . . . . 5 - 23
5.6.2 Finite Differences of a Numeric Functions . . . . . . . . . . . . . . . . . . . . . . . . . . . . . 5 - 24

(ix) (x)

Discrete Mathematics 1-2 Theory of Sets

1.1 Introduction
Unit - I The notion of a set is a fundamental concept to all of Mathematics and every branch
of mathematics can be considered as a study of sets of objects of one kind or another. A
great mathematician Cantor was the founder of the theory of sets. Let us now consider
the idea of a set.

1 Theory of Sets 1.2 Sets


A set is a collection of well defined objects. An object in the set is called a member or
element of the set. The objects themselves can be almost anything. e.g. Books, numbers,
cities, countries, animals, etc. In the above definition, the words set and collection for
almost all practical purposes are synonymous. Elements of a set are usually denoted by
Syllabus
lower case letters i.e. a, b, c, ... while sets are denoted by capital letters i.e.
Introduction and significance of Discrete Mathematics, Sets – Naïve Set Theory (Cantorian Set A, B, C, ...
Theory), Axiomatic Set Theory, Set Operatio ns, Cardinality of set, Principle of inclusion and
exclusion, Types of Sets - Bounded and Unbounded Sets, Diagonalization Argument, Countable The symbol ' Î' indicates the membership in a set.
and Uncountable Sets, Finite and Infinite Sets, Countably Infinite and Uncountably Infinite Sets,
If "x is an element of a set A" then we write x Î A.
Power set.
If "x is not an element of a set A" then we write as x Ï A.
Contents
Examples :
1.1 Introduction 1) The set of letters forming the word "MATHEMATICS"
1.2 Sets 2) The set of students in a class SE Computer Engineering
1.3 Methods of Describing Sets
3) s = {1, 2, 3, 4, 5, 6}
1.4 Some Special Sets
4) The set of professors in SPPU university.
1.5 Subsets
5) The set of all telephone numbers in the directory.
1.6 Types of Sets
1.7 Venn Diagrams 1.3 Methods of Describing Sets
1.8 Operations on Sets The following are the most useful and common methods of describing sets.
1.9 Algebra of Set Operations
1.3.1 Roster Method (Listing Method)
1.10 Principle of Duality . . . . . . . . . . . . . . . . . . Dec.-06, 08, 11, 12, 13, 14, 15, 17, 19
. . . . . . . . . . . . . . . . . . May-05, 06, 08, 14 · · · · · · · · · Marks 6 A set may be described by listing all the members of the set between a pair of braces.
1.11 Cardinality of Sets In this method the order in which the elements are listed is immaterial and it is used
for small sets. e.g. A = {1, 2, 3, 4, 5}, B = {a, b, c, d, e, f, g}
1.12 The Principle of Inclusion and Exclusion for Sets
. . . . . . . . . . . . . . . . . . Dec.-04, 05, 07, 08, 10, 13, 14, 15, 19 1.3.2 Statement Form
. . . . . . . . . . . . . . . . . . May-05, 06, 07, 08,
In this form, set is formed especially where the elements have a common
. . . . . . . . . . . . . . . . . . 14, 15, 17, 19, · · · · · · · · · · · · Marks 13
characteristic.
1.13 Multiset . . . . . . . . . . . . . . . . . . Dec.-13, May-19 · · · · · · · · · · · Marks 4

(1 - 1) ®
TM TECHNICAL PUBLICATIONS - An up thrust for knowledge
TECHNICAL PUBLICATIONS - An up thrust for knowledge
Discrete Mathematics 1-3 Theory of Sets Discrete Mathematics 1-4 Theory of Sets

e.g. 1) The set of all even integers i) A Ì B i.e. A is a subset of B.


2) The set of all Prime Ministers of India ii) $ at least one element in B which is not in A.
i.e. B is not subset of A.
1.3.3 Set Builder Notation
e.g. If A = {1, 2, 3} and B = {1, 2, 3, 4, 5}
It is not always possible to describe a set by the listing method or statement form.
then A is a proper subset of B.
A more compact or concise way of describing the set is to specify the common
It is denoted by A Ì B or A Ì B
property of all elements of the set
e.g. A = { x|x is real and x 2 > 100 } 1.5.2 Improper Subsets
B = { x|x is real and x 20 – x 10 + 1 = 0 } Every set is a subset of itself and null set is a subset of every set.
Subsets A and Q are called improper subsets of A.
1.4 Some Special Sets
The following sets are important and occur frequently in our discussion. 1.5.3 Equal sets SPPU : May-18

1) Set of natural numbers = N = {1, 2, 3, ....} Two sets A and B are said to be equal sets if A Ì B and BÌ A. We write A = B
2) Set of all integers = Z = {...–3, –2, –1, 0, 1, 2, 3 ....} e.g. If A = { x | x 2 = 1 } and B = {1, – 1 } then A = B
3) Set of all positive integers = Z + = {0, 1, 2, 3, ...}
p
1.6 Types of Sets
4) Set of all rational numbers = Q = ìí | p, q Î R, (p, q) =1 q ¹ 0üý
îq þ Depending upon some properties there are mainly following types of sets.
5) Set of real numbers = Â 1) Universal Set :
6) Set of complex numbers = C A non empty set of which all the sets under consideration are subsets is called
universal set.
1.5 Subsets
It is denoted by U. Universal set is not unique.
A set A is said to be a subset of the set B if every element of A is also an element of
B. 2) Singleton Set :

We also say that A is contained in B and denoted by A Ì B A set having only one element is called a singleton set. e.g. A = {5}, B = {f}

If A is a subset of B i.e. A Ì B, then the set 3) Finite Set :

B is called the superset of A. A set is said to be finite if it has finite number of elements.
The set with no elements is called an empty set or null set. It is denoted by f or { }. The number of elements in a set is called the cardinality of that set. It is denoted by
|A| cardinality of set may be finite or infinite.
If A Ì B then there are two possibilities.
e.g. {1, 2, 3, 4 , 5} Þ|A| = 5
i) A = B
ii) A ¹ B, A Ì B 4) Infinite Set :
A set is said to be infinite if it has infinite number of elements
1.5.1 Proper Subset
e.g. N = set of natural numbers
A set A is called proper subset of B iff |N| = ¥

® ®
TECHNICAL PUBLICATIONS - An up thrust for knowledge TECHNICAL PUBLICATIONS - An up thrust for knowledge

Discrete Mathematics 1-5 Theory of Sets Discrete Mathematics 1-6 Theory of Sets

5) Power Set : viii) f Ì f, f Ì {f }, f Î {f }, {f } Ì {f } are true


The set of all subsets of a set A is called the power set of A. ix) f Î f is false
The power set of A is denoted by P(A) x) {a, b} Î {a, b, c, {a, b, c}} is false
Hence P(A) = {X | X Ì A} xi) {a, b, c } Î {a, b, c, {a, b, c}} is true.
If A has n elements then P(A) has 2 n elements. xii) {a, b} Ì {a, b, c, {a, b, c}} is true
e.g. xiii) {a, f } Î {f, {a, f }}

i) A = {a, b}, P(A) = { f, {a }, {b}, {a, b}}. 1.7 Venn Diagrams


ii) P(A) = f iff A = f
We often use pictures in mathematics. The relation between sets can be conveniently
i.e. empty set has only subset f, \ P( f) = {f } illustrated by certain diagrams called Venn diagrams.
iii) A = {a, b, c}, P(A) = {f, {a }, {b}, {c }, {a, b}, {b, c }, {a, c }, {a, b, c }}. This representation first time used by the British Logician John Venn.
6) Power set of power set of A : In Venn diagram
The power set of the power set of A is denoted by P(P(A)). i) Universal set (U) is represented by a large rectangle.

e.g. If A = {a, b}, p(A) = { f, {a }, {b }, {a, b }} ii) Subsets of U are represented by circles or some closed curve
iii) If AÌ B then the circle representing A lies inside of circle representing B.
P(P(A)) = {f, p(A), {f}, {a}, {b}, {a, b}, {{a}, {b}}{f, {a}}
iv) If A and B are disjoint sets then circles of A and B do not have common area.
{{a}, {a, b }}, {f, { b }}, {{b }, {a, b }}
v) If A and B are not disjoint sets then circles of A and B have some common area.
{ f {a, b}}, {f, {a}, {b}}, { f, {a}, {a, b}}
{ f, {b}, {a, b}}, {{a}, {b}, {a, b}}} 1) A Ì U U 2) A Ì B U
A
If |A| = 2 then |P(A)| = 2 2 , |P(P(A)| = 2 4 = 16
A
B
7) A set itself can be an element of some set
Hence one should understand the difference between an element of a set and subset
of a set. Fig. 1.7.1

e.g. If A = {x, y}, P(A) = {f, A, {x }, {y }} 1.8 Operations on Sets


Then To define new sets by combining the given sets, we require set operations. These
i) x Î A is true. But x Ì A is false as x is an element of A not subset of A operations are analogous to the algebraic operations +, –, × of numbers.
ii) {x } Ì A is true as {x} is a subset of A
1.8.1 Union of Two Sets SPPU : May-18
iii) {x } Î A is false as {x} is not an element of A
Let A and B be two sets. The union of two sets A and B is the set of all those
iv) {x } Î P(A) is true. elements which are either in A or in B or in both sets.
v) f Î P(A), f Ì P(A) are true. If is denoted by A È B
vi) {x } Î P(A) is true.
\ A È B = {x | x Î Aor x Î B }
vii) {{x }} Î P(A) is false but {{x }} Ì P(P(A))

® ®
TECHNICAL PUBLICATIONS - An up thrust for knowledge TECHNICAL PUBLICATIONS - An up thrust for knowledge
Discrete Mathematics 1-7 Theory of Sets Discrete Mathematics 1-8 Theory of Sets

By Venn diagram A È B is represented as 1.8.3 Disjoint Sets

U I) Two sets A and B are said to be disjoint sets if


AÇ B = f
A B
e.g. A = {1, 2}, B = {x, y} are disjoint sets.

1.8.4 Complement of a Set


AB
Fig. 1.8.1 Let A be a given set. The complement of A is denoted by A or A' and defined as

Examples A = {x | x Ï A}.

1) A = {1, 2, 3, 4}, B = {x, y, z} By Venn diagram A' is represented as


A È B = {1, 2, 3, 4, x, y, z}
2) A = {1, 2, 3, 4, 5, 6, 7, 8, 9, 10}, B = {2, 3, 5, 7, 11, 13}
A È B = {1, 2, 3, 4, 5, 6, 7, 8, 9, 10, 11, 13} A
A or A'
1.8.2 Intersection of Two Sets

The intersection of two sets A and B is the set of all elements which are in A and
Fig. 1.8.3
also in B. It is denoted by A Ç B -
\ A Ç B = {x | x Î A and x Î B } Examples :

By Venn diagram A Ç B is represented as 1) A = {Set of even numbers} and È = IR

then A or A' = set of odd numbers


A U
2) U = {1, 2, 3, 4, 5, .... 15}, A = {1, 2, 3, 4, 5, 6, 7}
B
A = {8, 9, 10, 11, 12, 13, 14, 15}

AB
Properties of complement of sets

1) U = f and f = U
AB:
2) ( A) = A
Fig. 1.8.2
3) A È A = U, AÇ A=f
Examples :
4) A È U = U, AÇ U=A
1) A = {1, 2, 3, 4, 5}, B = {1, 4}
\ A Ç B = {1, 4} = B 1.8.5 Difference of Sets

2) AÇ f = f Let A and B be two sets. The difference A-B is the set defined as
3) A = {a, b, c}, B = {x, y} A – B = {x | x Î A and x Î B }

AÇ B = f It is also known as the relative complement of B in A.


® ®
TECHNICAL PUBLICATIONS - An up thrust for knowledge TECHNICAL PUBLICATIONS - An up thrust for knowledge

Discrete Mathematics 1-9 Theory of Sets Discrete Mathematics 1 - 10 Theory of Sets

Similarly, 1.8.6 Symmetric Difference of Sets


B – A = {x | x Î B and x Ï A} The symmetric difference of two sets A and B is denoted by A Å B and defined as
By Venn diagram it is represented as A Å B = {x | x Î A – B or x ¬ B– A }
i.e. A Å B = (A – B) È (B– A)
A U
If is also denoted by A D B.
By Venn diagram it is represented as
B

B–A
U
A–B A B

Fig. 1.8.4

Examples :

1) A = {1, 2, 3, 4, 5}, B = {3, 4, 5, 6, 7} AB:

\ A – B = {1, 2}, B – A = {6, 7} Fig. 1.8.5


2) A = Set of prime numbers
Examples :
B = Set of odd integers
1) A = {1, 2, 3, 4, 5, 6}, B = {3, 4, 5, 6, 7, 8}
A – B = {2}
A – B = {1, 2}, B – A = {7, 8}
3) A = {a, b, f, {a, c }}
A Å B = {1, 2, 7, 8}
A – {a, b} = {{a, c }, f }
2) A = f, B = {x, f, {f}}
{a, c} – A = {c}
A Å B = {x, {f}}
Properties of difference 3) A = {a}, B = {a, b}
Let A and B be two sets. Then P(A) Å P(B) = {{b}, {a, b}}
1) A = U–A
Properties of symmetric difference
2) A – A = f, f– f = f
1) AÅA = f
3) A – A = A, A – A = A, U – f = f,
2) AÅf = A
4) A – f = A, A– f = U – A
3) AÅU = U – A = A
5) A – B = A Ç B
4) AÅA = U
6) A – B = B – A iff A=B
5) A Å B = ( A È B) – ( A Ç B)
7) A – B = A iff AÇ B = f
8) A – B = f iff AÌ B

® ®
TECHNICAL PUBLICATIONS - An up thrust for knowledge TECHNICAL PUBLICATIONS - An up thrust for knowledge
Discrete Mathematics 1 - 11 Theory of Sets Discrete Mathematics 1 - 12 Theory of Sets

1.9 Algebra of Set Operations Similarly prove that (A È B) Ç (A È C) Ì A È ( BÇ C)

The set operations obey the same rules as those of numbers such as commutativity, Hence the proof
associativity and distributivity. In addition there are similar rules to logic such as Similarly reader can prove
Idempotent, Absorption De Morgan's Law's and so on.
A Ç ( BÈ C) Ì (A Ç B) È ( A Ç C)
Theorem : Let A, B, C be any sets then
6) Prove that AÈ B = AÇ B
1) Commutativity with respect to È and Ç
Let x Î AÈ B Þ x Ï AÈ B
a) A È B = B È A b) A Ç B = B Ç A
Þ x Ï A and x Ï B
2) Associativity w.r.t. È and Ç
Þ x Î A and x Î B
a) A È (BÈ C) = ( A È B) È C
b) A Ç (BÇ C) = ( A Ç B) Ç C Þ x Î AÇ B

3) Distributivity Þ AÈ B Ì AÇ B

a) A È (BÇ C) = ( A È B) Ç (A È C) Similarly AÇ B = AÈ B
b) A Ç (BÈ C) = ( A Ç B) È (A Ç C) Hence AÈ B = AÇ B
4) Idempotent Laws Similarly the reader can prove
a) A È A = A b) A Ç A = A
( A Ç B) = A È B
5) Absorption Laws
7) A = {x | x Ï A}
a) A È ( A Ç B) = A b) A Ç ( A È B) = A
= {x | x Î A} = A
6) De Morgan's Laws
a) (A È B) = A Ç B b) A Ç B = A È B 1.10 Principle of Duality SPPU : Dec.-06, 08, 11, 12, 13, 14, 15, 17, 19,
May-05, 06, 08, 14
7) Double Complement : ( A) = A The principle of duality states that any proved result involving sets and complements
and operations of union and intersection gives a corresponding dual result by replacing
Proof : Proofs of 1), 2), 4), 5) are easy, so reader can prove these
È by f and È by Ç and vice versa.
3) To Prove A È (BÇ C) = (A È B) Ç ( A È C)
e.g. The dual of A È A = U is A Ç A = f
Let x Î A È (BÇ C) Þ x Î A or x Î BÇ C
Examples
Þ x Î A or (x Î B and x Î C)
Example 1.10.1 If U = {x Î Z ¢ | – 5 < x < 5} and A = {x Î Z ¢ | – 2 < x < 3} Where Z ¢ = Set of
Þ (x Î A or x Î B) and (x Î A or x Î C)
integers. State the elements of the sets.
Þ (x Î A È B) and ( x Î A Ç C) A, A Ç A, AÇ U, AÈ U, A Ç U, AÇ A
Þ x Î ( A È B) Ç ( A Ç C) Solution :

Hence A È (BÇ C) Ì (A È B) Ç (A È C) A = {x Î Z ¢ (–5 < x < –2) È (3 < x < 5)}

® ®
TECHNICAL PUBLICATIONS - An up thrust for knowledge TECHNICAL PUBLICATIONS - An up thrust for knowledge

Discrete Mathematics 1 - 13 Theory of Sets Discrete Mathematics 1 - 14 Theory of Sets

AÇ A = A Example 1.10.4 Let U = Z ¢ = Set of all integers


A = Set of even integers
AÇ U = A , AÈ U = U
B = Set of odd integers
AÇ U = A C = Set of prime numbers
AÇ A = f Find A – B,, B – A,, C – A, A – C, B – C
Solution :
Example 1.10.2 If A = {x , y, {x , z } f }. Determine the following sets
i) A – {x, z} ii) {{x, z} – A A – B = B– B = f
iii) A – {{x, y}} iv) {x, z} – A B– A = A – A = f
v) A – P(A) vi) {x} – A
C – A = Set of non prime integers which are not even
vii) A – {x} viii) A – f
ix) f – A x) {x , y, f } – A SPPU : Dec.-08, Marks 4
A – C = B – C = Set of odd integers which are not prime = { ± 1, ± 9, ± 15, ... }

Solution : B– C = A – C = Set of even integers which are not prime = { ± 4 , ± 6, ± 8, ... }

i) A – {x, z} = {x, y, f } Example 1.10.5 Let A, B, C be three sets and A Ç B = A Ç C

ii) {{x, z }} – A = f A Ç B = A Ç C is it necessary that B = C ? Justify.


Solution :
iii) A – {{x, y}} = A
Yes, B = C.
iv) {x, z} – A = {z}
Consider B = B Ç U = BÇ ( A È A)
v) A – p(A) = {x, y, {x, z}}
= (B Ç A) È ( B Ç A)
vi) {x} – A = f
= (A Ç B) È ( A Ç B)
vii) A – {x} = {y, {x, z }, f }
= (A Ç C) È ( A Ç C)
viii) A – f = {x, y, {x, z}}
= (A È A) Ç C
ix) f– A = f
= UÇ C
x) {x, y, f} – A = {{x, z}}

Example 1.10.3 If A = {f, b}, construct the following sets A – f, {f} – A, A È P ( A) \ B=C

SPPU : Dec.-19, Marks 03


Example 1.10.6 Let A, B, C be three sets
Solution : We have A = {f, b }
i) Given that A È B = A È C, is it necessary that B = C ?
\ A – f = {b} ii) Given that A Ç B = A Ç C, is it necessary that B = C ?
{f} – A = f and P(A) = {f, {f}, {b}, A} Solution :

A È P(A) = {f, {f}, {b}, A} i) No,


Let A = {x, y, z}, B = {x}, C = {z}
AÈ B = A and AÈ C = A
\ AÈ B = AÈ C = A But B¹ C
® ®
TECHNICAL PUBLICATIONS - An up thrust for knowledge TECHNICAL PUBLICATIONS - An up thrust for knowledge
Discrete Mathematics 1 - 15 Theory of Sets Discrete Mathematics 1 - 16 Theory of Sets

ii) No, Example 1.10.10 Show that [ A Ç ( B È A )] È B = B

Let A = {x, y}, B = {y, z, w}, C = {y, p, q} Solution :

A Ç B = {y} = BÇ C But B¹ C [A Ç (BÈ A)] È B = [(A Ç B) È ( A Ç A)] È B

Example 1.10.7 If A Å B = A Å C, is B = C ? = [(A Ç B) È f] È B

Solution : = (A Ç B) È B = B

Yes, Let x Î B Þ x Î A or x Ï A Example 1.10.11 Prove that A Ç ( B – C )] Ì A – ( B Ç C )

Suppose x Î A then x Î A Ç B Þ x Ï A Å B Solution : Let x Î A Ç (B– C) then


Hence x Ï A Å C Þ x Î A Ç C Þ x Î C
Þ x Î A and x Î B– C
i.e. if x Î A then BÍ C
Þ x Î A and ( x Î Band x Ï C)
Suppose x Ï A then x Ï A Ç B so that x Î A Å B
Þ x Î A and x Ï ( BÇ C)
Þ x Ï A Å C Þ x Ï AÈ B Þ x ÎC
Þ x Î A – ( BÇ C)
Hence BÍ C
\ A Ç (B– C) Ì A – (BÇ C)
Similarly C Í B
Example 1.10.12 Salad is made with combination of one or more eatables, how many different
Hence B=C
salads can be prepared from onion, carrot, cabbage and cucumber?
Example 1.10.8 Let A and B are two sets. If A Í B, then prove that P(A) Í P(B). where
SPPU : Dec.-13, Marks 4
P(A) and P(B) are power sets of A and B sets. SPPU : Dec.-17, Marks 6
Solution : The number of different salads can be prepared from onion, carrot, cabbage
Solution : Let A and B be two sets. and cucumber with combination of one or more eatables is 24 – 1 = 16 – 1 = 15
The powr set of A is the set of all subsets of a set A.
Example 1.10.13 Explain the concepts of countably infinite set with example.
Let P(A) be the power set of A and P(B) be the power set of B. SPPU : Dec.-14, Marks 4
If A is a subset of B i.e. A Í B then all elements of A are in B. Solution : A set is said to be countable if its all elements can be labelled as 1, 2, 3, 4, ...
If X Í P(A) but A Í B Þ X Î P(B) A set is said to be countably infinite
Thence P(A) Í P(B) " ´ ÎP(A) if, i) It is countable

Example 1.10.9 If f is an empty set then find p( f), p( p( f)) p( p( p( f))) ii) It has infinitely many elements i.e. It's cardinality is ¥.

Solution : For example

p( f) = {f } 1) The set of natural numbers {1, 2, 3, ...} is countably infinite.

p (p( f)) = {f, {f }} 2) The set of integers is countably infinite.


3) The set of real numbers is infinite but not countable.
p ( p ( p ( f))) = {f, {f }, {{f }}, {f, {f }}}

® ®
TECHNICAL PUBLICATIONS - An up thrust for knowledge TECHNICAL PUBLICATIONS - An up thrust for knowledge

Discrete Mathematics 1 - 17 Theory of Sets Discrete Mathematics 1 - 18 Theory of Sets

Example 1.10.14 Draw Venn diagram and prove the expression. Also write the dual of each of U U
A B A B
the given statements.
i) ( A È B È C ) C = ( A È C ) C Ç ( A È B ) C
ii) (U Ç A) È (B Ç A) = A SPPU : Dec.-11, Marks 6
Solution : Consider the following Venn diagrams.
1 UA= 2 BA=
ii) Consider the following Venn diagrams.

A U A U
A B U U B B
A B

C C 3 (U  A)  (B  A) = 4 A=
Fig. 1.10.4
c
1 ABC= 2 (A  B  C) = From Venn diagrams (3) and (4)
(U Ç A) È (BÇ A) = A
Fig. 1.10.1
Example 1.10.15 Using Venn diagram show that :

A B A B U A È ( B Ç C) = ( A È B ) Ç ( A È C) SPPU : May-05, Marks 4


Solution : Consider the following venn diagrams
B = {x / x Ï B}

A U A U
B B
C C

c c
3 (A  B) = 4 (A  C) =

Fig. 1.10.2 C C

1 B= 2 B  C=
A B U
A U A U
B B
From (2) and (5)
(A  B  C)c = (A  C)c  (A  B)c

C
C C
c c
5 (A  B)  (A  C) 3 A  (B  C) = 4 AB=
and
Fig. 1.10.3
®
® TECHNICAL PUBLICATIONS - An up thrust for knowledge
TECHNICAL PUBLICATIONS - An up thrust for knowledge
Discrete Mathematics 1 - 19 Theory of Sets Discrete Mathematics 1 - 20 Theory of Sets

ii) A Ç B Ç C = A - [(A - B) È (A - C)]


A U A U
B B

A B X A B X

C C
C C
5 A C= 6 (A B) (A C) =
ABC= A–B=

From 3 and 6 , 1 2

A È (B Ç C) = (A È B) Ç (A È C) A B X A B X

Example 1.10.16 Using Venn diagram, prove or disprove.


i) A Å (B Å C) = (A Å B) Å C
ii) A Ç B Ç C = A - [( A - B) È ( A - C)] SPPU : May-06, Marks 4 C C
Solution : i) A Å B = (A È B) - (A Ç B)
A–C= (A – B)  (A – C) =
A Å B : elements which are either in A or in B but not in both A and B. 3 4
and

A X A Y
B B

A B

C C
C
AÅB= (A Å B) Å C
1 2 A – [(A–B)  (A–C)] =

A B A B 5

From 1 and 5
A Ç B Ç C = A - [(A - B) È (A - C)]
C C
Example 1.10.17 Using Venn diagrams show that
BÅC= A Å (B Å C) A È ( B Ç C) = ( A È B) Ç ( A È C) SPPU : May-08, Dec.-12, Marks 3
3 and 4

From 2 and 4 ,
(A Å B) Å C = A Å (B Å C)

® ®
TECHNICAL PUBLICATIONS - An up thrust for knowledge TECHNICAL PUBLICATIONS - An up thrust for knowledge

Discrete Mathematics 1 - 21 Theory of Sets Discrete Mathematics 1 - 22 Theory of Sets

Solution : If B Ì A and C Ì A
A B X A B X then (A – B) È (A – C) = A
Or A, B, C are disjoint sets.
i.e. A Ç B Ç C = Æ

C C Then (A - B) È (A - C) = A
ii) (A - B) È (A - C) = Æ is true.
BC= A  (B  C) =
If A is empty set i.e. A = Æ
1 2
Example 1.10.19 Prove the following using Venn diagram.
A B X A B X
A Ç (B Å C) = ( A Ç B) Å ( A Ç C) SPPU : May-08, 14, Dec.-12, Marks 3
Solution :

A X A X
B B
C C

AB= AC=

3 4

A B X C C

BC= A  (B  C) =
1 2

C
A B X A B X
(AB)  (AC) =

From 2 and 5 ,
A È (B Ç C) = (A È B) Ç (A È C) C C

Example 1.10.18 Let A, B, C be sets. Under what conditions the following statements are AC=
AB=
true ? 4
3
i) ( A - B) È ( A - C) = A ii) ( A - B) È ( A - C) = Æ SPPU : Dec.-06, 15, Marks 3
Solution : i) (A - B) È (A - C) = A A B X

A X A X

B C OR
C C
B

(AB)  (AC)=

1 2 5

® ®
TECHNICAL PUBLICATIONS - An up thrust for knowledge TECHNICAL PUBLICATIONS - An up thrust for knowledge
Discrete Mathematics 1 - 23 Theory of Sets Discrete Mathematics 1 - 24 Theory of Sets

From 2 and 5 1.12 The Principle of Inclusion and Exclusion for Sets
SPPU : Dec.-04, 05, 07, 08, 10, 13, 14, 15, 19
A Ç (B Å C) = (A Ç B) Å (A Ç C) May-05, 06, 07, 08, 14, 15, 17, 19

1.11 Cardinality of Sets Theorem 6 : (Principle of Inclusion - Exclusion for 2 sets)


Let A and B be finite sets then
In the analysis of computer algorithms, we required to count the number of
operations executed by various algorithms. This is necessary to estimate the cost |A È B| = |A|+|B|–|A Ç B|
A B
effectiveness of a particular algorithm. Hence we require to study the cardinality of Proof : By venn diagram
finite sets and understand related properties. A È B = ( A – B) È B
A–B AB
Definition : As A–B and B are disjoint sets.
Let A be any finite set. The number of elements in the set A, is called the cardinality |A È B| = | A – B| +| B|
of the set A. It is denoted by |A| or n(A).
= | A| – | A Ç B| +| B|
e.g. If A = {x, y, z, p} then |A| = 4
|A È B| = | A| +| B| – | A Ç B| Fig. 1.12.1
Theorem 1 : If A = f then |A| = 0 Principle of Inclusion-Exclusion for three sets.
Theorem 2 : If A Ì B then |A| £ |B| Theorem 7 : Let A, B, C be finite sets. Then
Theorem 3 : (Addition Principle)
|A È BÈ C| = | A| +| B| +|C| – | A Ç B| –|A Ç C| –|BÇ C|
Let A and B be two finite sets which are disjoint then |A È B| = |A| + |B|
Proof Let D = BÈ C
Proof : If A = f, B=f then proof is obvious
\ |A È D| = |A|+|D| = |A Ç D|
Let us assume that A ¹ f, B¹ f
and |D| = |BÈ C| = |B|+|C|–|BÇ C|
Suppose A = { a 1 , a 2 , .... a n }, B = { b1 , b 2 , .... b m }
\ |A È BÈ C| = |A È D| = |A|+|B|+|C|–|BÇ C| – | A Ç D|
\ A Ç B = f, |A| = n, |B| = m
= |A|+|B|+|C|–|BÇ C| – | A Ç (BÈ C) |
\ A È B = { a 1 , a 2 , ... a n , b1 , b 2 , .... b m }
= |A|+|B|+|C|–|BÇ C| – | ( A Ç B) È ( A Ç C) |
\ |A È B| = m + n = |A| + |B|. Hence the proof
= |A|+|B|+|C|–|BÇ C| – | A Ç B| –|A Ç C| +| A Ç BÇ C|
Theorem 4 : Let A1 , A 2 , A 3 , .... A n be mutually disjoint finite sets then
= |A|+|B|+|C|–|A Ç B | –|A Ç C| –|B Ç C| + | A Ç B Ç C|
|A1 È A 2 È A 3 È ... È A n | = |A1| +|A 2|+|A 3|+ ... +|A n |
Theorem 8 : Let A, B, C, D be finite sets then
Theorem 5 : If A is finite set and B is any set then
|A È BÈ C È D| = | A| +| B| +|C| +| D| – | A Ç B| – | A Ç C| – | A Ç D|
|A – B| = |A|–|A Ç B| A B
Proof : By Venn diagram – | BÇ C| – | BÇ D| – |C Ç D|

A = (A – B) È (A Ç B) = | A Ç BÇ C| +|A Ç BÇ D| +| A Ç C Ç D| +| BÇ C Ç D|

(A – B) Ç (A Ç B) = f – | A Ç BÇ C Ç D|
A–B

\ By addition principle |A| = |A – B|+|A Ç B|


Fig. 1.11.1
Þ |A – B| = |A|–|A Ç B|
® ®
TECHNICAL PUBLICATIONS - An up thrust for knowledge TECHNICAL PUBLICATIONS - An up thrust for knowledge

Discrete Mathematics 1 - 25 Theory of Sets Discrete Mathematics 1 - 26 Theory of Sets

Examples : \ |A Ç B| ³ 3, |BÇ C| ³ 3, |A Ç C| ³ 3
Example 1.12.1 Among the integers 1 to 300 find how many are not divisible by 3, not by 5. \ |A Ç B| +|A Ç C| +|BÇ C| ³ 3|A Ç BÇ C|
Find also how many are divisible by 3 but not by 7. SPPU : Dec.-08, Marks 6 \ |A È BÈ C| ³ 32 – 3| A Ç BÇ C| = 32 – 3 × 3 = 23
Solution : Let A denotes the set of integers 1 to 300 divisible Hence |A È BÈ C| ³ 23
by 3, B denotes the set of integers 1 to 300 divisible Hence the number of students taking at least one course is ³ 23. The students taking
by 5, C denotes the set of integers 1 to 300 divisible. none of the course is £ 30 – 23 = 7

by 7 |A| = é
300ù
= 100 , |B| = é
300ù
= 60, |C| = é
300ù
= 42, |A Ç B| = é
300 ù Thus 7 or less students take none of the courses.
= 20
êë 3 úû êë 5 úû êë 7 úû êë 3 ´ 5úû
Example 1.12.3 How many integers between 1 to 2000 are divisible by 2 or 3 or 5 or 7.
Find |A Ç B| and |A - C|
Solution : Suppose set A denotes the number of integers between 1 to 2000 divisible
We have A Ç B = A È B = U – (A È B) by 2.
\ |A È B| = | A| +| B| – | A Ç B| Set B is the number of integers between 1 and 2000 divisible by 3.
Set C is the number of integers between 1 and 2000 divisible by 5.
= 100 + 60 – 20 = 140
Set D is the number of integers between 1 and 2000 divisible by 7.
\ |A Ç B| = |U|–|A È B| = 300 – 140 = 160
A = é
2000ù
= 1000
Hence 160 integers between 1 – 300 are not divisible by 3, not by 5. êë 2 úû

| A Ç C| = é
300 ù
B = é
|A – C| = | A| – | A Ç C|, 2000ù
= 14 = 666
ëê 3 ´ 7 ûú ëê 3 ûú
|A – C| = 100 – 14 = 86 C = é
2000ù
= 400
ëê 5 ûú
Hence, 86 integers between 1 – 300 are not divisible by 3 but not by 7.
D = é
2000ù
Example 1.12.2 Out of 30 students in a college, 15 take an art course, 8 take maths course, 6 = 285
êë 7 úû
take physics course. It is know that 3 students take all courses. Show that 7 or more
AÇB = é
2000ù
students take none of the courses? = 333
êë 2 ´ 3 úû
Solution : Let A be the set of students taking an art course
AÇC = é
2000ù
B be the set of students taking a maths course = 200
ëê 2 ´ 5 ûú
C be the set of students taking a physics course
AÇD = é
2000ù
= 142
We have |A| = 15, |B| = 8, |C| = 6, ëê 2 ´ 7 ûú
|A Ç BÇ C| = 3 é 2000ù
BÇ C = = 133
êë 3 ´ 5 úû
|A È BÈ C| = | A| +| B| +|C| – | A Ç B| – | A Ç C| – | BÇ C| +| A Ç BÇ C|

BÇ D = é
= 15 +8 +6 – | A Ç B| – | A Ç C| – | BÇ C| +3 2000ù
= 95
ëê 3 ´ 7 ûú
= 32 – | A Ç B| – | A Ç C| – | BÇ C|
é 2000ù
But |A Ç B| ³ |A Ç BÇ C|, |BÇ C| ³ |A Ç BÇ C| CÇ D = = 57
ëê 5 ´ 7 ûú
and |A Ç C| ³ |A Ç BÇ C|

® ®
TECHNICAL PUBLICATIONS - An up thrust for knowledge TECHNICAL PUBLICATIONS - An up thrust for knowledge
Discrete Mathematics 1 - 27 Theory of Sets Discrete Mathematics 1 - 28 Theory of Sets

A Ç BÇ C = é
2000 ù \ We have U = 260, |A| = 64, |B| = 94
= 66
ëê 2 ´ 3 ´ 5ûú
| C| = 58, A Ç C| = 28, |A Ç B| = 26
é 2000 ù =
A Ç BÇ D = 47 |B Ç C| = 22, |A Ç B Ç C| = 14,
êë 2 ´ 3 ´ 7 úû
We have,
A Ç CÇ D = é
2000 ù
= 28
êë 2 ´ 5 ´ 7 úû |A È B È C| = |A| + |B| + |C| – |A Ç B|
– |A Ç C} – |B Ç C| + |A Ç B Ç C|
BÇ CÇ D = é
2000 ù
= 19
ëê 3 ´ 5 ´ 7 ûú = 64 + 94 + 58 – 28 – 26 – 22 + 14 = 154

A Ç BÇ CÇ D = é
2000 ù
= 9 The total number of students taken none of the three types of courses
ëê 2 ´ 3 ´ 5 ´ 7 ûú
= |U| – |A È B È C|
Number of elements divisible by 2 or 3 or 5 or 7 are A È B È C È D .
= 260 – 154 = 106
From inclusion exclusion principle
Example 1.12.5 Among 100 students, 32 study mathematics, 20 study physics , 45 study
A È BÈ CÈ D = A + B + C + D
biology, 15 study mathematics and biology, 7 study mathematics and physics, 10 study
– [ A Ç B + BÇ C + A Ç C + A Ç D + BÇ D + CÇ D ] physics and biology and 30 do not study any of the three subjects.
+ [ A Ç B Ç C + A Ç B Ç D +|A Ç C Ç D|+|B Ç C Ç D|] a) Find the number of students studying all three subjects.

– [ A Ç BÇ CÇ D ] b) Find the number of students studying exactly one of the three subjects.
Solution : Let A, B, C denotes the set of students studying mathematics, physics and
\ A È BÈ CÈ D = 1000 + 666 + 400 + 285 – [333 + 200 + 142 + 133 + 95 + 57]
biology respectively.
+ [66 + 47 + 28 + 19] – 9
And X = 100
= 2351 – 960 + 160 – 9
A = 32
= 1542
B = 20
Example 1.12.4 In the survey of 260 college students, the following data were obtained :
C = 45
64 had taken a maths course,
AÇC = 15
94 had taken a cs course,
58 had taken a business course, AÇB = 7
28 had taken both a maths and a business course, BÇ C = 10
26 had taken both a maths and a cs course, A ¢Ç B¢Ç C¢ = 30
22 had taken both a cs and a business course,
A ¢Ç B¢Ç C¢ = 100 - A È B È C
14 had taken all types of courses.
How many students were - surveyed who had taken none of the three types of courses. Or A È B È C = 100 – 30
SPPU : May-17, Marks 3 = 70
Solution : Let A be the set of students, taken a maths course. a) A È BÈ C = A + B + C -[ A Ç B + A Ç C + BÇ C ] + A Ç BÇ C
Let B be the set of students, taken a cs course 70 = 32 + 20 + 45 – [7 + 15 + 10] + A Ç B Ç C
Let C be the set of students, taken a business course.
® ®
TECHNICAL PUBLICATIONS - An up thrust for knowledge TECHNICAL PUBLICATIONS - An up thrust for knowledge

Discrete Mathematics 1 - 29 Theory of Sets Discrete Mathematics 1 - 30 Theory of Sets

70 = 97 – [32] + A Ç B Ç C Example 1.12.6 A survey was conducted among 1000 people of these 595 are Democrats, 595
wear glasses, and 550 like icecream. 395 of them are Democrats who wear glasses, 350 of
70 – 65 = A Ç BÇ C
them are Democrats who like icecream, and 400 of them wear glasses and like icecream 250
Þ A Ç BÇ C = 5
of them are Democrats who wear glasses and like icecream. How many of them are not
5 students study all 3 subjects. Democrats do not wear glasses, and do not like icecream ? How many of them are
b) Number of students studying exactly one subject. Democrats who do not wear glasses and do not like icecream ?
Solution : X = 1000
X
A B Let A be the number of Democrats.
A = 595

B be the number of people who wear glasses.


B = 595
C
C be the number of people who like icecream.
C = 550
AÇB = 395
Number of students studying only mathematics is
AÇC = 350
A - A Ç B - A Ç C + A Ç BÇ C
BÇ C = 400
= 32 – 7 – 15 + 5
A Ç BÇ C = 250
= 15
A È BÈ C = A + B + C -[ A Ç B + BÇ C + A Ç C ] + A Ç BÇ C
Number of students studying only physics is = 595 + 595 + 550 – [395 + 350 + 400] + 250
B - BÇ A - BÇ C + A Ç BÇ C
A È BÈ C = 845
= 20 – 7 – 10 + 5
845 people are either Democrats or wear glasses or icecream.
= 8
Þ 1000 – 845 = 155 people are neither Democrats, nor wear glasses nor like
Number of students studying only biology is
icecream.
C - A Ç C - BÇ C + A Ç BÇ C
1000
= 45 – 15 – 10 + 5 A B

= 25

\ Number of students studying exactly one subject


= 15 + 8 + 25 = 48 C

® ®
TECHNICAL PUBLICATIONS - An up thrust for knowledge TECHNICAL PUBLICATIONS - An up thrust for knowledge
Discrete Mathematics 1 - 31 Theory of Sets Discrete Mathematics 1 - 32 Theory of Sets

Number of people who are Democrats who do not like icecream and do not wear Example 1.12.8 Among 200 students in a class, 104 students got an 'A', in first examination
glasses. and 84 students got 'A' in second examination. If 68 students did not get an 'A' in either
= A - A Ç B - A Ç C + A Ç BÇ C of the examination.
= 595 – [395 + 300] + 250 i) How many students got 'A' in both the examination.
= 845 – 695 ii) If number of students who got an 'A' in the first examination is equal to that who got

= 150 an 'A' in second examination. If the total number of students who got 'A' in exactly one
examination is 160 and if 16 students did not get 'A' in either examination. Determine
Example 1.12.7 It is known that at the university 60 percent of the professors play tennis, 50
the number of students who got 'A' in first examination, those who got ‘A’ in second
percent of them play bridge. 70 percent jog, 20 percent play tennis and bridge, 30 percent
examination and number of students who got 'A' in both examinations.
play tennis and jog, and 40 percent play bridge and jog. If some one claimed that 20
SPPU : Dec.-04, Marks 6
percent of the professors jog and play bridge and tennis, would you believe this claim ?
Solution : X ® Universal set
Why ? SPPU : Dec.-13, Marks 6
Solution : X = 200

100
A B
X

F S

Let A, B, C, denotes the number of professors play tennis, bridge and jog
respectively. Let F denote the set of students who got an 'A' in first examination and S denote the
A = 60 set of students who got an 'A' in second examination.
B = 50 68 students did not get A in either examination i.e.
C = 70 (F È S) ¢ = 68
AÇB = 20 Þ FÈ S = X - 1 (F È S) ¢
AÇC = 30 = 200 – 68
BÇ C = 40 Þ FÈ S = 132
A Ç BÇ C = 20
i) Number of students who got A in both the examination will be F Ç S .
A È BÈ C = A + B + C -[ A Ç B + A Ç C + BÇ C ] + A Ç BÇ C
Now FÈ S = F + S - FÇ S
= 60 + 50 + 70 – [20 + 30 + 40] + 20
132 = 104 + 84 – F Ç S
= 110
\ FÇ S = 56
which is not possible as A È B È C Ì X and the number of elements in A È B È C
cannot exceed number of elements in the universal set X.
® ®
TECHNICAL PUBLICATIONS - An up thrust for knowledge TECHNICAL PUBLICATIONS - An up thrust for knowledge

Discrete Mathematics 1 - 33 Theory of Sets Discrete Mathematics 1 - 34 Theory of Sets

= é
ii) F = S 500ù
A = 166
êë 3 úû
The total number of students who got A in exactly one examination is F - S È S - F
= é
i.e. F Å S 500ù
B = 100
ëê 5 ûú
and |F Å S| = 160
= é
500ù
16 students did not get A in either examination C = 45
ëê 11 ûú
\ (F È S) ¢ = 16
= é
500 ù
AÇB = 33
Þ FÈ S = X – (F È S) ¢ ëê 3 ´ 5ûú
Þ FÈ S
= é
= 200 – 16 = 184 500 ù
AÇC = 15
Now FÈ S = F Å S + FÇ S ëê 3 ´ 11ûú

= é
500 ù
As F Å S and F Ç S are pairwise disjoint sets. BÇ C =9
êë 5 ´ 11úû
Þ 184 = 160 + F Ç S
= é
500 ù
A Ç BÇ C =3
Þ FÇ S = 184 – 160 = 24 êë 3 ´ 5 ´ 11úû

\ FÈ S = F + S - FÇ S i) A È BÈ C = A + B + C -[ A Ç B + A Ç C + BÇ C ] + A Ç BÇ C
184 = 2 F - 24
= 166 + 100 + 45 – [13 + 15 + 9] + 3
2 F = 208
= 257
Þ F = 104 and F = S
ii)
Þ S = 104
X
\ Number of students who got A in first examination A B

= F - FÇ S
= 104 – 24 = 80

Similarly number of students who got A in second examination


C
= |S| –| F Ç S | = 104 - 24 = 80

Example 1.12.9 Consider a set of integers 1 to 500. Find


i) How many of these numbers are divisible by 3 or 5 or by 11 ? Dec.-14
Number of integers divisible by 3 or by 11 but not by all 3, 5 and 11.
ii) Also indicate how many are divisible by 3 or by 11 but not by all 3, 5 and 11.
iii) How many are divisible by 3 or 11 but not by 5 ? SPPU : May-05, Marks 6 = A È C - A Ç BÇ C
Solution : Let A denote numbers divisible by 3. = [ A + C - A Ç C ] - A Ç BÇ C

B denote numbers divisible by 5. = 166 + 45 – 15 – 3 = 193


C denote numbers divisible by 11.
A denotes cardinality of A similarly B and C denotes cardinality of B and C.
® ®
TECHNICAL PUBLICATIONS - An up thrust for knowledge TECHNICAL PUBLICATIONS - An up thrust for knowledge
Discrete Mathematics 1 - 35 Theory of Sets Discrete Mathematics 1 - 36 Theory of Sets

iii) Using DeMorgan's law

X N¢Ç T¢Ç F¢ = X - NÈ TÈ F
A B
8 = 60 – N È T È F
Þ NÈ TÈ F = 52
i) NÈ TÈ F = N + T + F -[ NÇ T + NÇ F + TÇ F ] + NÇ TÇ F
C Þ 52 = 25 + 26 + 26 – [11 + 9 + 8] + N Ç T Ç F
Þ NÇ TÇ F = 3
ii)
Number of integers divisible by 3 or 11 but not by 5.
= A È BÈ C – B 60
= 257 – 100 = 157 N T
8 8 10
Example 1.12.10 In the survey of 60 people, it was found that 25 read newsweek magazine,
3
26 read time, 26 read fortune. Also 9 read both newsweek and fortune, 11 read both 6 5
newsweek and time, 8 read both time and fortune and 8 read no magazine at all.
12
i) Find out the number of people who read all the three magazines. F
ii) Fill in the correct numbers in all the regions of the Venn diagram.
iii) Determine number of people who reads exactly one magazine.
SPPU : Dec.-05, 10, 19, Marks 6 Explanation :
Solution : X ® Universal set As NÇ TÇ F = 3
X = 60 and people who read N and T both = 11 which includes people of N Ç T Ç F also.
Let N denote the number of people who read newsweek magazine. Hence people who read only N and T but not F will be 11 – 3, hence 8 people read
\ N = 25 only N and T similarly T Ç F = 8.
Then people read only T and F = 8 – 3 i.e. 5.
T denote the number of people who read time magazine.
and NÇ F = 9
\ T = 26
Þ People read only N and F = 9 – 3 i.e. 6.
F denote the number of people who read fortune magazine.
Also,
\ F = 26
N = 25, which includes N Ç T , N Ç F and N Ç T Ç F .
NÇ T = 11
People who read only N = N – [ NÇ T + NÇ F ] + NÇ TÇ F
NÇ F = 9
= 25 – [11 + 9] + 3
TÇ F = 8
\ People who read only N = 8 …(1)
N¢Ç T¢Ç F¢ = 8
People who read only T = T – [ N Ç T + T Ç F ] + N Ç T Ç F
= 26 – [11 + 8] + 3

® ®
TECHNICAL PUBLICATIONS - An up thrust for knowledge TECHNICAL PUBLICATIONS - An up thrust for knowledge

Discrete Mathematics 1 - 37 Theory of Sets Discrete Mathematics 1 - 38 Theory of Sets

\ People who read only T = 10 … (2) ii)


People who read only F = F – [ NÇ F + TÇ F ]+ NÇ TÇ F
= 26 – [9 + 8] + 3 F G

\ People who read only F = 12 … (3)


28 12 18
iii) Number of people who reads exactly one magazine 8
17 7
= Equation [(1) + (2) + (3) ] = 8 + 10 + 12 = 30
10 R
Example 1.12.11 Suppose that 100 out of 120 mathematics students at a college take at least
one of the languages French, German and Russian. Also suppose
65 study French, 20 study French and German
45 study German, 25 study French and Russian Explanation

42 study Russian, 15 study German and Russian As FÇ G Ç R = 8


i) Find the number of students who study all the three languages. Students who study F and G but not all 3 will be
ii) Fill in correct number of students in each region of Venn diagram.
FÇ G – FÇ G Ç R
iii) Determine the number K of students who study
a) Exactly one language. = 20 – 8 = 12 … (1)
b) Exactly two languages. SPPU : Dec.-05, Marks 6 Students who study F and R but not all 3 will be
Solution : X ® Universal set
FÇ R – FÇ G Ç R
X = 120
= 25 – 8 = 17 … (2)
Let F be the number of students who study French. G be the number of students
who study German and R be the number of students who study Russian. Students who study G and R but not all 3 will be

| F È G È R | = 100 GÇ R – FÇ G Ç R

F = 65, FÇ G = 20 = 15 – 8 = 7 … (3)

G = 45, FÇ R = 25 Students who study only F but not G and not R.

R = 42, GÇ R = 15 = F -[ FÇ G + FÇ R ] + FÇ G Ç R

i) FÈ G È R = F + G + R -[ FÇ G + FÇ R + G Ç R ] + FÇ G Ç R = 65 – [20 + 25] + 8 = 28 … (4)

100 = 65 + 45 + 42 – [20 + 25 + 15] + F Ç G Ç R Students who study only G but not F and not R

100 = 92+ F Ç G Ç R = G -[ FÇ G + G Ç R ] + FÇ G Ç R

Þ FÇ G Ç R = 8 = 45 – [20 + 15] + 8 = 18 … (5)

Hence 8 students study all three languages. Students who study only R but not F and not G.
= R -[ FÇ R + G Ç R ] + FÇ G Ç R
= 42 – [25 + 15] + 8 = 10 … (6)

® ®
TECHNICAL PUBLICATIONS - An up thrust for knowledge TECHNICAL PUBLICATIONS - An up thrust for knowledge
Discrete Mathematics 1 - 39 Theory of Sets Discrete Mathematics 1 - 40 Theory of Sets

iii) a) Number of students who study exactly one language = |A ¢ Ç B ¢ Ç C¢|


= Equation (4) + Equation (5) + Equation (6) = X – A È BÈ C
= 28 + 18 + 10 = 56 = 1000 – 543 = 457
b) Number of students who study exactly two languages. ii)
= Equation (1) + Equation (2) + Equation (3)
= 12 + 17 + 7 = 36 A B

Example 1.12.12 Out of the integers 1 to 1000.


i) How many of them are not divisible by 3, nor by 5, nor by 7 ?
ii) How many are not divisible by 5 and 7 but divisible by 3 ?
C
SPPU : May-06, 08, 14, Marks 6, May-19, Marks 13
Solution : i) Let A denote numbers divisible by 3.

B denote numbers divisible by 5.


and C denote numbers divisible by 7. Number of integers divisible by 3 but not by 5 and not by 7 is|A Ç B ¢ Ç C¢|.

= é
1000ù |A Ç B ¢ Ç C¢| = |A Ç (B È C) ¢|
A = 333
ëê 3 ûú
= A -[ A Ç B + A Ç C ]+ A Ç BÇ C
= é
1000ù
B = 200 = 333 – [66 + 47] + 9 = 229
ëê 5 ûú
Example 1.12.13 In the class of 55 students the number of studying different subjects are as
= é
1000ù
C = 142
êë 7 úû given below : Maths 23, Physics 24, Chemistry 19, Maths + Physics 12, Maths +
Chemistry 9, Physics + Chemistry 7, all three subjects 4.
= é
1000ù
AÇB = 66
êë 3 ´ 5 úû Find the number of students who have taken :
i) At least one subject ii) Exactly one subject iii) Exactly two subjects.
= é
1000ù
AÇC = 47 SPPU : May-07, Marks 6
ëê 3 ´ 7 ûú
Solution : Let X denote universal set.
= é
1000ù
BÇ C = 28
ëê 5 ´ 7 ûú Let M denote set of students studying mathematics.
P denote set of students studying physics
= é
1000 ù
A Ç BÇ C = 9
êë 3 ´ 5 ´ 7 úû and C denote set of students studying chemistry.

A È BÈ C = A + B + C -[ A Ç B + A Ç C + BÇ C ] + A Ç BÇ C Then X = 55, M = 23, P = 24, C = 19, M Ç P = 12,

= 333 + 200 + 142 – [66 + 47 + 28] + 9 MÇ C = 9, P Ç C = 7

= 543 M Ç PÇ C = 4
i) M È PÈ C = M + P + C - [ M Ç P + M Ç C + PÇ C ] + M Ç PÇ C
This show 543 numbers are divisible by 3 or 5 or 7. Hence numbers which are not
divisible by 3, nor by 5, nor by 7. = 23 + 24 + 19 – [12 + 9 + 7] + 4 = 42

® ®
TECHNICAL PUBLICATIONS - An up thrust for knowledge TECHNICAL PUBLICATIONS - An up thrust for knowledge

Discrete Mathematics 1 - 41 Theory of Sets Discrete Mathematics 1 - 42 Theory of Sets

ii) Number of students studying mathematics but not physics and not chemistry. Example 1.12.14 100 of the 120 engineering students in a college take part in atleast one of
the activity group discussion, debate and quiz.
= M – [ M Ç P + M Ç C ] + M Ç PÇ C
Also : 65 participate in group discussion, 45 participate in debate, 42 participate in quiz,
= 23 – [12 + 9] + 4 = 6 ... (1)
20 participate in group discussion and debate 25 participate in group discussion and quiz
Number of students studying physics but not mathematics and not chemistry. 15 participate in debate and quiz. Find the number of students who participate in : i) All
= P – [ M Ç P + PÇ C ] + M Ç PÇ C the three activities ii) Exactly one of the activities. SPPU : Dec.-07, Marks 4
Solution : Let X denote universal set
= 24 – [12 + 7] + 4 = 9 ... (2)
X = 120
Number of students studying chemistry but not mathematics and not physics.
= C - [ M Ç C + PÇ C ] + M Ç PÇ C Let A denote number of students taking part in group discussion.
B denote number of students taking part in debate
= 19 – [9 + 7] + 4 = 7 … (3)
and C denote number of students taking part in quiz.
Hence number of students studying exactly one subject = 6 + 9 + 7 = 22
A È B È C = 100, A = 65, B = 45, C = 42,
iii) Number of students studying mathematics and physics but not chemistry
A Ç B = 20, A Ç C = 25, B Ç C = 15.
= M Ç P – M Ç PÇ C
i) A È BÈ C = A + B + C - [ A Ç B + A Ç C + BÇ C ] + A Ç BÇ C
= 12 – 4 = 8 … (4)
100 = 65 + 45 + 42 – [20 + 25 + 15] + A Ç B Ç C
Number of students studying physics and chemistry but not mathematics
Þ A Ç BÇ C = 8
= PÇ C - M Ç P Ç C
ii) Number of students taking part in group discussion but not in debate and not in
= 7–4=3 … (5) quiz.
Number of students studying mathematics and chemistry but not physics = A -[ A Ç B + A Ç C ] + A Ç BÇ C
= M Ç C – M Ç PÇ C = 65 – [20 + 25] + 8 = 28 ... (1)
= 9–4=5 … (6) Number of students taking part in debate but not in group discussion and not in
Hence number of students studying exactly two subjects quiz.

= 8 + 3 + 5 = 16 = B -[ A Ç B + BÇ C ] + A Ç BÇ C
= 45 – [20 + 15] + 8 = 18 ... (2)

Number of students taking part in quiz but not in group discussion and not in
X
M P debate.

6 8 9 = C -[ A Ç C + BÇ C + A Ç BÇ C ]
4 = 42 – [25 + 15] + 8 = 10 ... (3)
5 3
Hence number of students doing exactly one activity
7 = 28 + 18 + 10 = 56
C

® ®
TECHNICAL PUBLICATIONS - An up thrust for knowledge TECHNICAL PUBLICATIONS - An up thrust for knowledge
Discrete Mathematics 1 - 43 Theory of Sets Discrete Mathematics 1 - 44 Theory of Sets

= A Ç C - A Ç BÇ C
A B = 25 – 12 = 13 ... (2)

28 12 Number of students who know only 'C' and PASCAL but not COBOL.
18
8
17 7 = BÇ C - A Ç BÇ C
= 28 – 12 = 16 ... (3)
10 C
Hence number of students who know exactly two languages.
= 25 + 13 + 16 = 54
Example 1.12.15 It was found that in the first year computer science class of 80 students, 50
iii) Number of students who know only COBOL but not 'C' and not PASCAL.
knew COBOL, 55 'C' and 46 PASCAL. It was also know that 37 knew 'C' and COBOL,
28 'C' and PASCAL and 25 PASCAL and COBOL. 7 students however knew none of the = A -[ A Ç B + A Ç C ]+ A Ç BÇ C
languages. Find = 50 – [37 + 25] + 12 = 0 ... (4)
i) How many knew all the three languages ?
Number of students who know only 'C’ but not COBOL and not PASCAL.
ii) How many knew exactly two languages ?
iii) How many knew exactly one language ? SPPU : May-15, Dec.-15, Marks 4
= B -[ A Ç B + BÇ C ]+ A Ç BÇ C

Solution : Let A denote the set of students who know COBOL. = 55 – [37 + 28] + 12 = 2 ... (5)

B denote the set of students who know ‘C’. Number of students who know only PASCAL but not COBOL and not 'C'.
and C denote the set of students who know PASCAL. = C -[ A Ç C + BÇ C ]+ A Ç BÇ C
and X denote universal set. = 46 – [25 + 28] + 12 = 5 ... (6)
Then X = 80, A = 50, B = 55, C = 46
Hence number of students who know only one language = 0 + 2 + 5 = 7
A Ç B = 37, B Ç C = 28, A Ç C = 25
X
A ¢Ç B¢Ç C¢ = 7 B
Hence (A È B È C) ¢ = 7
A 25 2
Also A È B È C = X - (A È B È C) ¢ 12
13 16
Hence A È B È C = 80 – 7 = 73
i) A È BÈ C = A + B + C -[ A Ç B + A Ç C + BÇ C ] + A Ç BÇ C 5

73 = 50 + 55 + 46 – [37 + 28 + 25] + A Ç B Ç C C

Þ A Ç B Ç C = 12 Example 1.12.16 In a survey of 500 television watchers produced the following information
ii) Number of students who know only COBOL and 'C' but not PASCAL. 285 watch football, 195 watch hockey, 115 watch basketball, 45 watch football and
= A Ç B - A Ç BÇ C basketball, 70 watch football and hockey, 50 watch hockey and basketball and 50 do not
= 37 – 12 = 25 ... (1) watch any of the three games.
i) How many people in the survey watch all three games ?
Number of students who know only COBOL and PASCAL but not 'C'.
ii) How many people watch exactly one game ? SPPU : May-08, Marks 6

® ®
TECHNICAL PUBLICATIONS - An up thrust for knowledge TECHNICAL PUBLICATIONS - An up thrust for knowledge

Discrete Mathematics 1 - 45 Theory of Sets Discrete Mathematics 1 - 46 Theory of Sets

Solution : Let F denote the set of people who watch football. 2) Birth month of students studying in S.E. Computer Engineering and so on.
H denote the set of people who watch hockey and B denote the set of people who Multiset is the generalization of set.
watch basketball. Definition :
X ® Denote universal set A collection of objects that are not necessarily distinct is called a multiset (or Msets).
Then X = 500, F = 285, H = 195, B = 115. To distinguish set and multiset we denote multiset by enclosing elements in a square
F Ç B = 45, F Ç H = 70, H Ç B = 50 brackets e.g. [a, a, b]

F ¢ Ç B ¢ Ç H ¢ = 50 1.13.1 Multiplicity of an Element


i.e. |(F È B È H) ¢| = 50
Let S be a multiset and x Î S. The multiplicity of x is defined as the number of times
Þ F È B È H = X - ( F È B È H) ¢ the element x appears in the multiset S. It is denoted by m( x).
= 500 – 50 = 450 For example i) S = [a, b, c, d, d, d, e, e]

i) Number of people who watch all the three games = F Ç H Ç B . m( a) = 1, m ( b) = 1 , m(c) = 1, m(d) = 3, m(e) = 2

FÈ HÈ B = F + H + B -[ FÇ H + FÇ B + HÇ B ] + FÇ HÇ B 1.13.2 Equality of Multisets


450 = 285 + 195 + 115 – [70 + 45 + 50] + F Ç H Ç B
Let A and B be two multisets. A and B are said to be equal multisets iff
F Ç H Ç B = 20 m A ( x) = m B ( x), " x Î Aor B
ii) Number of people who watch only football. e.g. [a, b, a, b] = [a, a, b, b] but [ a, a] ¹ [ a]
= F -[ FÇ B + FÇ H ] + FÇ BÇ H
Subset of Multisets
= 285 – [45 + 70] + 20 = 190 ... (1)
A multiset 'A' is said to be the multiset of B if multiplicity of each element in A is
Number of people who watch only hockey. less or equal to it's multiplicity in B.
= H -[ HÇ B + HÇ F ] + FÇ BÇ H e.g. [a] Ì [a, a], [a, b, a, b, a] Ì [a, a, a, b, b, b].
= 195 – [50 + 70] + 20 = 95 ... (2)
Number of people who watch only basketball. 1.13.3 Union and Intersection of Multisets

= B -[ BÇ H + BÇ F ] + FÇ BÇ H If A and B are multisets then A È B and A Ç B are also multisets.


= 115 – [50 + 45] + 20 = 40 ... (3) The multiplicity of an element x Î A È B is equal to the maximum of the multiplicity
of x in A and in B.
\ Number of people who watch exactly one game.
The multiplicity of an element x Î A Ç B is equal to the minimum of the multiplicity
= 190 + 95 + 45 = 325 of x in A and in B.

1.13 Multiset SPPU : Dec.-13, May-19 Example :

We know that the set is the collection of well defined distinct objects. But there are so A = [a, b, c, a, b, c, a, a, a], B = [a, a, b, b, b, b]
many practical situations in which objects are not distinct. For example 1) The collection A È B = [a, a, a, a, a, b, b, b, b, c, c]
of alphabets in the word "Missicippi", collection = [m, i, i, i, i, s, s, c, p, p]
A Ç B = [a, a, b]

® ®
TECHNICAL PUBLICATIONS - An up thrust for knowledge TECHNICAL PUBLICATIONS - An up thrust for knowledge
Discrete Mathematics 1 - 47 Theory of Sets Discrete Mathematics 1 - 48 Theory of Sets

1.13.4 Difference of Multisets 2) Multisets are used to denote prime factors of every non-negative integer.

Let A and B be two multisets. The difference A – B is a multiset such that x Î A – B e.g. prime factors of 72 are 2 × 2 × 2 × 3 × 3

if (m A ( x) – m B ( x)) ³ 1 \ A = [2, 2, 2, 3, 3]

e.g. 1) A = [a, a, b, a], B = [a, b] 3) Multisets are used to denote zeros or poles of analytic functions.

\ A – B = [a, a] 4) In Computer Science, multisets are applied in a variety of search and sort
procedure.
2) A = [1, 2, 3, 4, 2, 2, 3, 3], B = [1, 1, 1, 2, 2, 2, 3, 3, 3, 3, 4]
Example 1.13.3 If P = {a , a , a , c, d , d }, Q = {a , a , b , c, c}. Find union, intersection and
A – B = [ ] or f
difference of P and Q. SPPU : May-19, Marks 3

1.13.5 Sum of Multisets Solution : We have

Let A and B be two multisets. The sum of A and B is denoted by A + B and defined P È Q = {a, a, a, b, c, c, d, d}
as for each P Ç Q = {a,a,c}
x Î A + B, m (x) = m A ( x) + m B ( x) P – Q = {a, d, d}
e.g. A = [a, b, c, c], B = [a, a, b, b, c, c]
A + B = [a, a, a, b, b, b, c, c, c, c] qqq
Examples :

Example 1.13.1 Find the multiset to solve equation


AÇ [1, 2, 2, 3, 4] = [1, 2, 3, 4]
AÈ [1, 2, 2, 3] = [1, 1, 2, 2, 3, 3, 4]
Solution : Maximum multiplicity of each element is as follows

m(1) = 2, m( 2) = 2, m( 3) = 2, m( 4) = 1

and minimum multiplicity of each element in A is as follows


m(1) = 1, m( 2) = 1, m( 3) = 1, m( 4) = 1

Therefore,
A = [1, 2, 3, 4] or A = [1, 1, 2, 2, 3, 3, 4]

Example 1.13.2 Explain examples of multisets with its significance. SPPU : Dec.-13, Marks 4

Solution : 1) Multisets are used to denote roots of the polynomial.

\ x 3 + 3x 2 + 3x +1 = 0

Roots are –1, –1, –1


\ A = [–1, –1, –1]

® ®
TECHNICAL PUBLICATIONS - An up thrust for knowledge TECHNICAL PUBLICATIONS - An up thrust for knowledge

Discrete Mathematics 2-2 Propositional Calculus

2.1 Introduction
Unit - I
A discrete structure is defined by a set of axioms. The properties of structure are
derived from the axioms as theorems. Such theorems are proved using valid rules of
reasoning. The propositional calculus (Mathematical logic) is concerned with all kinds of
reasoning. It has two aspects.

2 Propositional Calculus 1) It is analytic theory of art of reasoning whose goal is systematic and codify the
principles of valid reasoning.
2) It is inter-related with problems relating the foundation of mathematics.
A great mathematician Frege G. (1884-1925) developed the idea regarding a
mathematical theory as applied branch of logic.
Syllabus Every student of engineering should learn logic because principles of logic are
Propositional Logic - logic, Propositional Equivalences, Application of Propositional Logic - valuable to problem analysis, programming, logic designing, code designing and many
Translating English Sentences. more.

Contents 2.2 Statements or Propositions


A statement is a declarative sentence which is either true or false but not both. The
2.1 Introduction
truth or falsity of a statement is called its truth value. The truth value of a true
2.2 Statements or Propositions statement is denoted by 'T' and the false statement is denoted by 'F'. They are also
2.3 Laws of Formal Logic denoted by 1 or 0.
2.4 Connectives and Compound Statements . . . . . Dec.-06, 18, May-05, 07, 08, · · Marks 6 Statements are usually denoted by A, B, C, ....... or a, b, c.
2.5 Propositional or Statement Formula
Examples :
2.6 Tautology
1) There are 5 days in a week
2.7 Contradiction
2) 2 + 5 = 7
2.8 Contingency . . . . . . . . . . . . . . . . . . Dec.-10, 12, · · · · · · · · · · · · · · Marks 6
2.9 Precedence Rule
3) y + 3 = 8

2.10 Logical Equivalence . . . . . . . . . . . . . . . . . . Dec.-12, · · · · · · · · · · · · · · · · · Marks 3 4) It will rain tomorrow


2.11 Logical Identities 5) There are 12 months in a year
2.12 The Duality Principle Examples (2) and (5) are true statements
2.13 Logical Implication Example (1) is false statement
2.14 Important Connectives In example (3), it's truth value depends upon the value of y. If y is 5 then sentence is
2.15 Normal Forms . . . . . . . . . . . . . . . . . . Dec.-04, 06, 08, 10, 12, 14 true and if y ¹ 5 then sentence is false. Therefore (4) is not a statement.
. . . . . . . . . . . . . . . . . . May-17, · · · · · · · · · · · · · · · · · Marks 4 In example (4), it's truth value cannot be predicted at this moment but it can be
2.16 Methods of Proof . . . . . . . . . . . . . . . . . . Dec.-09, May-06, 10 · · · · · · · · Marks 4 definitely determined tomorrow. Hence it is a statement.
2.17 Quantifiers . . . . . . . . . . . . . . . . . . Dec.-08, 09, 10, 11, 15,
. . . . . . . . . . . . . . . . . . May-15· · · · · · · · · · · · · · · · · · Marks 4

(2 - 1) ®
TM TECHNICAL PUBLICATIONS - An up thrust for knowledge
TECHNICAL PUBLICATIONS - An up thrust for knowledge
Discrete Mathematics 2-3 Propositional Calculus Discrete Mathematics 2-4 Propositional Calculus

2.3 Laws of Formal Logic 2.4.2 Truth Table


There are two laws of formal logic A table showing the truth value of a statement formula is called truth table.

2.3.1 Law of Contradiction 2.4.3 Negation


It states that the same statement cannot be both true and false
Let p be any simple statement, then the negation of p is formed by writing "It is false
that" before p or introducing the word "not" at the proper place. The negation of p is
2.3.2 Law of Excluded Middle
also obtained by writing "p is false". The negation of p is denoted by ~ p or Ø p or p. If
It p is a statement then either p is true or p is false and there cannot be middle the statement p is true, then ~ p is false and if p is false then ~ p is true.
ground. If a student appear for the examination of discrete structure then student will be
The truth table for negation of p is given below
either pass or fail in that exam. There is no middle stage.

2.4 Connectives and Compound Statements SPPU : Dec.-06,18, May-05,07,08


p ~p

A statement which cannot be further split into simple sentences, is called primary or T F

primitive or atomic statements. F T

In day to day life, we use the words NOT, AND, OR, IF-Then, as well as, BUT,
Examples :
WHILE to connect two or more sentences. But these connectives are flexible in their
meanings, and lead to inexact and ambiguous interpretations. However, mathematics is a Sr. No. Statement p ~p
very precise language and every symbol of mathematics has the unique meaning or
interpretation in mathematical ocean. 1. p : Atharva is intelligent boy ~ p : Atharva is not intelligent boy
OR
Hence we take some special connectives with precise meaning to suit our purpose. It is false that Atharva is intelligent boy
Following are the common connectives with symbols or their rotations.
2. p : Delhi is the capital of India ~ p : Delhi is not the capital of India
Sr. No. Name of connectives Symbol OR
It is false that Delhi is the capital of India
1 Negation ~ or Ø OR
2 And (Meet) ^ It is not the case that Delhi is the capital
of India
3 Or (Join) v

4 If .... then ® Note : ~ p is the unary connective as only one statement is required to form
5 If an only if (iff) «
negation.

2.4.4 Conjunction
2.4.1 Compound Statement
A compound statement which is obtained by combining two primary statements by
A statement which is formed from primary statements by using logical connectives is
using the connective "and" is called conjunction i.e. the conjunction of two statements p
called a compound statement.
and q is the statement p Ù q. It is read as "p and q" or "p meet q".
e.g. If p : I am studying in SE computer class
The statement p Ù q has the truth value T whenever both p and q have the truth value
q : I am learning discrete structure subject T, otherwise p Ù q has the truth value F.
The compound statement is
I am studying in SE computer class and I am learning discrete structure subject.

® ®
TECHNICAL PUBLICATIONS - An up thrust for knowledge TECHNICAL PUBLICATIONS - An up thrust for knowledge

Discrete Mathematics 2-5 Propositional Calculus Discrete Mathematics 2-6 Propositional Calculus

It's tabular representation is as follows : T F F

F T T
p q pÙq
F F F
T T T
Example 2.4.2 1) If p : The data is wrong, q : There is an error in the program.
T F F
Then p Ú q : The data is wrong or there is an error in the program.
F T F The connective 'v' is used in the inclusive sense i.e. at least one possibility exists or even
F F F both possibilities exist.
2)
Example 2.4.1
Sr. p q Disjunction p Ùq
No.
Sr. No. p q Conjunction p Ùq
1. 6 < 2 (F) 5 + 3 = 4 (F) 6 > 2 or 5 + 3 = 4 F (Q p and q both are false )
1. 3 > 2 (T) 5 is prime (T) 3 > 2 and 5 is prime T
2. Delhi is in India (T) 5.3 = 10 (F) Delhi is in India or 5.3 = 10 T (Q p is true)
2. Delhi is in India (T) 3 + 30 = 30 (F) Delhi is in India and 3 + 30 = 30 F
3. 6 – 3 = 3 (T) 5 is prime (T) 6 – 3 = 3 or 5 is prime T (Q p and q are true)
3. Construct a truth table for the conjunction of n < 20 and n > 5, n Î N 3) Construct a truth table for disjunction of p : n is prime, q : n > 10 for nÎ N.
Solution : When n < 20 and n > 5 are true then the conjunction "n < 20 and n > 5" is Solution : The truth table for p Ú q is as follows :
true.
The truth table is as follows : p : n is prime q : n > 10 p Ú q : n is prime or n > 10

T (n = 13) T (13 > 10) T (Q n = 13 > 10 or 13 is prime)


p : n < 20 q:n>5 p Ù q : n < 20 and n > 5 i.e. 5 < n < 20
T (n = 5) F (5 < 10) T (Q n = 5 is prime)
T (n = 15) T (n = 15) T (Q 5 < 15 < 20)
F (n = 12) T (12 > 10) T (Q n = 12 > 10)
T (n = 2) F (n = 2) F (Q n < 5)
F (n = 6) F (6 < 10) F (Q Both p and q are false)
F (n = 30) T (n = 30) F (Q n > 20)

F (n = 50) F --- F (There does not exist any number s.t. n > 20 and n < 5)
2.4.6 Conditional Statement (If ..... then ....)
If p and q are two statements, then the statement p ® q which is read as "if p then q"
2.4.5 Disjunction
is called as conditional statement.
A compound statement obtained by combining two simple statements by using the The conditional statement p ® q can also be read as i) p only if q ii) p implies q
connective "or" is called the disjunction. iii) q if p iv) p is sufficient for q
i.e. If p and q are simple statements then the compound statement "p or q" is the The statement p is called the hypothesis or antecedent and q is called conclusion or
disjunction of p and q. It is denoted by p Ú q. p Ú q is read as "p or q" or "p join q". consequent.
If p is false and q is false then p Ú q is false. Otherwise p Ú q is true. The truth table of If p is true and q is false then p ® q is false. Otherwise p ® q is true.
p Ú q is as follows : The truth table of p ® q is as follows :

p q pÚq p q p®q
T T T T T T

® ®
TECHNICAL PUBLICATIONS - An up thrust for knowledge TECHNICAL PUBLICATIONS - An up thrust for knowledge
Discrete Mathematics 2-7 Propositional Calculus Discrete Mathematics 2-8 Propositional Calculus

T F F If p and q have the same truth value then p « q is true. In other cases p « is false.
F T T The truth table for p « q is as follows
F F T
p q p«q p®q q ®p (p ® q) Ù (q ® p)
Examples : T T T T T T
1) Let p : Atharva is a graduate T F F F T F
q:3+5=8 F T p T F F
\ p ® q : If Atharva is a graduate then 3 + 5 = 8 F F T T T T
Let N be the set of natural numbers and Q be the set of rational numbers. We know
that every natural number is a rational number. By above table
Let p : n is a natural number p « q and ( p ® q) Ù (q ® p) are equivalent.
q : n is a rational number Examples

p : n ÎN q : n ÎQ p ® q : if n Î N then n Î Q 1) Let p : x = 4 q : x + 9 = 13

T (n = 2) T (n Î Q) T (Q n = 2 is rational) \ p «q : x = 4 if and only if x + 9 = 13


T (n = 5) F (Impossible) F (Q Every natural number is 2) 5 + 6 = 11 iff 11 + 6 = 17
rational)
3) We know that an integer n is an even iff n is divisible by 2.
æ 3ö æ3 ö T (Q $ number which is not
F çn = ÷ T ç Î Q÷
è 2ø è2 ø natural but it is rational)
p : n is even q : n is divisible by 2 p « q : n is even iff n is
F (n = 2 ) F ( 2 ÏQ) T (Q $ number which is divisible by 2
neither natural nor rational)
T (n = 10) T (2 divides 10) T (Q n = 10 is even and divisible by 2)
Note : If p then q T (n = 10) F (Impossible) F (Q There does not exist even integer which is not
divisible by 2)
p only if q
F (n = 5) T (Impossible) F (Q There does not exist odd integer which is
q if p divisible by 2)
p is sufficient for q F (n = 5) F (5 is not divisible by 2) T (Q Integer is not even iff it is not divisible by 2)
q is necessary for p
Above all are equivalent to p ® q 2.4.8 Special Propositions

If p ® q is a conditional statement, then


2.4.7 Biconditional (If and only if)
i) q ® p is called its converse
If p and q are two statements, then the compound statement "p if and only if q" is
ii) ~ p ® ~ q is called its inverse
called a biconditional statement. It is denoted by P ® or p « q.
¬
iii) ~ q ® ~ p is called its contrapositive
The biconditional statement is also read as
The truth table of these propositions are as follows
"p if and only if q" or "p iff q" or "p implies and implied by q" or "p is necessary and
sufficient for q".

® ®
TECHNICAL PUBLICATIONS - An up thrust for knowledge TECHNICAL PUBLICATIONS - An up thrust for knowledge

Discrete Mathematics 2-9 Propositional Calculus Discrete Mathematics 2 - 10 Propositional Calculus

p q p®q q®p ~p ~q ~ p®~ q ~q®~p \ p Ù~q


ii) p : I will get first class
T T T T F F T T
q : I study well
T F F T F T T F
r : Score above 80 in mathematics
F T T F T F F T
\ p « (q Ù r)
F F T T T T T T
iii) p : Computers are cheap
Example 2.4.3 Let p denote the statement, "The material is interesting'. q denote the q : Softwares are costly
statement, "The exercises are challenging", and r denote the statement, "The course is \ p Ùq
enjoyable". iv) p : It is very hot
Write the following statements in symbolic form : q : It is very humid
i) The material is interesting and exercises are challenging.
r : Ramesh is having heart problem
ii) The material is interesting means the exercises are challenging and conversely.
\ (p Ù q) Ú r
iii) Either the material is interesting or the exercises are not challenging but not both.
v) p : In small restaurant food in good
iv) If the material is not interesting and exercises are not challenging, then the course is
q : Service is poor
not enjoyable.
\ p Ùq
v) The material is uninteresting, the exercises are not challenging and the course is not
enjoyable. SPPU : Dec.-06, May-08, Marks 6
vi) p : I finish my submission before 5:00 p.m.

Solution : q : It is very hot

i) pÙq r : I will go
s : I will play a game of hockey
ii) (p ® q) Ù (q ® p)
\ (p Ù ~ q) ® (r Ù s)
iii) p Å ~ q
iv) (~ p Ù ~ q) ® ~ r Example 2.4.5 Use p : I will study discrete mathematics, q : I will go to a movie, r : I am in
a good mood. Write the following in English sentence :
v) ~ p Ù ~ q Ù ~ r
a) ~ r ® q b) ~ q Ù p c) q ® ~ p d) ~ p ® ~ r SPPU : Dec.-18, Marks 04
Example 2.4.4 Express following statement in propositional form :
Solution :
i) There are many clouds in the sky but it did not rain.
a) ~ r ® q means, If I am not in a good mood, then I will go to a movie
ii) I will get first class if and only if I study well and score above 80 in mathematics.
iii) Computers are cheap but softwares are costly. b) ~ q Ù p means, I will not go to movie and I will study discrete mathematics.
iv) It is very hot and humid or Ramesh is having heart problem. c) q ® ~ p means, If I will go to a movie then I will not study discrete maths.
v) In small restaurants the food is good and service is poor. d) ~ p ® ~ r means, If I will not study discrete mathematics then I am not in a good
vi) If I finish my submission before 5.00 in the evening and it is not very hot I will go mood.
and play a game of hockey. SPPU : May-05, Marks 6 Example 2.4.6 Express the contrapositive, converse and inverse forms of the following
Solution : p 1
statement if 3 < b and 1 + 1 = 2, then sin = .
3 2 SPPU : May-07, Marks 6
i) p : There are many clouds in the sky
q : It rain

® ®
TECHNICAL PUBLICATIONS - An up thrust for knowledge TECHNICAL PUBLICATIONS - An up thrust for knowledge
Discrete Mathematics 2 - 11 Propositional Calculus Discrete Mathematics 2 - 12 Propositional Calculus

Solution : 2.5 Propositional or Statement Formula


p : 3<b
A statement formula contains one or more simple statements and some logical
q : 1+1=2 connectives.
p 1
r : sin = A statement formula is a string consisting of variables, parentheses and logical
3 2
connective symbols.
Symbolic form : p Ù q ® r
It is called a well formed if it can be generated by the following rules :
Contrapositive : (~ r ® ~ (p Ù q))
i) A statement variable p standing alone is a well formed formula.
i.e. ~ r ® (~ p Ú ~ q) ii) If p is well formed formula then ~ p is also well formed formula.
p 1
i.e. if sin ¹ then 3 ³ b or 1 + 1 ¹ 2 iii) If p and q are well formed formulas then p Ù q, p Ú q, p ® q or q ® p and p « q are
3 2
well formed formulas.
Converse : r ® (p Ù q)
iv) A string of symbols is well formed formula iff it is obtained by finitely many
p 1
i.e. if sin = then 3 < b and 1 + 1 = 2 applications of rules i), ii) and iii)
3 2
\ A statement formula is not a statement and has no truth values. But if put definite
Inverse : ~( p Ù q) ® ~ r statements in place of variables in a given formula we get a statement. It's truth value
i.e. (~ pÚ ~ q) ® ~ r depends upon the truth values of variables.
p 1
i.e. if 3 ³ b or 1 + 1 ¹ 2 then sin ¹ 2.6 Tautology
3 2
Example 2.4.7 Express the contrapositive, converse, inverse and negation forms of the A statement formula that is true for all possible values of it's propositional variables
conditional statement given below. is called a Tautology.
'If x is rational, then x is real'. SPPU : Dec.-06, Marks 4 e.g. p Ú ~ p is a tautology
Solution :
2.7 Contradiction
Let p : x is rational
A statement formula that is always false for all possible values of variables is called a
q : is real contradiction or absurdity.
Symbolic form : p ® q Example : p Ù ~ p is a contradiction
Contrapositive : (~ q ® ~ p)
2.8 Contingency SPPU : Dec.-10, 12
If x is not real, then x is not rational
Converse : (q ® p) A statement formula which is neither tautology nor contradiction is called a
contingency.
If x is real then x is rational
Example 2.8.1 Prove that p ® p is a tautology.
Inverse - (~ p ® q)
If x is not rational, then x is not real Solution : We construct truth table for p ® p

Negation : ~ ( p ® q) p p p®p
º ~ ( p Ú ~ q) T T T
º ~ pÙ~~q F F T
º ~p Ù q As p ® p is always true. Hence p ® p is a tautology.
® ®
TECHNICAL PUBLICATIONS - An up thrust for knowledge TECHNICAL PUBLICATIONS - An up thrust for knowledge

Discrete Mathematics 2 - 13 Propositional Calculus Discrete Mathematics 2 - 14 Propositional Calculus

Example 2.8.2 Prove that p Ú ~ p is a tautology and ~ (p Ú ~ p) is a contradiction. ii) Consider the truth table
Solution : Let us construct truth table for the statement ~ (p Ú ~ p)
p q ~p p®q qÚ~ p (p ® q) « (q Ú ~ p)

p ~p p Ú~ p ~ (p Ú ~ p) T T F T T T

T F F F F T
T F T F
F T T T T T
F T T F F F T T T T

As p Ú ~ p is always true. Hence p Ú ~ p is a tautology and ~ ( p Ú ~ p) is always false. As ( p ® q) « (q Ú ~ p) is always true. Hence it is a tautology.
Hence ~ ( p Ú ~ p) is a contradiction. iii) Consider the truth table
Example 2.8.3 Show that p Ù ~ p is a contradiction and ~ ( p Ù ~ p) is tautology.
p q r p ® q p ® r q ® r p ® (q ® r) (p ® q) ® (p ® r) [ p ® (q ® r) ] ®
Solution : We construct truth table for ~ ( p Ù ~ p) [ (p ® q) ® (p ® r) ]
T T T T T T T T T
p ~p p Ù~ p ~ (p Ù ~ p)
T T F T F F F F T
T F F T T F T F T T T T T

F T F T T F F F F T T T T

F T T T T T T T T
As p Ù ~ p is always false. Hence p Ù ~ p is a contradiction. As ~ (p Ù ~ p) is always F T F T T F T T T
true, Hence ~ (p Ù ~ p) is a tautology. F F T T T T T T T
Example 2.8.4 Determine whether each of the following statement formula is a tautology, F F F T T T T T T
contradiction or contingency. Hence [ ( p ® (q ® r) ] ® [ ( p ® q) ® ( p ® r) ] is a tautology.
i) ( p Ù q) Ù ~ ( p Ú q)
iv) Consider the truth table
ii) ( p ® q) « ( q Ú ~ p)
iii) ( p ® ( q ® r)) ® (( p ® q) ® ( p ® r)) p q r p®q q ®r (p ® q) Ù (q ® r) p®r [ (p ® q) Ù(q ® r) ]
iv) [( p ® q) Ù ( q ® r)] ® ( p ® r) SPPU : Dec.-12, Marks 6 ® (p ® r)
Solution : i) Consider the truth table T T T T T T T T

T T F T F F F T
p q pÙq pÚq ~ (p Úq) (p Ùq) Ù ~ (p Ú q)
T F T F T F T T
T T T T F F T F F F T F F T

T F F T F F F T T T T T T T

F T F T F F F T F T F F T T

F F T T T T T T
F F F F T F
F F F T T T T T
Hence ( p Ù q) Ù ~ ( p Ú q) is a contradiction Hence given statement formula is a tautology.

® ®
TECHNICAL PUBLICATIONS - An up thrust for knowledge TECHNICAL PUBLICATIONS - An up thrust for knowledge
Discrete Mathematics 2 - 15 Propositional Calculus Discrete Mathematics 2 - 16 Propositional Calculus

Example 2.8.5 Show that ( p ® q) Ù ~ q ® ~ p is a tautology without using truth table. Solution : Consider the truth table
Solution : We know that p ® q is true if p is true and q is also true.
I II
\ We need only to show that p ® q and ~ q both are true imply ~ p is true.
P Q R QÚR P ® (Q Ú R) P®Q P ® R (P ® Q) Ú (P ® R)
As the truth value of ~ q is T, the truth value of q is F. And as p ® q is true, this
means that p is false (Q F ® F is true) T T T T T T T T

\ The truth value of p is T. Hence the proof. T T F T T T F T

Example 2.8.6 Prove that [( p ® q) Ù ( r ® s) Ù ( p Ú r)] ® ( q Ú s) is a tautology. T F T T T F T T

SPPU : Dec.-10, Marks 4 T F F F F F F F

Solution : Consider truth table F T T T T T T T

F T F T T T T T
p q r s p®q r ®s pÚr (p ® q) Ù (r ® s) Ù (p Úr) q Ús I ® (q Ús) F F T T T T T T
(I)
F F F F T T T T
T T T T T T T T T T

T T T F T F T F T T From truth table

T T F T T T T T T T p ® (Q Ú R) º ( P ® Q) Ú ( P ® R)
T T F F T T T T T T
2.9 Precedence Rule
T F T T F T T F T T
The order of preference in which the connectives are applied in a formula of
T F T F F F T F F T propositions that has no bracket is
T F F T F T T F T T i) ~ ii) Ù iii) Ú and Å iv) ® and «
T F F F F T T F F T Remark :
F T T T T T T T T T 1) 'Ú' includes 'exclusive or' and 'inclusive or'
F T T F T F T F T T 2) If Ú and Å are present in a statement then first apply the left most one. The same
rule is applicable to ® and «
F T F T T T F F T T

F T F F T T F F T T 2.10 Logical Equivalence SPPU : Dec.-12

F F T T T T T T T T In real life, we come across several similar things with respect to different aspects.
F F T F T F T F F T e.g. Two cars are similar with respect to average, two students are similar with respect
to F.E. marks.
F F F T T T F F T T
Likewise in logic, we can say that two propositions are similar with respect to their
F F F F T T F F F T
truth values.

Hence given statement formula is a tautology. Definition : Two propositions A and B are logically equivalent iff they have the same
truth value for all choices of the truth values of simple propositions involved in it.
Example 2.8.7 Prove by truth table p ® (Q Ú R) º ( P ® Q) Ú ( P ® R). SPPU : Dec.-12
Two propositions (formulas) are equivalent even if they have different variables. Two
statement formulas P and Q are equivalent

® ®
TECHNICAL PUBLICATIONS - An up thrust for knowledge TECHNICAL PUBLICATIONS - An up thrust for knowledge

Discrete Mathematics 2 - 17 Propositional Calculus Discrete Mathematics 2 - 18 Propositional Calculus

Iff P « Q is a tautology. 2.11 Logical Identities


If p is equivalent to Q then it can be represented as P Û Q or P º Q.
Sr. No. Name of identity Identity
The symbol Û is not a connective
1. Idempotence of Ú p º pÚp
Example 2.10.1 Prove by constructing the truth table p ® ( q Ú r) º ( p ® q) Ú ( p ® r) .
SPPU : Dec.-12, Marks 3 2. Idempotence of Ù p º pÙp

Solution : Consider truth table 3. Commutativity of Ú pÚq º qÚp

4. Commutativity of Ù pÙq º qÙp


p q r q Úr p ® (q Ú r) p®q p®r (p ® q) Ú (p ® r)
5. Associativity of Ú p Ú ( q Ú r) º ( p Ú q) Ú r
T T T T T T T T 6. Associativity of Ù p Ù ( q Ù r) º ( p Ù q) Ù r
T T F T T T F T 7. Distributivity of Ù over Ú p Ù ( q Ú r) º ( p Ù q) Ú ( p Ù r)
T F T T T F T T 8. Distributivity of Ú over Ù p Ú ( q Ù r) º ( p Ú q) Ù ( p Ú r)
T F F F F F F F 9. Double negation p º ~ (~ p)
F T T T T T T T 10. De Morgan's laws ~ ( p Ú q) º ~ p Ù ~ q
F T F T T T T T 11. De Morgan's laws ~ ( p Ù p) º ~ p Ù ~ q
F F T T T T T T 12. Tautology p Ú ~ q º Tautology
F F F F T T T T 13. Contradiction p Ù ~ p º Contradiction

14. Absorption laws p Ú (p Ù q) º p


In the columns of p ® (q Ú r) and (p ® q) Ú ( p ® r), truth values are same for all
possible choices of truth values of p, q and r. Hence 15. Absorption laws p Ù (p Ú q) º p

p ® (q Ú r) º ( p ® q) Ú ( p ® r ) Example 2.11.1 De Morgan's laws


Example 2.10.2 Prove that i) ~ ( p Ú q) º ~ p Ù ~ q
p « q º ( p ® q) Ù ( q ® p) º (~ p Ú q) Ù (~ q Ú p). ii) ~ ( p Ù q) º ~ p Ú ~ q
Solution : i) ~ (p Ú q) º ~ p Ù ~ q
Solution : Consider the truth table
Consider the truth table
p q ~p ~q p®q q ® p (p ® q) p « q ~ p Ú q ~ q Ú p (~ p Ú q) Ù (~ q Úp)
Ù (q ® p) p q ~p ~q pÚq ~ (p Ú q) ~ pÙ~ q
T T F F T T T T T T T T T F F T F F
T F F T F T F F F T F T F F T T F F
F T T F T F F F T F F F T T F T F F
F F T T T T T T T T T F F T T F T T

From above table From the table, truth values of ~ ( p Ú q) and ~ pÙ ~ q are same for each choice of
( p « q) º (p ® q) Ù (q ® p) º (~ p Ú q) Ù (~ q Ú p) p and q
Hence ~ ( p Ú q) º ~ p Ù ~ q
® ®
TECHNICAL PUBLICATIONS - An up thrust for knowledge TECHNICAL PUBLICATIONS - An up thrust for knowledge
Discrete Mathematics 2 - 19 Propositional Calculus Discrete Mathematics 2 - 20 Propositional Calculus

ii) ~ ( p Ú q) º ~ p Ú ~ q Example 2.11.3 p ® q and ~ p Ú q are logically equivalent.

Consider the table Solution : Consider the table

p q ~p ~q pÙq ~ (p Ù q) ~ pÚ~ q p q ~p ~ pÚq p®q


T T F F T F F T T F T T
T F F T F T T T F F F F
F T T F F T T F T T T T
F F T T F T T F F T T T

From the table, truth values of ~ ( p Ù q) and (~ pÚ ~ q) are same for each choice of p
Hence ~ p Ú q and p ® q are logically equivalent.
and q.
Hence ~ ( p Ù q) º (~ p Ú ~ q) 2.12 The Duality Principle
Example 2.11.2 Absorption laws Two statement formulas p and p* are said to be duals of each other if either one can
i) p Ú ( p Ù q) º p ii) p Ù ( p Ú q) º p be obtained from the other by replacing Ù , Ú , T, F by Ú , Ù , F, T respectively.
Solution : Example 2.12.1 Write the duals of
i) p Ú (p Ù q) º p i) ~ ( p Ú q) ii) p Ù ( q Ú r) iii) p Ú T
Consider the table Solution : The duals are i) ~ (p Ù q) ii) p Ú (q Ù r) iii) p Ù F
p q pÙq p Ú (p Ù q) p
2.13 Logical Implication
T T T T T
A proposition p is said to logically imply a proposition Q iff one of the following
T F F T T
condition holds :
F T F F F
1) ~ p Ú Q is a Tautology
F F F F F
2) p Ù Q is a Contradiction
From the table, in last two columns truth tables of p Ú (p Ù q) and p are same for each 3) p ® Q is a Tautology
choice of p and q. It is denoted by p Þ Q
Hence p Ú (p Ù q) º p e.g. 1) (p Ù q) Ù ~ (q Ú p) is a contradiction
ii) p Ù (p Ú q) º p
Hence p Ù q Þ q Ú p
Consider the table
The relation p Þ Q is reflexive, antisymmetric and transitive
p q pÚq p Ù (p Ú q) p Note : The symbols ® & Þ are not same
T T T T T Þ is not connective
T F T T T
2.14 Important Connectives
F T T F F

F F F F F I) NAND :
The word NAND is a combination of "NOT" and "AND" where "NOT" stands for
From the table, last two columns are identical hence p Ù (p Ú q) º p
negation and "AND" stands for the conjunction. It is denoted by the symbol ­.
® ®
TECHNICAL PUBLICATIONS - An up thrust for knowledge TECHNICAL PUBLICATIONS - An up thrust for knowledge

Discrete Mathematics 2 - 21 Propositional Calculus Discrete Mathematics 2 - 22 Propositional Calculus

If P and Q are two formulas then NAND connective is defined as 2.15.1 Disjunctive Normal Form (dnf)
P ­ Q « ~ (P Ù Q) A statement formula which consists of a disjunction ( Ú) of fundamental conjunctions
or elementary product ( Ù). It is abbreviated as dnf.
The connective ­ has the following equivalence
A disjunctive normal form of a given formula is constructed as follows :
1) P ­ P « ~ (P Ù P) « ~ P Ú ~ P Û ~ P
1) Replace ' ®' or ' «' by using logical connectives Ù , Ú & ~ . p ® q º ~ p Ú q,
\ P­ P º ~ P
p « q º (~ p Ú q) Ù (~ q Ú p)
2) ( P ­ Q) ­ ( P ­ Q) « ~ ( P ­ Q) « ~ ~ ( P Ù Q) « P Ù Q 2) Use De Morgan's laws to eliminate '~' before sums or products.
3) ( P ­ P) ­ (Q ­ Q) « ~ P­ ~ Q « ~(~ P Ù ~ Q) « P Ú Q 3) Apply distributive laws repeatedly and eliminate product of variables to obtain
Note : NAND connective is commutative but not associative the required normal form.

Examples :
II) NOR :
The connective "NOR" is a combination of "NOT" and "OR" where "NOT" stands for 1) (p Ù q) Ú (q) Ú (~ q Ù p)
negation and "OR" stands for the disjunction. It is denoted by the symbol ¯ and defined 2) (p Ù q Ù r) Ú (p Ù r) Ú (p Ù q) Ú (p Ù r)
as 3) ( p Ù r ) Ú ( p Ù q)
P ¯ Q « ~ (P Ú Q) \ P ¯ Q º ~ (P Ú Q) 4) (~ p Ù r ) Ú (~ q Ù r ) Ú (~ r )
The connective ¯ has the following equivalences All above examples are in disjunctive normal form.
i) P ¯ P « ~ (P Ú P) « ~ P Ù ~ P « ~ P
2.15.2 Conjunctive Normal Form (cnf)
ii) (P ¯ Q) ¯ (P ¯ Q) « ~ ( P ¯ Q) « P Ú Q
A statement formula which consists of a conjunction of the fundamental disjunctions
iii) (P ¯ P) ¯ (Q ¯ Q) « ~ P ¯ ~ Q « ~ (~ P Ú ~ Q) « P Ù Q
( Ú). It is denoted as cnf.
iv) The connective ¯ is commutative but not associative
Examples :
2.15 Normal Forms SPPU : Dec.-04, 06, 08, 10, 12, 14, May-17
1) p Ù (p Ú q)
If a given statement formula involves n atomic variables, then we have 2 possible n

2) ( p Ú q) Ù (~ p Ú q) Ù (~ q)
combinations of truth values of statements replacing the variables. The construction of
3) ( p Ú q Ú r ) Ù (~ p Ú r ) Ù ( p Ú ~ q Ú r )
the truth table involves finite number of steps but it may not be practical if number of
variables are more. Therefore we reduce the given statement formula to normal form All above examples are in conjunctive normal form.
and find whether a given statement formula is a tautology or contradiction or at least
2.15.3 Principal Normal Form
satisfiable.
A formula which is a conjunction or product of the variables and their p Ù ~ q Let p and q be two statement variables, then p Ù q, p Ù ~ q, ~ p Ù q, ~ p Ù ~ q are called
negations is called an elementary product. If p and q are statements then p, ~ p, ~ p Ù q, minterms of p and q. They are also called Boolean conjunctives of p and q. The number
p Ù ~ q, ~ p Ù ~ q are some examples of elementary products or fundamental conjunctions. of minterms with n variables is 2 n . None of the minterms should contain both a variable
A formula which is a disjunction or sum of the variables and their negations is called an and it's negation. \ p Ù ~ p is not minterm.
elementary sum or fundamental disjunctions. p, ~ p, ~ p Ú q, p Ú ~ q, ~ p Ú p Ú ~ q are some The dual of minterm is called a maxterm.
examples of an elementary sum or fundamental disjunctions.
\ For two statement variables p and q, maxterms are p Ú q, ~ p Ú q, p Ú ~ q and ~ p Ú ~ q.

® ®
TECHNICAL PUBLICATIONS - An up thrust for knowledge TECHNICAL PUBLICATIONS - An up thrust for knowledge
Discrete Mathematics 2 - 23 Propositional Calculus Discrete Mathematics 2 - 24 Propositional Calculus

I) Principal Disjunctive Normal Form Example 2.15.2 Find the conjunctive normal form and disjunctive normal form for the

A statement formula which consists of a disjunction of minterms only is called the following :
principal disjunctive normal form. i) ( p Ú q) ® q ii) p « ( p Ú q) SPPU : Dec.-06, 14, May-17, Marks 3

e.g. Solution :

i) ( p Ú q) ® q º ( p Ú q) Ú q
1) ( p Ú q) Ù (~ p Ù ~ q), ( p Ù q Ù r ) Ú (~ p Ù q Ù r ) are principal cnf.
2) p Ú (p Ù q), ( p Ù q) Ú (~ p Ù q) Ú (~ q), ( p Ù q Ù r ) Ú ( p Ù q) are not principal dnf. º ( p Ù q) Ú q

II) Principal Conjuctive Normal Form º (p Ù q) Ú q … Definition


A statement formula which consists of a conjunction of maxterms only is called the (p Ù q) Ú q º ( p Ú q) Ù (q Ú q)
principle conjunctive normal form.
º (~ p Ú q) Ù q … cnf
e.g.
p ´ (p Ú q) º (p Ú ( p Ú q)) Ù ((p Ú q) Ú p)
1) ( p Ú q) Ù (~ p Ú q), ( p Ú ~ q) Ù (~ pÚ ~ q) are principal cnf.
º (p Ú p Ú q) Ù ((p Ù q) Ú p)
2) (p Ú q) Ù (~ p) is cnf but not principal cnf.
º (p Ú q) Ù ( p Ú p) Ù (q Ú p)
Example 2.15.1 Obtain the conjunctive normal form and disjunctive normal form of the
following formulae given below : º (p Ú q) Ù p Ù (q Ú p) … cnf
i) p Ù ( p ® q ) ii) ~ (p Ú q) ® (p Ù q)
¬ SPPU : Dec.-12, Marks 4 º ((p Ù p) Ú (q Ù p)) Ù (q Ú p)
Solution : º (F Ú (q Ù p)) Ù (q Ú p)
i) p Ù (p ® q) º p Ù (~ p Ú q) ~ cnf º (q Ù p) Ù (q Ú p)
º (p Ù ~ p) Ú ( p Ù q) º (q Ù p Ù q) Ú (q Ù p Ù p)
º F Ú (p Ú q) º (F Ù p) Ú (q Ù p)
º (p Ù q) … Definition a single conjunctive º F Ú (q Ù p)
ii) ~ (p Ú q) ®
¬ (p Ù q) º ( ~ ~ (p Ú q) Ú ( p Ù q)) Ù ( ~ (p Ù q) Ú ~ (p Ú q))
º (q Ù p) … Definition
º ((p Ú q) Ú ( p Ù q) Ù ((~ p Ú ~ q) Ú (~ p Ù ~ q) Example 2.15.3 Find the conjunctive and disjunctive normal forms for the following without

º (p Ú q) Ù ((~ p Ú ~ q Ú ~ p) Ù (~ p Ú ~ q Ú ~ q)) using truth table


i) ( p ® q) Ù ( q ® p) ii) (( p Ù ( p ® q)) ® q) SPPU : Dec.-04, 10, 14, Marks 4
º (p Ú q) Ù (~ p Ú ~ q) Ù (~ p Ú ~ q)
Solution :
º (p Ú q) Ù (~ p Ú ~ q) … cnf
i) (p ® q) Ù (q ® p) º (~ p Ú q) Ù (~ q Ú p)
Further
Further, using the distributive law on the above cnf we have
(p Ú q) Ù (~ p Ú ~ q) º ((p Ú q) Ù ~ p) Ú ((p Ú q) Ù ~ q)
((~ p Ú q) Ù ~ q) Ú ((~ p Ú q) Ù p) = (~ p Ù ~ q) Ú (q Ù ~ q) Ú (~ p Ù p) Ú (q Ù p)
º (p Ù ~ q) Ú (q Ù ~ p) Ú ((p Ù ~ q) Ú (q Ù ~ q)
= (~ p Ù ~ q) Ú (q Ù p) … dnf
º F Ú (q Ù ~ p) Ú ( p Ù ~ q) Ú F (Q p Ù ~ p º q Ù ~ q)
º (p Ù ~ p) Ú ( p Ù ~ q) … Definition

® ®
TECHNICAL PUBLICATIONS - An up thrust for knowledge TECHNICAL PUBLICATIONS - An up thrust for knowledge

Discrete Mathematics 2 - 25 Propositional Calculus Discrete Mathematics 2 - 26 Propositional Calculus

ii) ( p Ù (p ® q)) ® q º ~ ( p Ù ( ~ p Ú q)) Ú q º ( p Ú (~ p Ù q Ù r )] Ù [q Ú (~ p Ù q Ù r )]


º ~ p Ú ~ (~ p Ú q) Ú q º [( p Ú ~ p) Ù ( p Ú q)] Ù [ p Ú r)] Ù [ (q Ú ~ p) Ù (q Ú q) Ù (q Ú r )]
º ~ p Ú (p Ù ~ q) Ú q º [T Ù ( p Ú q) Ù ( p Ú r )] Ù [(q Ú ~ p) Ù q Ù (q Ú r)]
Example 2.15.4 Obtain cnf of each of the following º ( p Ú q) Ù ( p Ú r ) Ù (q Ú ~ p) Ù q which is the required cnf
i) p Ù ( p ® q ) ii) q Ú ( p Ù ~ q ) Ú ( ~ p Ù ~ q ) SPPU : Dec.-08, Marks 4
Example 2.15.7 Find dnf by using truth table
Solution :
( p « ( q Ú r)) ® ~ p
i) p Ù (p ® q) º ( p Ù (~ p Ú q)) Solution : Consider the truth table
º ( p Ù ~ p) Ú ( p Ù q)
p q r ~p q Úr p « (q Ú r) [p « (q Ú r)] ® ~ p
º Ú ( p Ù q)
T T T F T T F
º ( p Ù q) … cnf
T T F F T T F
ii) q Ú (p Ù ~ q) Ú (~ p Ù ~ q) º ((q Ú p) Ù (q Ú ~ q) Ú (~ p Ú ~ q)
T F T F T T F
º (q Ú p) Ù T Ú ( ~ p Ù ~ q)
T F F F F F T ¬ 1
º (q Ú p) Ú (~ p Ù ~ q) F T T T T F T ¬ 2
º (q Ú p Ú ~ p) Ù (q Ù p Ú ~ q) F T F T T F T ¬ 3

º (q Ú T) Ù ( p Ú q Ú ~ q) F F T T T F T ¬ 4

º T Ù (p Ú T) º T Ù T F F F T F T T ¬ 5

º T º (pÚ ~ p) which is the required cnf (single disjunct)


Consider only 'T' from last column and choose corresponding values of 'T' from p, q
Example 2.15.5 Find DNF of (( p ® q) Ù ( q ® p)) Ú p and r. For the first marked row 1 corresponding p is T, q is F and r is F. So take
SPPU : Dec.-14, Marks 4, May-17, Marks 3 p Ù ~ q Ù ~ r or p Ù q ¢ Ù r ¢
Solution : For 2 nd T ® ~ p Ù q Ù r, 3 rd T ® ~ p Ù q Ù ~ r,
((p ® q) Ù (q ® p) Ú p º [(~ p Ú q) Ù (~ q Ú p)] Ú p (Q p ® q º – ~ p Ú q) 4 th T ® ~ p Ù q Ù r, and 5 th T ® ~ pÙ ~ q Ù ~ r

º [p Ú ( ~ p Ú q)] Ù [p Ú ( ~ q Ú p)] Hence the logically equivalent dnf form is

º [(p Ú ~ p) Ú q] Ù [p Ú p Ú ~ p] (Q p Ú p = p [ p « (q Ú r)] ® ~ p º (p Ù ~ q Ù r) Ú (~ p Ù q Ù r) Ú (~ p Ù q Ù ~ r)
p Ú ~ p = T) Ú (~ p Ù ~ q Ù r) Ú (~ p Ù ~ q Ù ~ r)
º (T Ú q) Ù ( p Ú ~ q)
Example 2.15.8 Obtain the principal dnf of (~ p Ú ~ q) ® (~ p Ù r)
º T Ù ( p Ú ~ q)
Solution :
º pÚ ~q which is the required DNF
(~ p Ú ~ q) ® (~ p Ù ~ r) º ~ (~ p Ú ~ q) Ú (~ p Ù r)
Example 2.15.6 Obtain the cnf : ( ~ p Ù q Ù r ) Ú ( p Ù q ) SPPU : Dec.-12, Marks 4
º (~ (~ p) Ù ~ (~ q)] Ú (~ p Ù r)
Solution :
º (p Ù q) Ú (~ p Ù r) which is dnf
(~ p Ù q Ù r ) Ú ( p Ù q) º ( p Ù q) Ú (~ p Ù q Ù r )
º (p Ù q Ù (r Ú ~ r)) Ú (~ p Ù r Ù (q Ú ~ q))
® ®
TECHNICAL PUBLICATIONS - An up thrust for knowledge TECHNICAL PUBLICATIONS - An up thrust for knowledge
Discrete Mathematics 2 - 27 Propositional Calculus Discrete Mathematics 2 - 28 Propositional Calculus

º (p Ù q Ù r) Ú (p Ù q Ù ~ r) Ú (~ p Ù r Ù q) Ú (~ p Ù r Ù ~ q) As mathematical proof is a logical argument that verifies the truth of the theorem.
There are several ways of proving a theorem which are based on one or more rules of
which is the required principal dnf.
inference.
Example 2.15.9 Find the principal cnf of ( p Ù q) Ú (~ p Ù r)
There are following most commonly used rules of inference.
Solution :
2.16.1 Law of Detachment (or Modus Ponens)
[( p Ù q) Ú ( ~ p Ù r) º [(p Ù q) Ú ~ p] Ù [ (p Ù q) Ú r]
Whenever the statements p and p ® q are accepted as true, then we must accept the
º (p Ú ~ p) Ù (q Ú ~ p) Ù (p Ú r ) Ù ( q Ú r)
statement q as true. This rule is represented in the following form
º (q Ú ~ p Ú ( r Ù ~ r ) Ù (p Ú r Ú ( q Ù ~ q)) Ù (q Ú r Ú ( p Ù ~ p) ) p®q
º (q Ú ~ p Ú r ) Ù (q Ú ~ p Ú ~ r ) Ù (p Ú r Ú q) Ù (p Ú r Ú ~ q) ) p
\ q
Ù (q Ú r Ú p) Ù (q Ú r Ú ~ p)
The assertions above the horizontal line are called premises OT hypothesis. And the
º (p Ú q Ú r ) Ù (~ p Ú q Ú r ) Ù (~ p Ú q Ú ~ r) Ù (p Ú ~ q Ú r)
assertion below the line is called the conclusion.
which is the required principle cnf. This rule constitutes a valid argument as (p ® q) Ù p ® q is a tautology.
Example 2.15.10 Find the principal dnf of ~ p Ú q The truth table is as follows

Solution :
p q p®q (p ® q) Ùp [(p ® q) Ù p] ® q
~ p Ú q º [~ p Ù (q Ú ~ q)] Ú [q Ù ( p Ú ~ p)] T T T T T

º (~ p Ù q) Ú (~ p Ù ~ q) Ú (q Ù p) Ú (q Ù ~ p) T F F F T

º (~ p Ù q) Ú (~ p Ù ~ q) Ú (p Ù q) F T T F T

which is the required Principle dnf. F F T F T

2.16 Methods of Proof SPPU : Dec.-09, May-06, 10 This form of valid argument is called the law of detachment as conclusion q is
detached from premise p ® q and p.
We have learned statements or propositions and their truth values. Now, we will
It is also called as the law of direct inference.
discuss ways by which statements can be linked to form a logically valid argument.
Whenever an assertion is made, which is claimed to be true, one has to state an Example
argument which produces the truth of the assertion. To construct a proof we need to If Sushma gets a first class with distinction in B.E. then she will get a good job easily.
derive new assertions from existing ones by different ways. This is also done by using Let p : Sushma gets a first class with distinction in B.E.
valid of inference. q : She will get a good job easily.
Valid argument : A valid argument is a finite sequence of statements p1 , p 2 , .... p n p®q
called as premises (or assumptions or hypothesis) together with a statement C, called the The inference rule is p
conclusion such that p1 Ù p 2 Ù p 3 Ù ... Ù p n ® C is a tautology. \ q

Hence this form of argument is valid.

® ®
TECHNICAL PUBLICATIONS - An up thrust for knowledge TECHNICAL PUBLICATIONS - An up thrust for knowledge

Discrete Mathematics 2 - 29 Propositional Calculus Discrete Mathematics 2 - 30 Propositional Calculus

2.16.2 Modus Tollen (Law of Contrapositive) It's symbolic form is as follows


Modus Tollen is a rule of denying. It can be stated as "If p ® q is true and q is false, s 1 : ( p Ù q) ® r
then p is false. This is represented in the following form. s2 : r ®s
p®q s 3 : ~s ® ~ p Ú ~ q
~p To check validity of this statement, one way is to use truth table or prove logically.
\ ~q
Suppose assignment is invalid. This means that for some assignment of truth values s 1

Above argument is valid as ( p ® q) Ù ~ q ® ~ p is a tautology. In above example is is T, s 2 is T but s 3 is F. s 3 will have truth value F if ~ s is T and ~ p Ú ~ q is F
b ® q and ~ q : Sushma will not get a good job easily then ~ p : She has not a first
i.e. s is F and ~ p is F and ~ q is F
class with distinction in B.E.
i.e. s is F, and p is T and q is T
2.16.3 Disjunctive Syllogism
As s 2 is T, the truth values of r and s both are F. Since s, is T, r is F implies either p
This rule states that "If p Ú q is true and p is false then q is true. or q is F. This is contradiction since by assumption both p and q are true.
It is represented in the following form as Hence given statement is valid.
pÚq
Example 2.16.2 Test the validity of the argument. If a person is poor, he is unhappy. If a person
~p
\ q is unhappy, he dies young. Therefore poor person dies young. SPPU : Dec.-09, Marks 4
Solution :
This argument is valid as (p Ú q) Ù ~ p ® q is a tautology.
Let p : Person is poor
2.16.4 Hypothetical Syllogism
q : Person is unhappy
It is also known as the transitive rule.
r : Person dies young
It can be stated as follows
In symbolic form argument is
"If ( p ® q) and (q ® r ) are true then p ® r is true.
s1 : p ® q
This rule is presented in the following form. s2 : q ® r
p®q s: p® r
q®r
\ p® r The above argument is the rule of hypothetical syllogism. Hence it is valid.

This argument is valid as ( p ® q) Ù (q ® r ) ® ( p ® r ) is a tautology. Example 2.16.3 Determine the validity of the following argument.
s 1 : If I like discrete structure then I will study
Example 2.16.1 Determine whether the argument is valid or not. If I try hard and I have
s 2 : Either I will study or I will fail
talent then I will become a musician. If I become a musician, then I will be happy. _________________________________________
Therefore if I will not be happy then I did not try hard or I do not have talent. s : If I fail then I do not like discrete structure
SPPU : May-10, Marks 4
Solution :
Solution :
Let p : I like discrete structure
Let p : I try hard
q : I will study
q : I have talent
r : I will fail
r : I will become musician
In symbolic form
s : I will be happy
® ®
TECHNICAL PUBLICATIONS - An up thrust for knowledge TECHNICAL PUBLICATIONS - An up thrust for knowledge
Discrete Mathematics 2 - 31 Propositional Calculus Discrete Mathematics 2 - 32 Propositional Calculus

s1 : p ® q Example 2.16.5 I am happy if my program runs. A necessary condition for the program to
s2 : q Ú r run is it should be error free. I am not happy. Therefore the program is not error free.
s : r ®~ p
Solution :
We know that for the validity of argument s 1 Ù s 2 should logically imply s. Let p : I am happy
Assign the truth values T, T, T, to p, q, r respectively. Then s 1 is T and s 2 is also T q : My program runs
But s is T ® F is F r : Program should be error free
Hence argument is invalid. In symbolic form
Example 2.16.4 Determine the validity of the argument q®p
s 1 : All my friends are musicians q®r
s 2 : John is my friend ~p
\ ~r
s 3 : None of my neighbours are musicians
____________________________________
To check validity, consider the truth table
S : John is not my neighbour
SPPU : May-06
p q r q ®p q ®r ~p (q ® p) Ù (q ® r) Ù ~ p (I) ® ~ r
Solution :
Let p : All my friends are musicians (I)

q : John is my friend F F T T T T T F

r : My neighbours are musicians


Last column is not T
s : John is my neighbour
\ Given argument is not tautology.
\ s1 : p
\ Given argument is invalid.
s2 : q
s3 : ~ r
2.17 Quantifiers SPPU : Dec.-08, 09,10, 11, 15, May-15

In symbolic form In grammar a predicate is the word in a sentence which expresses what is said about
p object. i.e. properties of an object or relation among objects. For example "is a good
q teacher", "is a clever student" are predicates. In logic predicate has a broaden role than
~r in grammar. Predicate is presented by using a variable x in place of holder. e.g. x is a
~s prime number.

As all my friends are musicians and John is my friend Þ John is musician. An assertion that contains one or more variables is called a predicate. It's truth value
is predicated after assigning truth values to its variables.
\ p Ù q ® John is musician
A predicate p containing n variables x 1 , x 2 , ... x n is called an n-place predicate and
p Ù q Ù ~ r ® John is musician and my neighbours are not musicians denoted by p (x 1 , x 2 , x 3 , ... x n ). Each variable x i is called as argument.
\ p Ù q Ù ~ r ® John is not my neighbour The values which the variables may assume constitute a collection is called the
p Ù q Ù ~ r ® ~ s is true. universe of discourse or domain of discourse.
There are two types of quantifiers.
Therefore given argument is valid.

® ®
TECHNICAL PUBLICATIONS - An up thrust for knowledge TECHNICAL PUBLICATIONS - An up thrust for knowledge

Discrete Mathematics 2 - 33 Propositional Calculus Discrete Mathematics 2 - 34 Propositional Calculus

2.17.1 Universal Quantifiers $ x [p Ú Q(x)] º p Ú ( $ x Q(x))

If p(x) is a predicate with x as an argument then the universal quantifier for p(x) is 2) Distributivity of " over Ù
the statement. " x [p(x) Ù Q(x)] º " x p(x) Ù "x Q(x)
"For all values of x, p(x) is true". We denote the phrase "For all" by " " x [p Ù Q(x)] º p Ù ("x Q(x))
" means for all or for each or for every. 3) $ x [p Ù Q(x)] º p Ù [ $ x Q(x)]
If p(x) is true for all values of x then 4) " x [p Ú Q(x)] º p Ù ["x Q(x)]
" x p(x) is true. 5) ~ [$ xp(x)] º " x [~ p(x)]
For example - p(x) : x ³ 0, where x is any positive integer. 6) ~ [" x p(x)] º $ x [~ p(x)]
\ The proposition " x p(x) is true. 7) " x p(x) Þ $ x p(x)
However, if x is an integer " x p(x) is false. 8) " x p(x) Ú " x Q(x) Þ " x ( p(x) Ú Q(x)
9) $ x(p(x) Ù Q(x)) Þ $ x p(x) Ù $ x Q(x)
2.17.2 Existential Quantifier
II) Rules of Inference for addition and deletion of quantifiers
In some situations, we only require that there is at least one value for the predicate is
true. Suppose for the predicate p(x), " x p(x) is false, but there exists at least one value of 1) Rule 1 : Universal Instantiation
x for which p(x) is true, then we say that in this proposition, x is bound by existential " x p (x)
, k is some element of the universe
quantification. \ p ( k)
The phrase "there exists an x" is called an existential quantifier.
2) Rule 2 : Existential Instantiation
The symbol "$" is used to denote the logical quantifier "there exists" or "there exists $ x p (x)
an x" or "there is x" or "for some x" or "for at least one x". , k is some element for which p(k) is true.
\ p ( k)
The existential quantifier for p(x) is denoted by $ x p(x).
3) Rule 3 : Universal Generalization
2.17.3 Negation of Quantified Statement px
" x p(x)
Consider the statement " x p(x). It's negation is "It is not the case that for all x, p(x) is
true". This means that for some x = a, p(a) is not true or $ x s.t. ~ p(x) is true. 4) Rule 4 : Existential Generalization
Hence the negation of " x p(x) is logically equivalent to $ x [~ p(x)]. p(k)
\ $ x p(x)
Sr. No. Statement Negation k is some element of the universe
1. " x p(x) $x [~ p(x)]
III)
2. $x [~ p(x)] " x p(x)

3. " x[~ p(x)] $x p(x) Sr. No. Quantifiers Expression

4. $x p(x) " x[~ p(x)] 1. $ x"y p(x, y) There exists a value of x such that for all values of y, p(x, y) is true.
2. "y $ x p(x, y) For each value of y, there exists x such that p(x, y) is true.
I) Equivalence involving quantifiers
3. $ x $y p(x, y) There exist value of x and value of y such that p(x, y) is true.
1) Distributivity of $ over Ú
4. "x "y p(x, y) For all values of x and y. p(x, y) is true.
$ x [p(x) Ú Q(x)] º $ x p(x) Ú $ x Q(x)
® ®
TECHNICAL PUBLICATIONS - An up thrust for knowledge TECHNICAL PUBLICATIONS - An up thrust for knowledge
Discrete Mathematics 2 - 35 Propositional Calculus Discrete Mathematics 2 - 36 Propositional Calculus

Example 2.17.1 Represent the arguments using quantifiers and find its correctness. Example 2.17.4 Determine the validity of the following argument

All students in this class understand logic. Ganesh is a student in this class. Therefore s 1 : All my friends are musician
Ganesh understands logic. SPPU : Dec.-11, Marks 4 s 2 : John is my friend

Solution : s 3 : Name of my neighbours are musician


s : John is not my neighbour.
Let C(x) : x is a student in this class
Solution : Let the universe of discourse be the set of people.
L(x) : x understands logic
In symbolic form Let F(x) : x is my friend
" x (C(x) ® L(x)) M(x) : x is a musician
C(a) N(x) : x is my neighbour
\ L(a)
It's symbolic form is
Here a means Ganesh s 1 : " x [F(x) ® M(x)]
This is Modus Ponen s 2 : F(a) (a = John]
s 3 : " x [N(x) ® ~ M(x)]
Therefore this argument is valid.
\ s : ~ N(a)
Example 2.17.2 Let p(x) : x is even
Suppose ~N(a) has value F.
Q(x) : x is a prime number
R(x, y) : x + y is even \ N(a) is T. Since s 3 is T, we must have ~M(a) is T or M(a) is F. But s 1 is T. Hence
we must have F(a) to be false but this is contradiction. Hence if s is false either of s 1 or
a) Using the information given above write the following sentences in symbolic form.
s 3 should be false. Hence argument is valid.
i) Every integer is an odd integer
ii) Every integer is even or prime Example 2.17.5 For the universe of all integers. Let

iii) The sum of any two integers is an odd integer. SPPU : Dec.-10, Marks 4
p(x) : x > 0
Q(x) : x is even
Solution :
R(x) : x is a perfect square
i) " x [~ p(x)]
S(x) : x is divisible by 4
ii) " x [p(x) Ú Q(x)] T(x) : x is divisible by 7
iii) " x " y [~ R(x, y)] Write the following statement in symbolic form
i) At least one integer is even
Example 2.17.3 Using information in example 2.17.2 write an english sentence for each of the
ii) There exists a positive integer that is even
symbolic statement given below
iii) If x is even then x is not divisible by 7
i) " x (~ Q ( x ))
iv) No even integer is divisible by 7
ii) $ y(~ p( y))
v) There exists an even integer divisible by 7
iii) ~ [$ x ( p( x ) Ù Q ( x ))] SPPU : Dec.-10, Marks 4
vi) If x is even and x is perfect square then x is divisible by 4.
Solution :
Solution :
i) All integers are not prime numbers
i) $ x Q(x)
ii) At least one integer is not even.
ii) $ x [ p(x) Ù Q(x)]
iii) It is not the case that there exists an integer which is even and prime.
iii) " x [Q(x) ® ~ T(x)]

® ®
TECHNICAL PUBLICATIONS - An up thrust for knowledge TECHNICAL PUBLICATIONS - An up thrust for knowledge

Discrete Mathematics 2 - 37 Propositional Calculus Discrete Mathematics 2 - 38 Propositional Calculus

iv) " x [Q(x) ® ~ T(x)] Example 2.17.7 Negate each of the following statements
i) " x, | x | = x
v) $ x [Q(x) Ù T(x)]
ii) $ x , x 2 = x SPPU : Dec.-09, 15, May-15, Marks 4
vi) " x [Q(x) Ù R(x) ® s(x)]
Solution :
Example 2.17.6 Rewrite the following statements using quantifier variables and predicate
i) $ x , x ¹ x ii) " x , x 2 ¹ x
symbols.
Example 2.17.8 Negate the following
i) All birds can fly
ii) Not all birds can fly i) If there is a riot, then someone is killed.

iii) Some men are genius ii) It is day light and all the people are arisen. SPPU : May-15, Dec.-15, Marks 4

iv) Some numbers are not rational Solution :

v) There is a student who likes Maths but not Hindi i) It is not the case that if there is a riot then someone is killed.
vi) Each integer is either even or odd SPPU : Dec.-08, Marks 4 ii) It is not the case that it is day light and all the people are arisen.
Solution : OR

i) Let B(x) : x is a bird i) Let p : There is a riot

F(x) : x can fly q : Someone is killed

Then the statement can be written as Given statement is p ® q

" x [B(x) ® F(x)] Hence ~ ( p ® q) º ~ (~ p Ú q) º p Ù ~ q

ii) $ x [B(x) Ù ~ F(x)] º There is a riot and someone is not killed.


iii) Let M(x) : x is a man ii) Let p : It is a day light
G(x) : x is a genius q : All the people are arisen
The statement in symbolic form as $ x [M(x) Ù G(x)] Given statement is p Ù q
iv) Let N(x) : x is a number Hence ~ (p Ù q) = ~ p Ú ~ q
R(x) : x is rational Hence either it is not a day light or all the people are not arisen.
The statement in symbolic form as $ x [N(x) Ù ~ R(x)] or ~ [" x (N(x) ® R(x)]
v) Let S(x) : x is a student qqq
M(x) : x likes Maths
H(x) : x likes Hindi
\ The statement in symbolic form as $ x [S(x) Ù M(x) Ù ~ H(x)]
vi) Let I(x) : x is an integer
E(x) : x is even
O(x) : x is odd
The statement in symbolic form as " x [I(x) ® E(x) Ú O(x)]

® ®
TECHNICAL PUBLICATIONS - An up thrust for knowledge TECHNICAL PUBLICATIONS - An up thrust for knowledge
Discrete Mathematics 3-2 Mathematical Induction

3.1 Introduction
Unit - I
Mathematical induction is a powerful technique in applied mathematics especially in
number theory, where many properties of natural numbers are proved by this method.
In day to day life, we are often required to generalise a particular pattern for the
prediction purpose. The generalisation is achieved by using a statement involving a

3
variable as natural number.
Mathematical Induction Mathematical induction is very useful technique or tool for the programmers to check
whether a program statement is loop invariant or not.
There are two principles of mathematical induction :
1) First principle of mathematical induction.
Syllabus 2) Second principle of mathematical induction.
Proof by Mathematical Induction and Strong Mathematical Induction.
3.2 First Principle of Mathematical Induction Statement
Contents Let P(n) be a statement involving a natural number n ³ n 0 such that,

3.1 Introduction 1) If P(n) is true for n = n 0 where n 0 £ N and


3.2 First Principle of Mathematical Induction Statement 2) Assume that P(k) is true for k = ³ n 0
3.3 Second Principle of Mathematical Induction Statement We prove P(k+1) is also true,
(Strong Mathematical Induction) . . . . . . . . . . . Dec.-04, 05, 06, 08, 10, 11,12, Then P(n) is true for all natural numbers n = n 0 .
. . . . . . . . . . . . . . . . . . 13, 14, 15, 18, May-05, 06, 07, 08, Step 1 is called as the basis of induction.
. . . . . . . . . . . . . . . . . . 14, 15, 17, 18, 19 · · · · · · · · · · Marks 6 Step 2 is called as the induction step.

3.3 Second Principle of Mathematical Induction Statement


(Strong Mathematical Induction) SPPU : Dec.-04, 05, 06, 10, 11, 12, 13,
14, 15, 18, May-05, 06, 07, 08, 14, 15, 17, 18, 19
Let P(n) be a statement involving a natural number n ³ n 0 such that,
1) If P(n) is true for n = n 0 where n 0 £ N and
2) Assume that P (n) is true for n 0 < n £ k i.e. P(n 0 + 1) ( n 0 + 2)............ P(k) are true.
we prove that P(k + 1) is true,
Then P(n) is true for all natural numbers n ³ n 0 .

Example 3.3.1 n( n + 1)
Prove that : 1 + 2 + 3 + 4 … + n =
2
SPPU : May-17, Dec.-18, Marks 4
Solution : Let, P(n) be the given statement

Consider the following steps,


Step 1 : Basis of induction

(3 - 1) ®
TM TECHNICAL PUBLICATIONS - An up thrust for knowledge
TECHNICAL PUBLICATIONS - An up thrust for knowledge

Discrete Mathematics 3-3 Mathematical Induction Discrete Mathematics 3-4 Mathematical Induction

For n = 1, L.H.S. = 1 Example 3.3.3 Show by induction that , n ³ 1


1(1 + 1) n( 2n - 1) ( 2n + 1)
R.H.S. = =1 1 2 + 3 2 + 5 2 + ..( 2 n - 1) 2 =
2 3 SPPU : May-14, Marks 4

\ For n = 1, L.H.S. = R.H.S. Solution : Let P(n) be the given statement,


\ P(1) is true.
1. Basis of induction :
Step 2 : Assume that P(k) is true
k(k + 1) For n = 1, L.H.S. = 1 2 = 1,
i.e. 1 + 2 + 3 + ...... + k = … (1)
2 1 (1) ( 3)
R.H.S. = =1
3
Consider, 1 + 2 + 3 + ...... + k + (k+ 1)
k(k + 1) k(k + 1) + 2(k + 1) (k + 1) + (k + 2) Þ L.H.S. = R.H.S.
= + k +1 = =
2 2 2
Hence P(1) is true
Hence P(k + 1) is true
2. Induction step : Assume that P(k) is true.
\ By the principle of mathematical induction P(n) is true for all n.
k (2k - 1) (2k + 1)
Example 3.3.2 Prove by mathematical induction for n ³ 1. i.e. 1 2 + 3 2 + 5 2 + ... + (2k - 1) 2 = ... (1)
3
n (n + 1) (n +2)
1× 2 + 2 × 3 + 3 × 4 + ... + n (n + 1) =
3 SPPU : May-05, Marks 6 Hence
Solution : Let P(n) the given statement k (2k - 1) (2 k + 1 )
[1 2 + 3 2 + 5 2 + ... + (2k - 1) 2 ] + (2k + 1) 2 = + (2k + 1) 2 ... (Using 1)
3
1. Basis of induction
(2k + 1)
For n 0 = 1 L.H.S. = 1.2 = 2 = [2k 2 - k + 3 (2k + 1)]
3
1( 2) ( 3) (2k + 1)
R.H.S. = = 2 Þ L.H.S. = R.H.S. = [2k 2 + 5k + 3]
3 3
Hence P(1) is true. (2k + 1)
= [2k 2 + 2k + 3k + 3]
3
2. Induction step
(2k + 1)
Assume that, P(k) is true = [(2k + 3) (k + 1)]
3
k (k + 1) (k + 2)
i.e. 1 × 2 + 2 × 3 + 3 × 4 +... + k (k + 1) = ... (1) (k + 1)(2 k + 1) (2k + 3)
3 =
3
Then we have
[1 × 2 + 2 × 3 + 3 × 4 +... + k (k + 1)]+ (k + 1) ( k + 2) (k + 1) [2 (k + 1) - 1][2 (k + 1) + 1]
=
3
k (k + 1) (k + 2)
= + (k + 1) (k + 2) … (Using 1)
3 Hence assuming P(k) is true P(k + 1) is also true. Therefore by mathematical
ék ù ( k + 1) ( k +2) (k +3) induction P(n) in true for all n ³ 1.
= (k + 1) (k + 2) ê + 1ú =
ë3 û 3 Example 3.3.4 n 2 (n + 1) 2
Show that 1 3 + 2 3 + 3 3 + ... + n3 = = (1 + 2 + 3 + ... + n) 2
4
Hence assuming P(k) is true, P(k + 1), is also true. Therefore by mathematical
SPPU : Dec.-12, May-18, Marks 4
induction P(n) is true for all n ³ 1.

® ®
TECHNICAL PUBLICATIONS - An up thrust for knowledge TECHNICAL PUBLICATIONS - An up thrust for knowledge
Discrete Mathematics 3-5 Mathematical Induction Discrete Mathematics 3-6 Mathematical Induction

Solution : Let P(n) be the given statement, Solution : Let P(n) be the given statement,
2
é n(n + 1) ù n 2 ( n + 1) 2 1. Basis of induction : For n = 1
1 3 + 2 3 + 3 3 + ... + n 3 = ê =
ë 2 úû 4 1 1
We have, L.H.S. = =
1.3 3
1. Basis of induction :
1 (2) 1
R.H.S. = =
For n = 1, L.H.S. = 1, 1(3) 3
1 ( 1 + 1) 2 Þ L.H.S. = R.H.S.
R.H.S. = =1
4
Hence P(1) is true.
Þ L.H.S. = R.H.S.
2. Induction step : Assume that P(k) is true
Hence P(1) is true.
12 22 k2 k (k + 1)
2. Induction step : Assume that P(k) is true. i.e. + + ... = ... (1)
1× 3 3× 5 (2k - 1) (2k + 1) 2 (2k +1)
k2 (k + 1) 2
i.e. 1 3 + 2 3 + 3 3 + ... + k 3 = … (1)
4 Then we have,
é 12 22 k2 ù (k + 1) 2
Then we have ê 1 × 3 + 3 × 5 + ... + ú+
ë (2k - 1) (2k + 1) û [2 (k + 1) - 1][2 (k + 1) + 1]
3 3 3 3 3 2 3
(1 + 2 + 3 + ... + k ) + (k + 1) = (1 + 2 + 3 + ... + k) + (k + 1) (Using 1)
2 k (k +1) (k +1) 2
æ k (k + 1) ö = + ... (Using 1)
= ç ÷ + (k + 1) 3 2 (2k +1) (2k +1) (2k +3)
è 2 ø
(k +1) é k (2k + 3) + 2 (k + 1) ù
é k2 ù =
= (k + 1) 2 ê + k + 1ú (2k +1) êë 2 (2k + 3) ú
û
ë 4 û
(k + 1) é 2k 2 + 5k + 2ù
é k 2 + 4k + 4 ù = ê ú
= (k + 1) 2 ê ú 2k + 1 ë 2 (2k + 3) û
êë 4 úû
(k + 1) é 2k 2 + 4k + k + 2ù
= ê ú
(k + 1) 2 (k + 2) 2 (2k + 1) ë 2 (2k + 3) û
=
4
2
(k + 1) é 2k (k + 2) + 1 (k + 2) ù
æ (k + 1)(k + 2) ö (k +1) 2 (k + 2) 2 =
(2k + 1) êë 2 (2k + 3) ú
= ç ÷ = û
è 2 ø 4
(k + 1) (k + 2)
=
Hence assuming P(k) is true, P(k + 1) is also true. Therefore by mathematical 2 (2k + 3)
induction P(n) is true for all n ³ 1.
(k + 1) [(k + 1) + 1]
=
Example 3.3.5 12 22 n2 n (n + 1) 2 [2(k + 1) + 1]
Show that + + ... + =
1× 3 3 × 5 (2n - 1) (2n + 1) 2(2n + 1)
\ P(k + 1) is true.
Therefore by mathematical induction P(n) is true for all n ³ 1.

® ®
TECHNICAL PUBLICATIONS - An up thrust for knowledge TECHNICAL PUBLICATIONS - An up thrust for knowledge

Discrete Mathematics 3-7 Mathematical Induction Discrete Mathematics 3-8 Mathematical Induction

1 1 1 1 n 1 1 1 1 n
Example 3.3.6 Show that a) + + + ... + = b) Let P(n) : + + + ... + =
1× 2 2 × 3 3 × 4 n (n + 1) n+ 1 1× 3 3× 5 5× 7 (2 n - 1)(2n + 1) 2n + 1
1 1 1 1 n
b) Show that + + + ... + =
1× 3 3× 5 5× 7 ( 2n - 1) ( 2n + 1) 2n + 1 1. Basis of induction : For n = 1
1 1 1 1 n 1
c) Show that + + + ... + = L.H.S. =
1
= , R.H.S. =
1
1 × 4 4 × 7 7 × 10 ( 3n - 1) ( 3n + 1) 3n + 1 1.3 3 3
SPPU : Dec.-05, Marks 6 1 1
=
Solution : Let P(n) be the given statement, 3 2.1+1
Þ L.H.S. = R.H.S.
a) 1. Basis of induction :
1 1 Hence P(1) is true.
For n = 1 L.H.S. = =
1.2 2
2. Induction step : Assume that P(k) is true
1 1
R.H.S. = = 1 1 1 1 k
1+1 2 i.e. + + + .... + = ... (1)
1× 3 3× 5 5× 7 (2k - 1)(2k + 1) 2k + 1
L.H.S. = R.H.S.
Then we have,
Hence P(1) is true.
é 1 1 1 ù 1
2. Induction step : Assume that P(k) is true. ê 1 × 3 + 3 × 5 + ... + (2k - 1)(2k + 1) ú + (2k + 1)(2k + 3)
ë û
1 1 1 1 k
i.e. + + + ... + = ... (1) =
k
+
1
1× 2 2× 3 3× 4 k (k + 1) k+ 1 2k + 1 (2k + 1)(2k + 3)
Then we have 2k 2 + 3k + 1
=
é 1 1 1 1 ù 1 k 1 (2k + 1) (2k + 3)
ê 1 × 2 + 2 × 3 + 3 × 4 + ... + k (k + 1) ú + (k + 1) (k + 2) = k + 1 + (k + 1)(k + 2) ... (Using 1)
ë û (2k + 1) (k + 1)
=
k (k + 2) + 1 (2k + 1)(2k + 3)
=
(k + 1) (k + 2) k +1
=
k 2 + 2k + 1 2k + 3
=
(k + 1) (k + 2) k+ 1
=
2(k + 1) + 1
(k + 1) 2
=
(k + 1) (k + 2) Hence assuming P(k) is true. P(k + 1) is also true. Therefore P(n) is true for all n ³ 1.
(k + 1) 1 1 1 1 n
k+ 1 c) Let P(n) : + + + ... + =
= = 1 × 4 4 × 7 7 × 10 (3n - 2)(3n + 1) 3n + 1
k + 2 (k + 1) + 1
1. Basis of induction
Hence assuming P(k) is true, P(k + 1) is also true. Therefore P(n) in true for all n ³ 1.
1 1 1
For n = 1, L.H.S. = = , R.H.S. =
1.4 4 4
1 1
=
1.4 3.1+1

® ®
TECHNICAL PUBLICATIONS - An up thrust for knowledge TECHNICAL PUBLICATIONS - An up thrust for knowledge
Discrete Mathematics 3-9 Mathematical Induction Discrete Mathematics 3 - 10 Mathematical Induction

Þ L.H.S. = R.H.S. Consider,


1 – a k+1
Hence P(1) is true. 1 + a + a 2 +....+ a k + a k+1 = + a k+1 … (Using 1)
1– a
2. Induction step : Assume that P(k) is true.
1 – a k+1 + (1 – a) a k+1
=
i.e.
1
+
1
+
1
+ ... +
1
=
k
... (1) 1– a
1 × 4 4 × 7 7 × 10 (3k - 2)(3k + 1) 3k + 1
1– a k+1 + a k+1 – a k+2
=
Then we have, 1– a
é 1 1 1 ù 1 1– a k+2
ê 1 × 4 + 4 × 7 + ... + (3k - 2)(3k + 1) ú + (3k + 1) (3k + 4) ... (Using 1) =
ë û 1– a
k 1
= + Hence P(k + 1) is true.
(3k + 1) (3k + 1)(3k + 4)
Therefore by the mathematical induction P(n) is true for all n ³ 0.
2
3k + 4k + 1
= Example 3.3.8 Use mathematical induction to show that n ( n 2 – 1) is divisible by 24. Where n
(3k + 1) (3k + 4)
is any odd positive number. SPPU : Dec.-14, Marks 4
(3k + 1) (k + 1)
= Solution : If n (n 2 –1) = n 3 – n is divisible by 24.
(3k + 1)(3(k + 1) + 1)
Then n3 – n = 24 (m) where m is any positive integral.
k +1
=
3(k + 1) + 1 Let P(n) be the given statement,

Hence assuming P(k) is true. P(k + 1) is also true. Therefore P(n) is true for all n ³ 1. 1. Induction step : For n = 1,

Example 3.3.7 1 – a n+ 1 n(n 2 – 1) = 0 which is divisible by 24.


Prove by induction for n ³ 0. 1 + a + a 2 +... + a n = .
1– a
SPPU : Dec.-10, Marks 4
( )
For n = 3, n n 2 - 1 = 24 which is divisible by 24.
\ P(1) and P(3) is true.
Solution : Let P(n) be the given statement,
2. Induction step :
1. Basis of induction
Assume that P(k) is true.
1– a
For n = 0, L.H.S. = 1, R.H.S. = =1
1– a i.e. k(k 2 – 1) = k 3 – k is divisible by 24.

1– a 2 \ k (k 2 – 1) = k 3 – k = 24 (m 0 ), m 0 ¬ z … (1)
For n = 1, L.H.S. = 1 + a, R.H.S. = =1+a
1– a
Consider
\ For n = 0, 1, L.H.S. = R.H.S.
(k +1) [( k +1) 2 – 1] = ( k +1) 3 – ( k +1)
Hence P(0), P(1) are true. = k 3 + 3k 2 + 3k + 1– k –1
2. Induction step : Assume that P(k) is true = k 3 + 3k 2 + 2k
1– a k+1 = (k 3 – k) +3k 2 + 3k … (Using 1)
\ 1 + a + a 2 +....+ a k = … (1)
1– a

® ®
TECHNICAL PUBLICATIONS - An up thrust for knowledge TECHNICAL PUBLICATIONS - An up thrust for knowledge

Discrete Mathematics 3 - 11 Mathematical Induction Discrete Mathematics 3 - 12 Mathematical Induction

= 24 m 0 + 3k (k +1) (As k(k + 1) is multiple of 8 for k Solution : Let P(n) be the given statement.
odd positive integer and k ³ 3)
1) Basis of induction : For n = 1,
= 24m 0 + 3 ( 8 m 1 )
7 2 + 2 0 × 3 0 = 49 + 1 = 50, which is divisible by 25.
\ P(1) is true.
= 24 ( m 0 + m 1 )
2) Induction step :
= 24 m 2 (Q m 0 + m 1 = m 2 )
Assume that P(k) is true.
\ P(k + 1) is true.
i.e. 7 2k + 2 3k– 3 3 k– 1 = 25 (m)
\ By mathematical induction P(n) is true for all n odd positive number.
Consider
Example 3.3.9 Show that n 4 - 4 n 2 is divisible by 3 for all n ³ 2. SPPU : Dec.-15, Marks 4
7 2(k+1) + 2 3(k+1)– 3 × 3 k+1– 1 = 7 2k+2 + 2 3k 3 k
Solution : Let P(n) be the given statement,
= 497 2k + 2 3k × 3k
1. Basis of induction
= 49[25 m – 2 3k– 3 × 3 k– 1 ] + 2 3k × 3 k
For n = 2 = 25 (49m) – 49 2 3k– 3 × 3 k– 1 + 2 3k– 3 × 3 k– 1 ( 8 ´ 3)
2 4 - 4 ( 2 2 ) = 16 – 16 = 25 (49m) + 2 3k– 3 × 3 k– 1 (– 49 + 24)
= 0 is divisible by 3 as 0 is divisible by every number = 25(49 m) + 2 3k– 3 3 k– 1 ( –25)

\ P(2) is true. = 25 [ 49 m – 2 3k– 3 × 3 k– 1 ]


= 25 (P) : P Î N
2. Induction step : Assume that P(k) is true
\ P(k+1) is true.
i.e. k 4 - 4k 2 is divisible by 3
\ By mathematical induction, P(n) is true for all n.
Then we have,
Example 3.3.11 Using mathematical induction, prove that
(k + 1) 4 - 4(k + 1) 2 = k 4 + 4k 3 + 6k 2 + 4k + 1 - 4 (k 2 + 2k + 1) n( n + 1)
1 2 - 2 2 + 3 2 - 4 2 + ... + ( - 1) n - 1 n2 = ( - 1) n - 1
= (k 4 - 4k 2 ) + 4 (k 3 + 2k) + 6k 2 + 12k - 3 2 SPPU : Dec.-04, Marks 6

4 2 Solution : Let P(n) be the given statement,


k - 4 k is divisible by 3.
k 3 + 2k is divisible by 3 1. Basis of induction

Also 6 k 2 + 12 k - 3 = 3 (2k 2 + 4k - 1) is divisible by 3. For n = 1 L.H.S = 1, R.H.S. = 1

Hence (k + 1) 4 - 4 (k + 1) 2 is divisible by 3. Þ L.H.S. = R.H.S.

Hence assuming P(k) is true. Hence P(1) is true.


P(k + 1) in also true. Therefore P(n) is true for n ³ 2. 2. Induction step : Assume that P(k) in true.
Example 3.3.10 Prove that : 7 2n + 2 3n– 3 × 3 n– 1 is divisible by 25 "n. k(k + 1)
i.e. 1 2 - 2 2 + 3 2 - 4 2 + ... + ( - 1) k - 1 k 2 = ( - 1) k - 1 ... (1)
SPPU : May-19, Marks 3 2
Then we have,
[1 2 - 2 2 + 3 2 - 4 2 + ... + ( - 1) k - 1 k 2 ] + ( - 1) k (k + 1) 2 ... (Using 1)

® ®
TECHNICAL PUBLICATIONS - An up thrust for knowledge TECHNICAL PUBLICATIONS - An up thrust for knowledge
Discrete Mathematics 3 - 13 Mathematical Induction Discrete Mathematics 3 - 14 Mathematical Induction

k(k + 1) 2. Induction step : Assume that, P(k) is true.


= ( - 1) k - 1 + ( - 1) k (k + 1) 2
2
i.e. k5 - k is divisible by 5
é k ù
= ( - 1) k (k + 1) ê - + (k + 1)ú
ë 2 û Then we have
é - k + 2k + 2ù (k + 1) 5 - (k + 1) = (k5 + 5 C 1 k 4 + 5 C 2 k 3 + 5 C 3 k 2 + 5 C 4 k + 5 C5 ) - (k + 1)
= ( - 1) k (k + 1) ê ú
ë 2 û
= k5 + 5 k 4 + 10 k 3 + 10 k 2 + 5k + 1 - k - 1
k (k + 1)(k + 2)
= ( - 1)
2 = (k5 - k) + 5 [k 4 + 2k 3 + 2k 2 + k]
Hence assuming P(k) is true, P(k + 1) is also true. Therefore P(n) in true for all n ³ 1.
k5 - k is divisible by 5.
Example 3.3.12 Prove by mathematical induction that for n ³ 1 : and 5 (k 4 + 2k 3 + 2k 2 + k) is divisible by 5.
1× 1! + 2 × 2! + 3 × 3 ! + ... + n × n ! = (n + 1) ! - 1. SPPU : May-08, 15, Marks 6
Hence (k + 1) 5 - (k + 1) is divisible by 5.
Solution : Let P(n) be the given statement,
Hence assuming P(k) is true. P(k + 1) is also true. Therefore P(n) is true for n ³ 1.
1. Basis of induction
Example 3.3.14 Prove that 8 n - 3 n is a multiple of 5 by mathematical induction for n ³ 1.
For n = 1, L.H.S = 1, R.H.S. = 1 SPPU : May-06, 07, Dec.-13, Marks 6

Þ L.H.S. = R.H.S. Solution : Let P(n) be the given statement,

Hence P(1) is true. 1. Basis of induction :


2. Induction step : Assume that, P(k) is true. For n = 1 81 - 31 = 5
i.e. 1 × 1! + 2 × 2! + 3 × 3! + ... + k × k! = 5× 1
= (k + 1)! – 1 ... (1)
Then we have, Obviously a multiple of 5.
[1 × 1! + 2 × 2! + 3 × 3! + ... + k × k!] + (k + 1) × (k + 1)! ... (Using 1) \ P(1) is true.
= [(k + 1)! – 1] + (k + 1) × (k + 1)!
2. Induction step : Assume that, P(k) in true.
= (k + 1)! + (k + 1) × (k + 1)! – 1
i.e. 8 k - 3 k is multiple of 5 say 5 r
= (k + 1)! [k + 1 + 1] – 1
i.e. 8k - 3k = 5 r ... (1)
= (k + 2) (k + 1)! – 1
where r is an integer
= (k + 2)! – 1
Then we have,
Hence assuming P(k) is true, P(k + 1) is also true. Therefore P(n) is true for n ³ 1.
8k + 1 - 3k + 1 = 8k × 8 - 3k × 3
Example 3.3.13 Prove that for any positive integer n the number n 5 – n is divisible by 5.
= 8 k × (5+ 3) - 3 k × 3
SPPU : Dec.-08, Marks 6
Solution : Let P(n) be the given statement, = 8 k × 5+ (8 k × 3 - 3 k × 3)

1. Basis of induction : = 8 k × 5+ 3(8 k - 3 k )

For n = 1, 15 - 1 = 0 is divisible by 5. Obviously 8 k × 5 is multiple of 5 and also 8 k - 3 k is multiple of 5.


As 0 is divisible by every number. Therefore, 8 k + 1 - 3k + 1 is multiple of 5.
Hence P(1) is true. Hence assuming P(k) is true, P(k + 1) is also true. Therefore P(n) is true for all n ³ 1.

® ®
TECHNICAL PUBLICATIONS - An up thrust for knowledge TECHNICAL PUBLICATIONS - An up thrust for knowledge

Discrete Mathematics 3 - 15 Mathematical Induction Discrete Mathematics 3 - 16 Mathematical Induction

Example 3.3.15 Show that the sum of the cubes of three consecutive natural number is Basis of induction : For n = 1, P(1) implies that any 2 ´ 2 chessboard with one square
divisible by 9. SPPU : Dec.-06, Marks 6 removed can be covered using L shaped pieces. P(1) is true, as seen below.
Solution : Let n, n + 1, n + 2 be three consecutive natural numbers.
3
We have to show that n + (n + 1) + (n + 2) 3 3 is divisible by 9.
Let P(n) be the above statement,
1. Basis of induction : For n = 1

1 3 + 2 3 + 3 3 = 1 + 8 + 27 = 36 which is divisible by 9.

\ P(1) is true.

2. Induction step : Assume that P(k) is true.


i.e. k 3 + (k + 1) 3 + (k + 2) 3 in divisible by 9.
Then we have,
Induction step : Assume that, P(k) is true i.e. any 2 k ´ 2 k chessboard with one square
(k + 1) 3 + (k + 2) 3 + (k + 3) 3 = [(k +1) 3 + (k +2 ) 3 ] +[k 3 + 3 C 1 k 2 (3) + 3 C 2 k(3) 2 + 3 C 3 (3 3 )]
removed can be covered using L-shaped pieces.
= (k + 1) 3 + (k + 2) 3 + k3 + [9 k2 + 27 k + 27] Then, we have to show that P(k + 1) is true. For this consider, a 2 k + 1 ´ 2 k + 1
= [k 3 + (k + 1) 3 + (k +2) 3 ] + 9 [k 2 + 3k + 3] chesshoard with one square removed. Divide the chessboard into four equal halves of
size 2 k ´ 2 k , as shown below.
k 3 + (k + 1) 3 + (k + 2) 3 is divisible by 9 and 9 (k 3 + 3k + 3) is divisible by 9.
Þ (k + 1) 3 + (k + 2) 3 + (k + 3) 3 is divisible by 9. 2
k
2
k

C1
Hence assuming P(k) in true, P(k + 1) is also true. Therefore P(n) is true for all n ³ 1. k C2 k
2 2
Example 3.3.16 æ 5 n+1 – 1 ö
Using mathematical induction prove that 3 + 3.5 + 3.5 2 +.....+3.5 n × ç ÷. k C3 C4 k
è 4 ø 2 2

For non-negative number n. SPPU : Dec.-11, Marks 6 2


k
2
k

Solution : Cancelling 3 from the both sides of given.


The square which has been removed, would have been removed from one of the four
Statement, We get chessboards, say C 1 . Then by induction hypothesis, C 1 can be covered using L-shaped
n +1
5 –1 pieces. Now, from each of the remaining chessboards, remove that particular piece (or
1 + 5 + 5 2 +....+5 n = … (1)
5–1 tile), lying at the centre of the large chessboards.

Let P(n) be the above statement. k k


2 2
To prove this refer example 3.3.7 for a = 5. C2 C1
k k
2 2
Example 3.3.17 Let n be a positive integer. Show that any 2 n ´ 2 n chessboard with one
square removed can be covered by L-shaped pieces, where each piece covers three squares at 2
k
C3 C4 2
k

a time.
k k
2 2
Solution : Let P(n) be the proposition that any 2n ´ 2n chessboard with one square
removed can be covered using L-shaped pieces.

® ®
TECHNICAL PUBLICATIONS - An up thrust for knowledge TECHNICAL PUBLICATIONS - An up thrust for knowledge
Discrete Mathematics 3 - 17 Mathematical Induction Discrete Mathematics 3 - 18 Mathematical Induction

Then by induction hypothesis, each of these 2 k ´ 2 k chessboards with a piece (or tile) would inform the king on the first hour. (when the king returned for the first time) that
removed can be covered by the L-shaped pieces. Also the three tiles removed from the she was wearing a white hat.
centre can be covered by one L-shaped piece. Hence the chessboard of 2 k + 1 ´ 2 k + 1 can Now let n = 2, i.e. two mathematicians wearing white hats.
be covered by L-shaped pieces. Consider, one of these two mathematicians. She saw that one of her colleagues was
Hence proved. wearing a white hat. She reasoned that if she were wearing a black hat, her colleague
would be the only one wearing a white hat. In that case, her colleague would have
Example 3.3.18 Suppose we have unlimited stamps of two different denominations, 3 rupees
figured out the situation and informed the king on the first hour. That did not happen
and 5 rupees. We want to show that it is possible to make up exactly any postage of 8 shows that she was also wearing a white hat. Conesquently she told the king on the
rupees or more using stamps of these two denominations. second hour (and so did the other mathematician with a white hat, since all the
Solution : For k = 8, we have one 5 rupees stamp and one 3 rupees stamp. mathematicians are smart).
2. Induction step : Assume that, if there were k mathematicians wearing white hats,
For k = 9, replace 5 rupees stamp by two 3 rupees stamp, similarly for k = 10, then they would have figured out that they were wearing white hats and informed the
replace all 3, 3 rupees stamps by, two 5 rupees stamp an so on. king so on the k th hour. Now, suppose that there were k + 1 mathematicians wearing
Hence let us assume that, it is possible to make up k rupees stamp using 3 rupees white hats.
and 5 rupees stamps (for k ³ 8). Every mathematician wearing a white hat saw that k of her colleagues were wearing
Now we have to show that it is also possible to make up (k + 1) rupees stamps using white hats. However, that her k colleagures did not inform the king of their findings on
3 rupees and 5 rupees stamps. the k th hour can only imply that there were more then k. People wearing white hats.
We examine two cases : Consequently she knew that, she must be wearing a white hat also on the (k + 1) th hour.
She (together with all other mathematicians wearing white hats) would inform the king
1) Suppose we make up stamps of k rupees using at least one 5 rupees stamp.
their conclusion.
Replacing a 5 rupees stamp by two 3 rupees stamp, we can make up k + 1
rupees stamps. qqq
2) Suppose we make up a stamp of k rupees using 3 rupees only. Since k ³ 8 we
must have at least 3, 3 rupees stamps. Replacing these 3, 3 rupees stamps by two
five rupees stamps. We can make up stamps of k + 1 rupees.
Hence proved.

Example 3.3.19 The king summoned the best mathematicians in the kingdom to the palace to
find out how smart they were. The king told them
"I have placed white hats on some of you and black hats on the others. You may look at,
but not talk, to, one another. I will leave now and will come back every hour on the hour.
Every time I return, I want those of you who have determined that you are wearing white
hats to come up and tell me immediately."
As it turned out, at the nth hour every one of the n mathematician who were given white
hats informed the king that she knew that she was wearing a white hat. Why ?
Solution :
1. Basis of induction : For n = 1, there is only one mathematician wearing a white
hat. Since the king said that white hats were placed on some one of the mathematician
(king never lie). The mathematician who saw that all other mathematicians had on black
hats would realize immediately that she was wearing a white hat. Consequently she
® ®
TECHNICAL PUBLICATIONS - An up thrust for knowledge TECHNICAL PUBLICATIONS - An up thrust for knowledge

Discrete Mathematics 4-2 Relations

4.1 Introduction
Unit - II
In the chapter 1, we dealt with sets, elements, different types and general properties
of sets. There may exists various relationships among elements of sets. We will study
relations and functions with various properties and examples.
Relation may involves equality or inequality of elements. In mathematics the

4 Relations
expressions like '' is equal to " is similar to ", " is greater than", "is parallel to " are some
relations.

4.2 Cartesian Product


Let A and B be two non empty sets. The cartesian product of A and B is denoted by
Syllabus A ´ B and defined as

Relations and their Properties, n-ary relations and their applications, Representing relations, A ´ B = {(a, b) / a Î A and b Î B}
Closures of relations, Equivalence relations, Partial orderings, Partitions, Hasse diagram,
e.g. If A = {1, 2, 3, 4}, B = {a, b} then
Lattices, Chains and Anti-Chains, Transitive closure and Warshall‘s algorithm.
A ´ B = {(1, a), (1, b), (2, a), (2, b), (3, a) (3, b) (4, a) (4, b)}
Contents B ´ A = {(a, 1), (a, 2), (a, 3), (a, 4), (b, 1), (b, 2), (b, 3), (b, 4)}

4.1 Introduction (a, b) Ï A ´ B or B ´ A

4.2 Cartesian Product and (1, a) ¹ (a, 1)

4.3 Relation Hence A´B ¹ B ´ A


4.4 Matrix Representation of a Relation So the cartesian product is not commutative.
4.5 Diagraphs Similarly,
4.6 Special Types of Relations A ´ B ´ C = {(a, b, c) / a Î A and b Î B and c ÎC}
4.7 Types of Relations on Set . . . . . . . . . . . . . . . . Dec.-10, 11, 12, 14, 18, 19, In general
. . . . . . . . . . . . . . . . . . May-14, 17, 18, 19, · · · · · · · Marks 6
A1 ´ A 2 ´ ......´ A n = {a 1, a 2 , a 3 ........... , a n ) / a 1 Î A1 and a 2 Î A 2 , .... and a n Î A n )}
4.8 Partitions of a Set . . . . . . . . . . . . . . . . . . Dec.-11, 12, May-17, · · · · · · Marks 3
4.9 Closure of a Relation . . . . . . . . . . . . . . . . . . Dec.-05, 07, 12, 13, 14, 15, 16 Theorem 1 : If |A| = m and |B| = n then |A ´ B| = |B ´ A| = mn

. . . . . . . . . . . . . . . . . . May-06, 07, 08, 15, 18, · · · · Marks 6 Proof : The proof is trivial by set theory.
4.10 Partially Ordered Set . . . . . . . . . . . . . . . . . . Dec.-06 Theorem 2 : If A, B, C are non empty sets then A Í B Þ A ´ C Í B ´ C
4.11 Principle of Duality . . . . . . . . . . . . . . . . . . Dec.-10, 11, 13, 14, 15, 16,
Proof : Let (x,y) be any element in A ´ C, then
. . . . . . . . . . . . . . . . . . May-14, 15, 19 · · · · · · · · · · Marks 6
Þ x Î B and y ÎC
Þ (x, y) Î B ´ C
Hence A ´ C Í B ´ C

Theorem 3 : If A, B, C, are sets then

(4 - 1) ®
TM TECHNICAL PUBLICATIONS - An up thrust for knowledge
TECHNICAL PUBLICATIONS - An up thrust for knowledge
Discrete Mathematics 4-3 Relations Discrete Mathematics 4-4 Relations

i) A ´ (B Ç C) = (A ´ B) Ç ( A ´ C) The range of R is the set of all second co-ordinates of the ordered pairs (a,b) Î R.
ii) (A Ç B) ´ C = (A ´ C) Ç (B ´ C) It is denoted by R (R) = {(b/(a, b) Î R}.
5) The null set is the subset of A ´ B .
iii) A ´ (BÈ C) = (A ´ B ) È (A ´ C)
\ f is a relation called null relation or empty relation.
iv) (A È B) ´ C = (A ´ C) È (B ´ C)
Example :
Proof :
Let A = {1, 2, 3}, B = {x, y}
i) A ´ (B Ç C) = {(x, y) / x Î A and (y Î B Ç C) }
then R 1 = {1, x) (1, y), (3, x)} is a relation from A to B
= {(x, y) / x Î A and / (y Ï B and y Î C)}
\ D (R 1 ) = {1,3}
= {(x, y) / x Î A, y Î B and y Î A, y ÎC}
R (R 1 ) = {x, y}
= {(x, y) / (x, y) Î A ´ B and (x, y) Î A ´ C}
4.4 Matrix Representation of a Relation
= {(x, y) / (x, y) Î (A ´ B) Ç (A ´ C)}
Let A = {a 1 , a 2 , a 3 .... a n } , B = {b1 , b 2 , b 3 , .... b m } and R Í A ´ B . Then the relation
A ´ (B Ç C) = (A ´ B) Ç (A ´ C)
matrix of R is denoted by M R = [ m ij ] n ´ m and defined by
Similarly students can prove remaining results.
ì0 if (a i , b j ) Ï R i.e. a i Rb j
m ij = í
Note : If  is the set of real numbers then î1 if (a i , b j ) Î R i.e a i R b j
 ´  =  2 ® Euclidean plane
Example 4.4.1 Let A = {1, 2, 3, 4} and R = {(x, y)/ x < y } then find M R .
 ´  ´  =  3 ® 3D space
Solution : R = {(1, 2) (1, 3) (2, 3) (1, 4) (2, 4) (3, 4) }
4.3 Relation 1 2 3 4

Let A and B be two non empty sets. A relation from A to B is any subset of A ´ B. It é0 1 1 1ù
\ M R = [ M ij ] 4 ´ 4 = ê 0 0 1 1ú
is denoted by R : A ® B ê ú
ê0 0 0 1ú
e.g. Let A = {x,y,z), B = {1, 2, 3) ê
ë0 0 0 0úû
then A´B = {(x, 1), (x, 2), (x, 3), (y, 1), (y, 2), (y, 3), (z, 1), (z, 2) (z, 3)}
Example 4.4.2 If A = {1, 2, 3, 4, 5, 6} and a R b iff a divides b for a, b Î A. Find relation
R 1 = {(x,1), (y, 2), (z,1), )}, R 2 = {(z, 3)}, R 3 = { f }
matrix.
are relations from A to B Solution : R = {(1, 2} (1, 3), (1, 4), (1, 5), (1, 6), (2, 4) (2, 6), (3, 6), (1, 1) (2, 2), (3, 3),
But R 4 = {(1, x), (2, x)}is not relation from A to B. R 4 is the relation from B to A. (4, 4), (5, 5) (6, 6)}

Important Results : 1 2 3 4 5 6

1) If (a, b) Î R then it is denoted by a R b. é1 1 1 1 1 1ù


ê0 1 0 1 0 1ú
2) If R is a relation from A to B then R Í A ´ B ê ú
\ Relation Matrix = M R = ê 0 0 1 0 0 1ú
3) If R Í A ´ B then R is a relation from A to A and R is called a relation on A. ê0 0 0 1 0 0ú
ê ú
4) If R is a relation from A to B, then the set of all first elements of the ordered ê0 0 0 0 1 0ú
pairs (a,b) Î R is called the domain of R. It is denoted by D(R) = {a/(a, b) Î R}. ê0 0 0 0 0 1úû 6 ´ 6
ë

® ®
TECHNICAL PUBLICATIONS - An up thrust for knowledge TECHNICAL PUBLICATIONS - An up thrust for knowledge

Discrete Mathematics 4-5 Relations Discrete Mathematics 4-6 Relations

Example 4.4.3 A = {x, y, z} Find the relation matrices of the following relations. é1 1 1ù
i) R 1 = {(x, x) (y, y) (z, z)} ii) R 2 = {(x, y) (y, x), (y, z), (z, y)} iii) R 3 = {(x, x) and A × B = ê1 1 1ú
ê ú
(x, y), (x, z) (y, x) (y, y) (y, z), (z, x) (z, y) (z, z) êë1 1 1úû
Solution : (5,5) (6,6)
4.4.2 Properties of Relation Matrix
x y z
1 0 0ù Let R 1 be a relation matrix from A to B and R 2 be a relation matrix from B to C
i) M R1 = é
ê0 1 0ú then relation matrices satisfy the following properties :
ê ú
êë0 0 1úû 3 ´ 3 i) M R1 × R = M R1 × M R 2
2
x y z ii) M = Transpose of M R1
R –1
1
0 1 0ù
ii) M R2 = é iii) M = M M
ê1 0 1ú (R1R 2 ) –1 R –1 R –1
2 1
ê ú
êë0 1 0úû 4.5 Diagraphs

é1 1 1ù A relation can be represented pictorially by drawing its graph. Let A be any non
iii) M R 3 = ê1 1 1ú empty set and R be a relation on A. R can be represented by graphically by the
ê ú
êë1 1 1úû following procedure.
i) The elements of a set A are represented by small circles or point i.e. (o) or ...
4.4.1 Relation Matrix Operations These elements are called as vertices or nodes.

We know that relation matrix is a boolean matrix as all entries are either 0 or 1 ii) If (a, b) Î R or a R b then vertices a and b are joined by a continuous arc with an
arrow from a to b. These arcs are known as edges of the graph i.e. if aRb then
I) Let A = [ a ij ] m ´ n , B = [b ij ] m ´ n be two relation matrices then
A + B = [ a ij + b ij ] = [c ij ] m ´ n
a b
where c ij = 1 if a ij = 1, or b ij = 1
= 0 if a ij = 0, and b ij = 0 iii) If aRa then the vertex a is joined to itself by a loop around a. e.g. if aRa then

II) Let A = [ a ij ] m ´ n and B = [b jk ] n ´ k then A × B = [ a ij ; b jk ] = [d jk ] a

where d jk = 1 if a ij = 1 and b jk = 1
= 0 if a ij = 0 or b jk = 0 This graphical representation of the relation is called as diagraph or directed graph of
R. Let us consider the following examples.
é1 1 1ù é1 1 1ù
e.g. If A = ê 0 1 1ú , B = ê 0 1 1ú 1) aRa 2) aRb
ê ú ê ú
êë 1 1 0úû êë 0 0 0úû a b

é1 1 1ù a
then A + B = ê 0 1 1ú
ê ú
êë1 1 0úû 3) bRa 4) aRb Ù bRa
a b a b

® ®
TECHNICAL PUBLICATIONS - An up thrust for knowledge TECHNICAL PUBLICATIONS - An up thrust for knowledge
Discrete Mathematics 4-7 Relations Discrete Mathematics 4-8 Relations

5) aRa Ù aRb Ù bRa Ù bRb Example 4.5.2 é1 0 0 1ù


ê0 1 0 1ú
Let A = {a, b, c, d} and M R =ê ú Draw diagraph of R.
ê1 1 0 0ú
ê1 0 0 0úû
a b ë
Solution : We have
6) aRb Ù bRc Ù CRa Ù aRa
b
R = {(a,a) (a,d) (b,b) b,d) (c,a) (c,b) (d,a)}
The digraph of R is as follows :
a

7) aRa Ù bRb Ù cRc


a b

a b c

8) aRb Ù bRa Ù aRc Ù cRa a b

d c

Fig. 4.5.2

c Example 4.5.3 Find the relation from the diagraph.

Examples :
Example 4.5.1 A = {1,2,3,4,5,6} and a R b iff a/b (a divides b). Draw diagraph of R.

Solution : We have 1
ì(1,1) ( 2, 2) ( 3, 3) ( 4 , 4) ( 5, 5) ( 6, 6)ü
ï ï
R = í (1, 2) (1, 3) (1, 4) (1, 5) (1, 6) ý
ï ï 2
î ( 2, 4) ( 2, 6) ( 3, 6) þ
The diagraph of R is as follows :
3

1 4
2
Fig. 4.5.3
6
3 Solution : From the given diagraph, the corresponding relation on A = {1, 2, 3, 4} is

R = {(1, 1) (1, 3) (3, 1) (2, 3) (3, 2), (2, 4) (4, 2) (4, 4) (3, 4) (4, 3)}
5 4

Fig. 4.5.1

® ®
TECHNICAL PUBLICATIONS - An up thrust for knowledge TECHNICAL PUBLICATIONS - An up thrust for knowledge

Discrete Mathematics 4-9 Relations Discrete Mathematics 4 - 10 Relations

4.6 Special Types of Relations

4.6.1 Inverse Relation (OR Converse Relation)


1 2
Let R be a relation from A to B. The inverse relation of R is denoted by R –1 is a
relation from B to A defined as
R –1 = {(y, x) | y Î B, x Î A and (x,y) Î R}

i.e. if x R y then yR –1 x
It is also denoted by R c 4 3

It is also known as the converse relation.


Fig. 4.6.2
Example : Theorem 1 : If R, R 1 , R 2 are relations from A to B then
1) If R = {(1,2) (2,3) (3,1) (4,5) } then R –1 or R c = {(2,1) (3,2) (1,3) (5,4)} i) (R –1 ) –1 = R
é1 0 1 1ù é1 1 0 0ù
ii) (R 1 È R 2 ) -1 = R –1 È R 1–1 = R 1–1 È R –1
ê1 2 2
0 1 1ú ê0 0 0 0ú
2) If M R =ê ú then M –1 = ê ú = [M R ] t iii) ( R 1 Ç R 2 ) -1 = R 1–1 Ç R –1
ê0 0 1 1ú R ê1 1 1 0ú 2
ê0 ú
0 0 1û ê1 1 1 1úû
ë ë 4.6.2 Complement of a Relation
3) If the diagraph R is given by
Let R be a relation from A to B. The complement of R is denoted by R or R ¢ : A ® B
\ R = {(1,1) (1,2) (2,3) (4,2) (4,1) (4,3) (3,4) } and defined as R = {(x, y) | (x, y) Ï R, x Î A, y Î B}
i.e. x R y iff x R/ y

Note R is the complement ary set of R w.r.t. universal domain A ´ B.

Examples :
1 2
Example 4.6.1 Let A = {1, 2, 3, 4}, B = { a, b},
A ´ B = {(1, a) (1, b) (2, a) (2, b) (3, a) (3, b) (4, a) (4, b)},
R = {(1, a) (2, a) (3, a) (4, a)},
S = {4, a) (4, b) (3, a) (3, b)}.
Find R , S, R Ç S, R È S.
4 3
Solution :
Fig. 4.6.1 R = {(1,b) (2,b) (3,b) (4,b)}

then the diagraph of R –1 is obtained by changing the direction of arrow only which S = {(1, a) (1, b) (2, a) (2, b)}
is given below. R Ç S = {(1, b) (2, b)}
R È S = {(1, a) (1, b) (2, a) (2, b) (3, b) (4, b)}

® ®
TECHNICAL PUBLICATIONS - An up thrust for knowledge TECHNICAL PUBLICATIONS - An up thrust for knowledge
Discrete Mathematics 4 - 11 Relations Discrete Mathematics 4 - 12 Relations

Example 4.6.2 If A = {a, b, c, d} and R = {(a, b) (c,d) (c,c) (d,a) (a,a) (b,b) (d,d) } is a relation i.e. x ® y y®x x®x
on A. Draw diagraph of R . y®y x®y

Solution : 2) (x, x) (x, x) (x, x)


R = {(a, c) (a, d) (b, a) (b, c) (b, d) (c, a) (c, b) (d, b) (d, c)} 3) (y, z) - -

\ The diagraph of R is as follows : 4) (z, w) (w, w) (z, w)

a b
\ R 1 × R 2 = {(x, x)}, (x, y), (z, w)} x y
The diagraph of R 1 × R 2 is,
w z

Fig. 4.6.4
d c
Example 4.6.5 Let A = (1, 2, 3, 4). Let R 1 = {(x, y)/ x+y = 5} and R 2 = (x, y) / (y – x) = 1}
Fig. 4.6.3 verify ( R 1 × R 2 ) C = R C × R 1C
2
Example 4.6.3 é1 0 1ù Solution : We have A = {1, 2, 3, 4}
Obtain the matrix of R if M R = ê 0 0 1ú
ê ú Rl = {(1,4) (2,3), (3,2) (4,1)}
êë1 1 0úû
Solution : The matrix of R is obtained from M R be replacing 0 and 1 by 1 and 0 R 2 = {(1,2) (2,3) (3,4)}
respectively R 1 × R 2 = {(2,4) (3,3) (4,2)}
é0 1 0ù
M R = ê1 1 0ú (R 1 × R 2 ) C = {(4,2) (3,3) (2,4)}
ê ú
êë 0 0 1úû
R 1C = {(4,1), (3,2), (2,3), (1,4)}
Note : 1) (R 1 È R 2 ) = R 1 Ç R 2 RC = {(2,1), (3,2) (4,3)}
2
2) (R 1 Ç R 2 ) = R 1 È R 2 R C × R C = {(2,4), (3,3) (4,2)} = ( R 1 × R 2 )
2 1
4.6.3 Composite Relation Hence ( R 1 × R 2 ) C = RC × RC
2 1
Let R 1 : A ® B and R 2 : B ® C be two relations.
4.7 Types of Relations on Set
The composition of R 1 and R 2 is denoted by SPPU : Dec.-10, 11, 12, 14, 18, 19, May-14, 17, 18, 19
R 1 o R 2 or R 1 R 2 : A ® C defined as
Let R be a relation from A to A. Then we can say that R is a relation on set A.
R 1 × R 2 = {(x,z) | x R 1 y, yR 2 z i.e. (x, y) Î R 1 and (y, z) Î R 2 for x Î A, z Î C }
\ R Í A ´ A.
Examples :
A ´ A is the universal set for R. If R = A ´ A then R is called the Universal Relation on
Example 4.6.4 Let A = (x, y, z, w) and R 1 = {(x, y) (x, x) (y, z) (z, w) }, R 2 = {(x, x) A.
(y, x) (y, y) (w, w) }, Find R 1 , R 2 and diagraph of R 1 × R 2 .
Solution : 4.7.1 Identity Relation

In R 1 In R 2 R1 × R2 Let A be any set. The relation on A is called Identity relation if


1) (x, y) (y, x) (x, x) I A = {(a, a) | for a Î A}
(y, y) (x, y)

® ®
TECHNICAL PUBLICATIONS - An up thrust for knowledge TECHNICAL PUBLICATIONS - An up thrust for knowledge

Discrete Mathematics 4 - 13 Relations Discrete Mathematics 4 - 14 Relations

e.g. If A = {1, 2, 3, 4} then 4.7.3 Symmetric Relation


R 1 = {(1, 1) (2, 2)}, R 2 = {(4, 4)} are identity relations. Let R be a relation on set A. R is said to be symmetric relation if whenever a R b
The diagraph of the identity relation is the graph with loops only and it's relation then b Ra for a, b Î A.
matrix is the diagonal matrix with 0 and 1. Notes :
1) A relation is symmetric iff R = R –1
4.7.2 Reflexive Relation
2) The relation matrix of symmetric relation is a symmetric matrix.
1) Let R be a relation on set A. A relation R is said to be reflexive relation if
(a, a) Î R, " a Î A. Example : Let A = {1,2, 3, 4}. then
2) A relation R is said to be not reflexive if $ at least one element a Î A such that i) R1 = {(1, 1) (2, 2) (3, 4) (4, 3)} is a the symmetric relation.
(a, a) Ï R i.e. a R a.
ii) R 2 = {(1, 1) (2, 2) (3, 3) (4, 4)} is a symmetric as well as reflexive relation.
3) A relation R is said to be irreflexive if for every a Î A, (a, a) Ï R i.e. a R
/ a
iii) R 3 = { (1, 1) (2, 2) (3, 3) (4, 4) (3, 4)} is reflexive but not symmetric relation.
Notes :
iv) R4 = {(a, b)} a ¹ b is the smallest non- symmetric relation.
1) If R is a reflexive relation on set A then
i) The relation matrix (M R ) is the identity matrix. Example 4.7.1 Let A = Set of all straight lines in a plane. x R y iff x || y i.e. x is parallel
to y. S. T. R is a symmetric relation.
ii) Diagraph of reflexive relation will have loop for every element of A.
Solution : R = {x R y | x is parallel to y}
2) The relation matrix of irreflexive relation has all diagonal elements zero. It's
diagraph is free from loops. Suppose x R y then x is parallel to y.
iff y is parallel to x.
Examples
iff y R x
Example 1 :
i.e. whenever x R y, then y R x.
Let A = {1,2, 3,4}. Then
\ R is a symmetric relation.
i) R1 = {(1, 1) (2, 2) (3, 3) (4, 4)} is the reflexive relation. It is known as the smallest
reflexive relation on set A. 4.7.4 Compatible Relation
ii) R 2 = {(1, 1) (2, 2) (3, 3) (4, 4) (2, 4) (1, 2) (3, 2)} is reflexive relation. A relation R on a set A is said to be compatible relation if it is reflexive and
iii) R 3 = { (1, 1) (2, 2) (3, 4)} is not reflexive relation as 3 Î A but 3 R 33 . symmetric relation. The relation matrix of a compatible relation is a symmetric matrix
/ 4 a.
iv) R 4 = {(2, 3) (3, 4)} is irreflexive relation A as " a Î A, a R with the diagonal elements 1.
e.g. If A = {x, y, z} then
Example 2 :
R = {(x, x) (y, y) (z, z) (x, y) (y, x)} is a compatible relation.
Let A = Set of all straight lines in a plane. x R y iff x is parallel to y.
Note :
For any line x Î A, x is parallel to x.
1) If R is a compatible relation on set A S.T. |A| = n then |R| ³ n
\ x Rx " x Î A
\ R is a reflexive relation.

® ®
TECHNICAL PUBLICATIONS - An up thrust for knowledge TECHNICAL PUBLICATIONS - An up thrust for knowledge
Discrete Mathematics 4 - 15 Relations Discrete Mathematics 4 - 16 Relations

4.7.4.1 Asymmetric Relation ii) R 2 = {(x, y) (y, z) (z, x) (z, y) (x, z) (y, x) (x, x) (y, y) b
A relation R on a set A is said to be asymmetric relation if whenever aRb then (z, z) } is transitive relation.
/ .
bRa iii) R 3 = {(x, y) (z, y)} is a transitive relation as there are no a
Hence R is not asymmetric relation if for some a and b in A, aRb and bRa. terms of the form x Ry and y R z,

e.g. If A = {1, 2, 3} R = {(1, 2) (2, 3) (3, 1)} is a symmetric relation. But R 1 = {(1, 2) Diagraph of transitive relation is as follows :
c
(2, 3) (3, 2)} is not asymmetric relation as (2, 3) and (3, 2) Î R 1 .
4.7.7 Equivalence Relation Fig. 4.7.1

4.7.5 Antisymmetric Relation Let R be a relation on a set A. Then R is said to be an equivalence relation on A iff R
A relation R on a set A is said to be antisymmetric relation if whenever aRb and bRa is reflexive symmetric and transitive relations. It is denoted by 'R' or '~'.
then a = b. A relation R is not antisymmetric if $ a, b Î A such that a ¹ b but aRb and Example :
bRa. The relation matrix of antisymmetric relation is never symmetric matrix. Example 4.7.2 Let A = R, set of all straight lines in a plane.
Examples : R 1 = {x R 1 y | x is parallel to y}, R 2 = {x R 2 y | x is perpendicular to y } check whether
1) Let A = R and aRb iff a £ b. R 1 and R 2 are equivalence relations or not.
Solution : i) For relation R 1 : we know that if x, y, z are straight lines in a plane, then
Suppose aRb and bRa then a £ b and b £ a.
x is parallel to x. \ R 1 is reflexive.
Þ a = b.
ii) If x R 1 y Þ x is || to y Þ y is parallel to x
\ R is antisymmetric relation. It is not symmetric relation.
Þ y R 1 x Þ R 1 is symmetric.
2) Let A = {1, 2, 3} then iii) If x R 1 y, y R 1 z Þ x || y, y || z Þ x || z Þ x R 1 z.
i) R 1 = {(1, 1) (2, 2)} is symmetric, antisymmetric relation. Þ R 1 is transitive relation.
ii) R 2 = {(1, 2) (2, 1)} is symmetric but not antisymmetric relation. \ R1 is an equivalence relation
iii) R 3 = {(1, 1) (1, 2) (2, 2)} is antisymmetric but not asymmetric relation. ii) For relation R 2 :
Any line is not perpendicular to itself.
4.7.6 Transitive Relation
\ x R/ 2 x, " x Î A
Let R be a relation on set A. R is said to be transitive relation if aRb and bRc, then \ R 2 is not reflexive.
aRc for a,b,c, in R.
\ R 2 is not equivalence relation.
\ A relation R is not transitive if $ a, b and c in A.
Example 4.7.3 If A = {x, y, z } and R = {(x, x) (y, y) (z, z)}. It is an equivalence relation ?
/
Such that aRb, bRc but aRc
Solution : The given relation R is reflexive, symmetric and transitive.
Examples :
\ R is an equivalence relation.
1) Let A = R, R be the relation " £ ".
4.7.8 Properties of Equivalence Relations
Then R is transitive as a £ b, b £ c Þ a £ c
1) If R 1 and R 2 are equivalence relations on a set A, then
2) If A = {x, y, z w} then,
R 1 Ç R 2 is an equivalence relation.
i) R 1 = {(x, y) (y, z)} is not transitive relation as x Ry, y Rz but x R/ z.
Þ If R 1 and R 2 are reflexive, symmetric and transitive then R 1 Ç R 2 are also
reflexive, symmetric and transitive.
® ®
TECHNICAL PUBLICATIONS - An up thrust for knowledge TECHNICAL PUBLICATIONS - An up thrust for knowledge

Discrete Mathematics 4 - 17 Relations Discrete Mathematics 4 - 18 Relations

\ R 1 Ç R 2 is an equivalence relation. Þ y + x = even


2) If R 1 and R 2 are equivalence relations on a set A, then it is not necessary that Þ yRx
R 1 È R 2 is an equivalence relation. \ R is symmetric relation.
Let A = {a, b, c) Suppose xRy and yRz Þ x + y = even, y + z = even
And R 1 = {(a, a) (b, b) (c, c) (a, b) (b, a)} Þ x + y + y + z = even
R 2 = { (a, a) (b, b) (c, c) (a, c) (c, a)} are equivalence relation. Þ x + z = even – 2y = even
R 1 È R 2 = {(a, a) (b, b) (c, c) (a, b) (b, a) (a, c) (c, a)} Þ x + z = even
(b, a) and (a, c) Î R 1 È R 2 but (b, c) Ï R 1 È R 2 . Þ xRz

\ R 1 È R 2 is not an equivalence relation. \ R is a transitive relation.

3) If R 1 and R 2 are equivalence relations then R 1 È R 2 is an equivalence relations \ R is an equivalence relation.


iff R 1 Í R 2 or R 2 Í R 1 . \ The equivalence class of 1 Î z is

4.7.9 Equivalence Classes [1] R = {xR 1 / x +1 = even}

Let R be an equivalence relation on a set A. The equivalence class of a ÎA is denoted = {xR 1 / x = even –1= odd}
by [a] R and defined as = {xR 1 / x is odd}
[a] R = {x Î A | x R a} [1] R = Set of all odd integers = {… –3, –1, 1, 3, 5, …}
= The set of those elements of A which are related to a. [ 2] R = {xR 2 |x +2 = even}
\ a Î [a] R = {xR 2| x = even}

Example 1 : = Set of all even integers

Let A = {x, y, z } [ 2] R = {… – 4, – 2, 0, 2, 4 …]

R = A ´ A = {(x, x) (y, y) (z, z) (x, y) (x, z) (y, z) (y, x) (z, x) (z, y)} Theorem 1 : Any two equivalence classes are either identical or disjoint.
is an equivalence relation. Theorem 2 : For any x Î A, x Î A, x Î [x] R
\ The equivalence class of x is
Theorem 3 : A = U [a]
[x] R = {x, y, z} = [y] R = [z] R a ÎA

All these three equivalence classes are identical. Example 4.7.5 Which of the following are relations from A to B
where A = {1, 2, 3, 4} ; B = {x, y, z}
Example 4.7.4 If A = z, R = {(x, y) | x + y = even, x, y Î z }. Is R equivalence relation ? if a) R 1 = {(1, x), (1, y), (1, z), (4, x)}
yes, find equivalence class of 1 and 2. (b) R 2 = {(x, 1), (y, 1), (z, 1), (x, 4)}
Solution : x + x = even number for any x Îz. (c) R 3 = {(1, x), (2, y), (3, z), (4, w)}

\ x Rx Þ R is reflexive relation. (d) R 4 = {(1, 1), (2, 2)}

Suppose x Ry Þ x + y = even

® ®
TECHNICAL PUBLICATIONS - An up thrust for knowledge TECHNICAL PUBLICATIONS - An up thrust for knowledge
Discrete Mathematics 4 - 19 Relations Discrete Mathematics 4 - 20 Relations

Solution : Given that R : A ® B Solution : Given that A= {1, 2, 8}, B = {1, 2, 3, 5}

Where A = Domain set a) R in Roster form is R = {(1, 2) (1, 3), (1, 5), ( 2, 3), ( 2, 5)}

and B = Co-domain set b) Domain of R is {1, 2}

A ´ B = {(1, x), (1, y), (1, z), (2, x), (2, y), (2, z), (3, x), Range of R is {2, 3, 5}
c) Diagraph :
(3, y), (3, z), (4, x), (4, y), (4, z)}
1 2
(a) R is a relation from A to B because R 1 Ì A × B
1

(b) R is not a relation from A to B as R Ë A × B


2 2

(c) R is not a relation from A to B as (4, w) Î R but w Ë B


3

(d) R is not a relation from A to B as R Ë A × B


4 4

5 3

Example 4.7.6 If A = {1,2,3,4}, B = {1, 4, 6, 8, 9} R is a relation from A to B such that aRb Example 4.7.8 If R is a relation on set A = {1, 2, 3, 4, 5}, R = {(1, 1) (1, 2) (2, 1) (3, 1)
iff b = a 2 . Find domain, range, relation matrix and diagraph. SPPU : Dec.-18, Marks 6 (4, 1) (5, 2)}. Find domain set, codomain set, range. Draw its diagraph and find relation
Solution : We have matrix.
R = {(1, 1) (2, 4) (3, 9)} Solution : Given that A = {1, 2, 3, 4, 5}

i) Domain of R is {1, 2, 3} and R is a relation on set A.

ii) Range of R is {1, 4, 9} Domain set = A = Co-domain set

iii) Relation matrix is R = {(1, 1) (1, 2) (2, 1) (3, 1) (4, 1) (5, 2)}
1 4 6 8 9 Range set = {1, 2} Ì A
1 é1 0 0 0 0ù
Its diagraph is as follows :
M(R) = 2 ê 0 1 0 0 0ú
ê ú
3 ê0 0 0 0 1ú
4 ëê 0 0 0 0 0úû
1
iv) Diagraph 2

2 3
5
1 4
9
The matrix form of R is
4 1 2 3 4 5
3 1 é1 1 0 0 0ù
2 ê1 0 0 0 0ú
M (R) = ê ú
Example 4.7.7 If R is a relation from A to B where A = {1, 2, 8}, B = {1, 2, 3, 5} and aRb 3 ê1 0 0 0 0ú
iff a < b. Find : a) R in Roster form b) Domain and Range c) Diagraph. 4 ê1 0 0 0 0ú
ê ú
SPPU : May-18, Marks 4 5 êë 0 1 0 0 0úû 5 ´ 5

® ®
TECHNICAL PUBLICATIONS - An up thrust for knowledge TECHNICAL PUBLICATIONS - An up thrust for knowledge

Discrete Mathematics 4 - 21 Relations Discrete Mathematics 4 - 22 Relations

Example 4.7.9 Let A = {1, 2, 3, 4, 5} and aRb if a < b. Find i) R in Roster form ii) Domain (ii) The relational matrix is
and Range of R iii) Diagraph of R. SPPU : Dec.-19, Marks 3 1 2 3 4 5 6
Solution : We have 1 é1 0 0 0 0 0ù
A = {1, 2, 3, 4, 5} 2 ê1 1 0 0 0 0ú
ê ú
M (R) = 3 ê1 0 1 0 0 0ú
i) By considering the given conditions,
4 ê1 1 0 1 0 0ú
R in Roster form is ê ú
5 ê1 0 0 0 1 0ú
R = {(1, 2), (1, 3), (1, 4), (1, 5), (2, 3) (2, 4), (2, 5) (3, 4) (3, 5) (4, 5)} 6 êë1 1 1 0 0 1úû

ii) Domain of R is = {1, 2, 3, 4} (iii)The diagraph of given relation is


Range of R is = {2, 3, 4, 5} vertex set = {1, 2, 3, 4, 5, 6}
iii) The vertex set of R is {1, 2, 3, 4, 5}
It's diagraph is as follows : 2
1

5
1

3
4
6

2 3 4
5

Example 4.7.10 Let A = {1, 2, 3, 4, 5, 6} aRb or (a, b) Î R iff a is a multiple of b. Find (iv) The indegree of a vertex in diagraph is the number of edges coming towards that
(i) Range set and R (6), R (3). vertex. The outdegree of a vertex is defined as the number of edges going out from a
(ii) Find relation matrix. vertex. Consider the following table.

(iii) Draw diagraph


Vertex Indegee Outdegree Degree
(iv) Find in and out degree of each vertex.
1 6 1 7
Solution : By considering given condition.
2 3 2 5
R = {(1, 1), (2, 1), (2, 2), (3, 1), (3, 3), (4, 1), (4, 2), (4, 4), (5, 1), (5, 5), 3 2 2 4
(6, 1), (6, 2), (6, 3), (6, 6)} 4 3 1 4

(i) The range set of R is {1, 2, 3, 4, 5, 6} 5 2 1 3

6 4 1 5
R (3) = {1, 3} as (3, 1) and (3, 3) Î R
and R (6) = {1, 2, 3, 6} because (6, 1), (6, 2), (6, 3), (6, 6) Î R Example 4.7.11 Let R be a relation on set of natural numbers such that
R = {(x, y) | 2x + 3y and x, y Î N}. Find
(i) The domain and codomain of R. ; (ii) R-1
Solution : Given that x, y Î N
® ®
TECHNICAL PUBLICATIONS - An up thrust for knowledge TECHNICAL PUBLICATIONS - An up thrust for knowledge
Discrete Mathematics 4 - 23 Relations Discrete Mathematics 4 - 24 Relations

\ The smallest values of x and y are 1, 1 respectively. and R = {(1, 1), (1, 2), (1, 3), (2, 3), (3, 3)}
\ 2x + 3y = 2 (1) + 3 (1) = 5 In R, (1, 1), (1, 2) Î R Þ (1, 2) Î R
(1, 1), (1, 3) Î R Þ (1, 3) Î R
which is the smallest value of co-domain
(1, 2), (2, 3) Î R Þ (1, 3) Î R
x = 1 , y = 2 Þ 2x + 3y = 8
(1, 3), (3, 3) Î R Þ (1, 3) Î R
x = 2 , y = 1 Þ 2x + 3y = 7
(2, 3), (3, 3) Î R Þ (2, 3) Î R
x = 2 , y = 2 Þ 2x + 3y = 10
Hence for any (a, b), (b, c) Î R Þ (a, c) Î R
x = 2 , y = 3 Þ 2x + 3y = 13
Thus R is a transitive relation on A.
x = 3 , y = 2 Þ 2x + 3y = 12
\ The domain of R is = {x | x Î N} Example 4.7.14 Let A = Set of all students in SPPU
xRy iff x and y belongs to same class of SPPU V, y Î A.
and the codomain of R is = {y | y Î N}
Is R an equivalence relation ? Find the equivalence class of Atharva in A.
(ii) The inverse relation is
Solution : Given that A = {Set of all students in SPPU}
R -1 = {(y, x) / x, y Î N}
and xRy iff x and y belong to same class of SPPU.
Example 4.7.12 Let A = {1, 2, 3, 4, 6} and let R be defined on A as xRy iff x | y (i.e. x
(i) Any student x and x belong to same class
divides y)
\ xRx , " x Î A Þ R is reflexive relation.
(i) Write R as a set of ordered pairs
(ii) Let xRy Þ x and y belong to same class of SPPU.
(ii) Find R -1 and describe R -1 in words
Solution : Given that, Þ y and x belong to same class of SPPU.
Þ yRx
A = {1, 2, 3, 4, 6} and x R y iff x divides y.
i.e. xRy Þ yRx, " x, y Î R.
(i) \ R = {(1, 2), (1, 3), (1, 4), (1, 6), (2, 4), (2, 6), (3, 6), (1, 1),
Thus R is symmetric relation.
(2, 2), (3, 3), (4, 4), (6, 6)} ... [Here 1|2, 1|1, 2|2, .... ]
(iii) Let xRy and yRz
(ii) The inverse relation R -1 is
Þ x and y belong to same class and y and z belong to same class of SPPU
R -1 = {(2, 1), (3, 1), (4, 1), (6, 1), (4, 2), (6, 2), (6, 3), (1, 1), (2, 2), Þ x, y, z belong to same class of SPPU
(3, 3), (4, 4), (6,6)} ÞxRz
Now R-1 is defined as : xR -1 y iff x is divisible by y.
–1 Þ R is transitive relation.

Example 4.7.13 Let A = {1, 2, 3) R is the relation on A whose matrix is : Thus R is an equivalence relation.
é1 1 1ù Suppose there is one student in SPPU whose name is Atharva.
MR = ê 0 0 1ú \ The equivalence class of Atharva is the set of all students who are studying in the
ê ú
êë 0 0 1úû class of Atharva.
show that R is transitive. SPPU : May-17, Marks 3
Example 4.7.15 Let A = {Set of all lines in a plane} and x, y Î A. Define xRy iff x is
Solution : Given that : A = {1, 2, 3} perpendicular to y. Is R transitive, reflexive and symmetric ?
Solution : Given that xRy iff x perpendicular y.

® ®
TECHNICAL PUBLICATIONS - An up thrust for knowledge TECHNICAL PUBLICATIONS - An up thrust for knowledge

Discrete Mathematics 4 - 25 Relations Discrete Mathematics 4 - 26 Relations

(i) There is no any line which is perpendicular to itself. Example 4.7.17 é1 0 0ù


Let A = {a, b, c} and M (R) = ê 0 1 1ú .
\ $/ any x such that xRx. ê ú
êë 0 1 1úû
Þ R is not reflexive.
Determine whether R is an equivalence relation ? Find the equivalence class of b in A.
(ii) Suppose xRy Þ x is perpendicular to y.
Solution : From relation matrix, the relation R is
Þ y is perpendicular to x.
Þ yRx R = {(a, a) (b, b) (c, c) (b, c) (c, b)}

i.e. xRy Þ yRx i.e. R is symmetric relation. (i) Here a, b, c Î A and (a, a) (b, b) and (c, c) Î R
(iii) Suppose xRy and yRz \ R is reflexive relation.
Þ x is perpendicular to y and y is perpendicular to z. (ii) As (b, c) Î R Þ (c, b) Î R
Þ x and z are parallel lines. R is symmetric relation.
Þ xRz (iii) In relation R, we have,
Þ R is not transitive relation. (b, b), (b, c) Î R Þ (b, c) Î R
Example 4.7.16 Let A = {set of lines in a plane} and x, y Î A. xRy iff x is parallel to y. Is R (b, c), (c, b) Î R Þ (b, b) Î R
equivalence relation ? Find the equivalence class of any line 'L'. (b, c), (c, c) Î R Þ (b, c) Î R
Solution : (c, c), (c, b) Î R Þ (c, b) Î R
(i) We know that every line is parallel to itself.
(c, b), (b, b) Î R Þ (c, b) Î R
\ xRx , " x Î A
(c, b), (b, c) Î R Þ (c, c) Î R
Þ R is reflexive relation.
So R is transitive relation. Thus R is an equivalence relation.
(ii) Suppose xRy
Now, the equivalence class of b in A is,
Þ x is parallel to y.
[b] = {x | (x, b) or (b, x) Î R}
Þ y is parallel to x.
Þ yRx Þ R is symmetric relation. [b] = {b, c}

(iii) Suppose xRy and yRz. Example 4.7.18 If R = {(1, 1), (2, 2), (3, 3), (4, 4), (5, 5) (3, 5), (5, 3), (1, 3), (3, 1)}
Þ x is parallel to y and y is parallel to z. is an equivalence relation on X = {1, 2, 3, 4, 5}. Find equivalence
Þ x is parallel to z. classes. SPPU : May-19, Marks 3

Þ xRz Þ R is transitive relation. Solution : If R is an equivalence relation on X and a Î X then [a] = a = {x|x~ a }.

Thus R is an equivalence relation. Therefore, equivalence classes are as follows


Let L be any line in A. [1] = 1 = {1, 3}
Equivalence class of L = [L] = {xRL | x is parallel to L} [2] = 2 = {2}, [3] = 3 = {1, 3, 5}
[L] = {x|x is parallel to L} [4] = 4 = {4}
= Set of all lines which are parallel to L. [5] = 5 = {3, 5}

Example 4.7.19 Let A be the set of integers and let R be defined as xRy iff x £ y. Is R
equivalence relation ?
® ®
TECHNICAL PUBLICATIONS - An up thrust for knowledge TECHNICAL PUBLICATIONS - An up thrust for knowledge
Discrete Mathematics 4 - 27 Relations Discrete Mathematics 4 - 28 Relations

Soltuion : Given that xRy iff x £ y, " x, y Î A Example 4.7.21 Let A = {1, 2, 3, 4, 5, 6, 7, 8, 9} and let N be the relation on A ´ A defined
by (a, b) ~ (c, d) iff a + d = b + c.
(i) We have, x £ x , " x Î A
(i) Prove that ~ is an equivalence relation.
Þ xRx, " x Î A
(ii) Find equivalence class of (2, 5) SPPU : Dec.-11
Þ R is reflexive relation.
Solution : Given that (a, b) ~ (c, d) iff a + d = b + c, " a, b, c, d Î A
(ii) If xRy Þ x £ y
(i) (a) We have a + b = b + a Þ (a, b) ~ (a, b)
then y £/ x Þ y R/ x
Þ '~' is reflexive relation.
Þ R is not symmetric relation.
(b) If (a, b) ~ (c, d) then a + d = b + c.
\ R is not an equivalence relation.
Þb+c=a+dÞc+b=d+a
Example 4.7.20 Let A = Z + the set of positive integers, and let Þ (c, d) ~ (a, b) by definition
R = {(a, b) Î A ´ A | a divides b}
Þ '~' is symmetric relation.
Is R symmetric, asymmetric or antisymmetric. SPPU : May-17, Marks 3
(c) Suppose (a, b) ~ (c, d) and (c, d) ~ (e, f)
Ans. : Given that : A = Z + = The set of positive integers
then a+d = b+c
and R = {(a, b)/a divides b}
and c+f = d+e
i) If (a, b) Î R then a divides b
Þ / + c/ + f = b + c/ + d
a+d / +e
\ b = aK, K Î Z +
1 1 a+f = b+e
Þ a = b but if K ¹ 1 then is not an integer
K K
Þ (a, b) ~ (e, f)
\ a does not divides b if
\ ~ is a transitive relation.
b is not multiples of K
Thus '~' is reflexive, symmetric and transitive.
\ (a, b) Î R Þ (b, a) may not be present in R.
Þ '~' is an equivalence relation.
\ R is not symmetric relation.
ii) If (a, b) Î R then a/b (ii) We have (2, 5) Î A × A
\ b = aK, K Î Z + The equivalence class of (2, 5) is the set of elements of A × A which are equivalent to
1
Þ a = b, As K Î Z + (2, 5)
K
b is not divide a \ [(2, 5)] = {(x, y) | (x, y) ~ (2, 5) , x, y Î A}
i.e. if (a, b) Î R then (b, a) Ï R. = {(x, y) | x + 5 = y + 2}
\ R is asymmetric relation. = {(x, y) | x - y = -3 or y = x + 3 ; x , y Î A}
iii) As (a, b) Î R, (b, a) Î R
Hence, [(2, 5)] = {(1, 3) (2, 5) (3, 6) (4, 7) (5, 8) (6, 9)} ... [Q (7, 10) Ï A × A]
then b = K 1 a, a = K 2 b, K 1 , K 2 Ï Z +
b = K 1 , a = (K 1 K 2 )b Example 4.7.22 Prove that in the set N × N, the relation R defined by (a, b) R (c, d) iff

Þ K 1K 2 = 1 Þ K 1 = K 2 = 1 ad = bc is an equivalence relation.
\ a =b Solution : We know that relation R is an equivalence relation if R is reflexive, symmetric
Thus R is antisymmetric relation. and transitive.

®
TECHNICAL PUBLICATIONS - An up thrust for knowledge TECHNICAL PUBLICATIONS® - An up thrust for knowledge

Discrete Mathematics 4 - 29 Relations Discrete Mathematics 4 - 30 Relations

(i) We have, ab = ba Þ (a, b) R (e, f)


Þ (a, b) R (a, b) \ R is transitive relation.
\ R is reflexive relation. Thus, R is an equivalence relation.
(ii) Suppose (a, b) R (c, d) (ii) An equivalence class of (3, 2) is the set of elements of A which are equivalent to
(3, 2)
Þ ad = bc
\ [(3, 2)] = {(x, y) | (x, y) R (3, 2) ; x, y Î R}
Þ bc = ad
= {(x, y) | x 2 + y 2 = 9 + 4 = 13 ; x, y Î Â}
Þ cb = da
[(3, 2)] = The set of points on circle x 2 + y 2 = 13.
Þ (c, d) R (a, b)
Example 4.7.24 Let R be the binary relation defined as R = {(a, b) Î Â 2 / (a – b) £ 3}
\ R is symmetric relation.
(i) Determine whether R is reflexive, symmetric, transitive and antisymmetric relation.
(iii) Let (a, b) R (c, d) and (c, d) R (e, f) (ii) How many binary relations are there on the finite set
Þ ad = bc and cf = ed Solution : Given that,
Þ adcf = bced R = {(a, b) Î R 2 | (a – b) £ 3}
Þ af = be Þ (a, b) R (e, f) (i) (a) We have, a – a = 0 £ 3
\ R is transitive relation. (a, a) Î R Þ R is reflexive.
Thus, R is an equivalence relation.
(b) Suppose (a, b) Î R Þ a – b £ 3
Example 4.7.23 Let A = R × R (R is set of real numbers) and define the following relation on
– (b – a) ³ 3
A. (a, b) R (c, d) iff a 2 + b 2 = c 2 + d 2
b–a £ –3
(i) Show that (A, R) is an equivalence relation.
(ii) Find equivalence class of (3, 2) (b – a) £ – 3 does not imply b – a £ 3
Solution : Given that (a, b) R (c, d) iff a 2 + b 2 = c 2 + d 2 e.g. a = – 100 , b = 2
(i) (a) We have, a 2 + b 2 = a 2 + b 2 a – b = – 100 – 2 = – 102 < 3
Þ (a, b) R (a, b) (– 100, 2) Î R

R is a reflexive relation. But b – a = 2 – (– 100) = 2 + 100 = 102 £/ 3


(b) If (a, b) R (c, d) then a 2 + b 2 = c 2 + d 2 Thus R is not symmetric relation.
Þ c 2 + d2 = a 2 + b 2 (c) If (a, b), (b, c) Î R
Þ (c, d) R (a, b) then a–b £ 3, b–c£3

R is symmetric relation. a–b+b–c £ 6


(c) Suppose (a, b) R (c, d) and (c, d) R (e, f) a–c £ 6
Þ a 2 + b 2 = c 2 + d 2 and c 2 + d 2 = e2 + f2 Þ a – c may or may not be £ 3.
Þ a 2 + b 2 = e2 + f 2 R is not tansitive relation.

TECHNICAL PUBLICATIONS® - An up thrust for knowledge TECHNICAL PUBLICATIONS® - An up thrust for knowledge
Discrete Mathematics 4 - 31 Relations Discrete Mathematics 4 - 32 Relations

(d) Suppose aRb and bRa Solution : Given that, R = {(x, y) | x – y is an odd positive integer}

Þ a – b £ 3 and b – a £ 3 (i) For any positive integer x, x – x = 0 which is even integer.


e.g. a = 1, b = 3, \ (x, x) Ï R

a – b = – 2 < 3 and b – a = 2 < 3 R is not reflexive relation.

but a ¹ b. (ii) Suppose (x, y) Î R

\ R is not antisymmetric relation. then x – y = Odd positive integer


2 Þ y – x = Odd negative integer
(ii) Let n be the number of elements in a finite set A, then they are n n binary
relations on set A. (y, x) Ï R
Example 4.7.25 Let S be the set of points in a plane. Let R be a relation such that xRy iff y \ R is not symmetric relation.
is within two centimeter from x. Is R equivalence relation ? (iii) Suppose (x, y), (y, z) Î R
Solution : Given that
\ x – y = Odd positive integer = p
xRy iff |x - y| < 2 , " x , y Î A
y – z = Odd positive integer = q
(i) We have, |x – x| = 0 < 2
x–y+y–z = p+q
\ xRx, " x Î A
Þ R is reflexive relation x – z = p + q which is even positive integer.

(ii) If xRy Þ |x – y| < 2 {e.g. (5, 4) Î R , (4, 1) Î R. But (5, 1) Ï R as 5 – 1 = 4}


\ (x, z) Ï R
Þ |y - x| < 2
\ R is not transitive relation.
Þ yRx \ R is symmetric relation.
(iv) Suppose (x, y) and (y, x) Î R
(iii) Suppose xRy and yRz
then x – y = Odd positive integer = p
Þ |x – y| < 2 and |y – z| < 2
y – x = Odd positive integer = q
Þ |x – y| + |y – z| < 2 + 2 = 4
Þ x ³ y and y ³ x
Þ |x – z| < 4
Þ x = y
Þ |x – z| </ 2
R is antisymmetric relation.
\ R is not transitive relation.
So R is not an equivalence relation. Example 4.7.27 Show that R = {(a, b) / a º b (mod m)} is an equivalence relation on z. Show
that x 1 º y 1 and x 2 º y 2 then x 1 + x 2 º y 1 + y 2
Example 4.7.26 Let R be a relation on the set of positive integers such that
Solution : Given that
R = {(x, y)|x – y is an odd positive integer}
Is R reflexive, symmetric, transitive, antisymmetric, an equivalence relation ? R = {(a, b)|a º b (mod m)}

We know that a º b (mod m) iff m | (a – b)

a – b = mk , k Î z

TECHNICAL PUBLICATIONS® - An up thrust for knowledge TECHNICAL PUBLICATIONS® - An up thrust for knowledge

Discrete Mathematics 4 - 33 Relations Discrete Mathematics 4 - 34 Relations

(i) Let a Î z , a – a = 0 = m (0) Example 4.7.28 For each of these relations on set A = {1, 2, 3, 4} decide whether it is

\ m|(a – a) Þ a º a (mod m) reflexive,


symmetric, transitive or antisymmetric.
(a, a) Î R Þ R is reflexive relation.
R1 = {(1, 1) (2, 2) (3, 3) (4, 4)}
(ii) Let (a, b) Î R
R 2 = {(1,1)(1, 2) (2, 2) (2,1) (3, 3) (4, 4)}, R 3 = {(1, 3) (1, 4) (2, 3) (2, 4) (3, 1) (3,4,)}.
Þ a º b (mod m)
SPPU : Dec.-10
Þ a – b = mk Solution : i) For R 1 :
Þ b – a = m (– k) As " a Î A, (a, a) Î R 1
Þ m | b–a \ R 1 is Reflexive, symmetric relation.
Þ b º a (mod m) $/ (a, b) and (b, c) Î R 1 \ R 1 is transitive relation.

Þ (b, a) Î R $/ (a, b) and (b, a) Î R 1 \ R 1 is antisymmetric relation.

\ R is symmetric relation. \ R 1 reflexive, symmetric, transitive and anti-symmetric relation.

(iii) Suppose (a, b) (b, c) Î R ii) For R 2 :

then a º b (mod m) and b º c (mod m) As " a Î A (a, a) Î R 2 and aR 2 b Þ bR 1 a, for a,b Î A


\ R 2 reflexive and symmetric relation.
Þ a – b = mk1 and b – c = mk2
For any aR2b and bR2C Þ aR2C
Þ a – b + b – c = mk1 + mk2
\ R2 is transitive realtion
Þ a – c = m(k1 + k2) = mk \ R2 is an equivalence relation
Þ m|(a - c) Þ a º c (mod m) But (1, 2) and (2, 1) Î R2 and 1 ¹ 2
\ R2 is not antisymmetric relation.
\ (a, c) Î R
iii) For R 3 :
\ R is transitive relation.
As 2 Î R 3 but (2, 2) Ï R 3
\ R is reflexive, symmetric and transitive.
\ R 3 is not reflexive relation.
Thus R is an equivalence relation,
As (1, 4) Î R 3 but (4, 1) Ï R 3
Now, x1 º y1 Þ x1 º y1 (mod m)
\ R 3 is not symmetric relation.
x2 º y2 Þ x2 º y2 (mod m) As (2, 3) and (3, 1) Î R 3 but (2, 1) Ï R3.
x1 – y1 = mk1 and x2 – y2 = mk2 \ R 3 is not transitive relation.
Þ x1 – y1 + x2 – y2 = mk1 + mk2 As (1, 3) and (3, 1) Ï R3 but 1 ¹ 3.

Þ (x1 + x2) – (y1 + y2) = m (k1 + k2) \ R3 is not transitive relation


As (1, 3) and (3, 1) Î R 3 but 1 ¹ 3
Þ m|(x1+ x2) – (y1 + y2)
\ R3 is not antisymmetric relation
Þ (x1 + x2) º (y1 + y2) (mod m)
Example 4.7.29 Consider the relation on A = { 1, 2, 3, 4, 5, 6} . R = { (i, j ) | i – j| = 2}. Is
Þ x1 + x2 º y1 + y2 (mod m)
R reflexive ? Is R symmetric ? Is R transitive ? SPPU : Dec.-14
Þ x1 + x2 º y1 + y2

TECHNICAL PUBLICATIONS® - An up thrust for knowledge TECHNICAL PUBLICATIONS® - An up thrust for knowledge
Discrete Mathematics 4 - 35 Relations Discrete Mathematics 4 - 36 Relations

Solution : Given that A = {1, 2, 3, 4, 5, 6} 4.8 Partitions of a Set SPPU : Dec.-11, 12, May-17

R = { (1, 3) (3, 1) (2, 4) (4, 2) (3, 5) (5, 3) (4, 6) (6, 4)} We shall now discuss the concept of partitions of a set which is similar to
equivalence class of set.
As 2 Î A but (2, 2) Ï R \ R is not reflexive.
For any (a, b) Î R Þ (b,a) Î R \ R is symmetric. Definition :

As (1, 3) (3, 1) Î R but (1, 1) Ï R \ R is not transitive. Let A be any non empty set. A set
P = {A1 , A 2 , A 3 , .... A n } of non empty subsets of A is called a partition of set A if
Example 4.7.30 Let A = { 1, 2, 3, 4, 5, 6, 7 } and R = {(x, y) | x – y is divisible by 3}. Show
n
that R is an equivalence relation ? Draw graph of R. i) A1 È A 2 È A 3 È .... È A n = A = U A i
i =1
SPPU : May-14, Dec.-12, 19, Marks 6
Solution : We have R = {x, y) | x – y is divisible by 3} i.e. Set A is the union of the sets A1 , A 2 , ... A n .
We know that x – x = 0 is divisible by 3 ii) A i Ç A j = f for i ¹ j i.e. All sets A i are mutually disjoints.
x R x, " x Î A Þ R is reflexive relation.
The partition of a set A is denoted by p.
As x R y Þ x – y is divisible by 3
An element of a partition set is called a block. The rank of a partition is called as the
Þ y – x is also divisible by 3
number of blocks of that partitions. It is denoted by r( p ). For any non empty set, its
Þ y – z is divisible by 3
partitions are not unique. There are different partitions of the same set.
Þ y R x for x, y Î A
\ R is a symmetric relation. e.g.
As, 1) Let A = Z = Set of integers
x R y and y R Þ x – y and y – z are divisible by 3 A
A1 = Set of even integers A1
Þ (x – y) + (y – z) is also divisible by 3
Þ x – z is divisible by 3 A 2 = Set of odd integers
A2
Þ xRz A 3 = {1, 2, 3, 4, ....},
\ R is a transitive relation.
A 4 = {0}
\ R is an equivalence relation.
Fig. 4.8.1
and R = {(1, 1), (2, 2) (3, 3) (4, 4) (5, 5) (6, 6) (7, 7) (1, 4) ( 4, 1) (1, 7) (7, 1) A 5 = {–1, –2, –3, –4, .....}

(2, 5), (5, 2) (3, 6) (6, 3)} P1 = {A1 , A 2 } and P2 = {A 3 , A 4 , A 5 } are different partitions of set A.

Its graph is as follows 2) If A = {1, 2, 3, 4, 5, 6, 7, 8, 9, 10} and it's subsets


A1 = {1, 4, 9} A 2 = {2, 6, 8, 10}, A 3 = {3, 5, 7},
1 7 3
The set P = {A1 , A 2 , A 3 } is such that
i) A1 , A 2 , A 3 are non empty sets
6
ii) A = A1 È A 2 È A 3
4
2 5 iii) A1 Ç A 2 = f, A1 Ç A 3 = f, A2 Ç A3 = f

Hence {A1 , A 2 , A 3 } form a partition for set A.


Fig. 4.7.2

TECHNICAL PUBLICATIONS® - An up thrust for knowledge TECHNICAL PUBLICATIONS® - An up thrust for knowledge

Discrete Mathematics 4 - 37 Relations Discrete Mathematics 4 - 38 Relations

4.8.1 Relation Induced by a Partition Example 4.8.2 Let A = (1, 2, 3, 4) consider partition.
P = {{1, 2, 3}, {4}}.
Let P be the partition of a non empty set A. We can define a relation R on a set A as
of A find the equivalence relation R on A determined by P. SPPU : May-17, Marks 3
x Ry iff x and y belong to the same block of the partition P. This relation R is called
Solution : Given that : A = {1, 2, 3, 4} and the partition P = {{1, 2, 3}, {4}}
as the relation induced by the partition.
We know that equivalence classes form a partition for the corresponding set.
e.g. i) Let A = Z and P = {A1 , A2 }
P1 = {1, 2, 3} and P2 = {4} are two equivalence classes.
where A1 = Set of even integers and
\ In each equivalence class, every element is related to all elements of that class.
A 2 = Set of odd integers
\ Due to P1 = (1, 1) (1, 2), (1, 3), (2, 2), (2, 3), (2, 1), (3, 3), (3, 1), (3, 2) and for P2 , (4, 4)
\ P is a partition for A = Z
Hence R = {(1, 1), (1, 2), (1, 3), (2, 1), (2, 2), (2, 3), (3, 1), (3, 2), (3, 3), (4, 4)}
Define x Ry Iff x and y belong to the same partition of Z
is an equivalence relation with respect to the given partition P.
i.e. x and y belong to either A1 or A 2
Example 4.8.3 Let A = {1, 2, 3, 4}, p = {{1}, {2, 3}, {4}}. Find the equivalence relation
i.e. x and y are either even or odd
induced by p.
This relation is an equivalence relation.
Solution : Given that
Theorem 1 : Let R be an equivalence relation on a set A then the set of equivalence
p = {{1}, {2, 3}, {4}} has 3 blocks
classes {[ a] R a Î A}.
The first block, {1} Þ iR1
Theorem 2 : Let A be any non empty set and let p be a partition of A. Then p induces
an equivalence relation on set A. The second block,{2, 3} Þ 2R 2 , 2R 3 , 3R 2 , 3R 3
The third block, {4} Þ 4R 4
Example 4.8.1 Let A = {x, y, z, u, v}, p = {{x, y}, {z}, {u, v}}. Find the equivalence relation
induced by p. \ The required relation is
Solution : R = {(1, 1), (2, 2), (2, 3), (3, 2), (3, 3), (4, 4)}

We have A = {x, y, z, u, v} and Define R is reflexive symmetric and transitive


\ R is an equivalence relation
x Ry iff x and y belongs to the same block of the partition of A.
Example 4.8.4 Define partition of a set. Let x = {1, 2, 3, 4, 5, 6, 7, 8, 9}. Determine whether
p = {{x, y}, {z}, {u, v}} has 3 blocks.
or not each of the following is a partition of x :
The first block {x, y} ® x, y Î same block
A = {{2, 4, 5, 8}, {1, 9}, {3, 6, 7}}
\ x Rx, xRy, yRx, yRy B = {{1, 3, 6}, {2, 8}, {5, 7, 9}} SPPU : Dec.-11
The second block, {z} Þ zRz Solution : Please refer section 4.8 for definition.
The third block, {u, v} Þ uRu, uRv, vRu, vRv i) For set A :
\ The required relation is The set A has 3 blocks,
R = {(x,x) (x,y), (y,x), (y,y), (z,z), (u,u), (u,v), (v,u), (v,v)} A1 = {2, 4, 5, 8}, A 2 = {1, 9} A 3 = {3, 6, 7}
The relation R is reflexive, symmetric and transitive · The union of all these blocks is a set X
\ R is an equivalence relation. A1 È A 2 È A 3 = X

TECHNICAL PUBLICATIONS® - An up thrust for knowledge TECHNICAL PUBLICATIONS® - An up thrust for knowledge
Discrete Mathematics 4 - 39 Relations Discrete Mathematics 4 - 40 Relations

· These blocks are mutually disjoints. 4.8.3 Product and Sum of Partitions
\ The set A forms a partition for the set X I) Let p 1 and p 2 be two partitions of a non empty set A. The product of p 1 and p 2
ii) For set B : is denoted by p 1 – p 2 = p is a partition p of A such that
1) p refines both p 1 and p 2
\ The set B has 3 blocks.
2) If p ¢ refines both p 1 and p 2 then p ¢ refines p.
B1 = {1, 3, 6}, B2 = {2, 8} B3 = {5, 7, 9}
II) Let p 1 and p 2 be the partitions of a non empty set A. The sum of p 1 and p 2 is
As B1 È B2 È B3 ¹ X, denoted by p = p 1 + p 2 is a partition of A such that

B is not a partition of set X. 1) Both p 1 and p 2 refine p


2) If p 1 and p 2 refine p ¢ then p refines p ¢
Example 4.8.5 If S = {1, 2, 3, 4, 5, 6, 7, 8, 9}. Determine whether or not each of the
following is a partition of S. 4.8.4 Quotient Set
i) A = {{1, 3, 5}, {2, 6}, {4, 8, 9}} Let A be any non empty set and R be an equivalence relation on A. The set of
ii) B = {{1, 3, 5}, {2, 4, 6, 8}, {7, 9}} mutually disjoint equivalence classes in which A is partitioned relatively to the
iii) C = {{1, 3, 5}}, {2, 4, 6, 8}, {5, 7, 9}} equivalence relation R is called the quotient set of A for relation R. If is denoted by A/R
iv) D = {{5}} SPPU : Dec.-12 or A.
Solution : Given that e.g. The quotient set of Zl, for an equivalence relation
S = {1, 2, 3, 4, 5, 6, 7, 8, 9} R = {( a, b) 3|{( a – b) , a, b Î Z } is
i) A is not a partition of S because S is not the union of all blocks of A. Z/R = {[0], [1], [2]}.
i.e. S ¹ A1 È A 2 È A 3 Theorem 1 : Let R 1 and R 2 be an equivalence relations induced by partitions p 1 and
ii) B is the partition of S as B1 È B2 È B3 = S and B1 , B2 , B3 are mutually disjoints. p 2 of a non empty set A respectively, then
iii) As blocks of set C are not disjoints. \ The set C is not a partition of S. i) The relation R = R 1 Ç R 2 induces the product partition p 1 × p 2

iv) D = {{s}} is a partition of S, called as trivial partition. ii) The relation R = (R 1 È R 2 ) * {i.e. The transitive closure of R 1 È R 2 } is an
equivalence relation on A and induces the partition p 1 + p 2 .
4.8.2 Refinement of Partitions 4.9 Closure of a Relation
Let p and p ¢ be partitions of a non empty set A. SPPU : May-06, 07, 08, 15, 18, Dec.-05, 07, 12, 13, 14, 15, 16

Then p ¢ is called refinement of p if every block (or an element) of p ¢ is contained in Depending upon the nature of relations, there are mainly three types of closures of
a block of p. relations.

e.g. Let A = {1, 2, 3}, p = {{1}, {2, 3}}, p ¢ = {{1}, {2}, {3}} 4.9.1 Reflexive Closure

{1}, {2}, {3} Ï p but {1}, {2, 3} Î p ¢ Let R be a relation on a set which is not reflexive relation. A relation R 1 = R È D is
called the reflexive closure of R if R È D is the smallest reflexive relation containing R.
\ p ¢ is a refinement of p
If A = {a, b, c, d} then D = {(a, a), (b, b), (c, c), (d, d)}
Theorem : Let p and p ¢ be partitions of a non empty set A and R 1 , R 2 be the
equivalence relations induced by p and p ¢ respectively. Then p ¢ refines p iff R 2 Ì R 1 .

TECHNICAL PUBLICATIONS® - An up thrust for knowledge TECHNICAL PUBLICATIONS® - An up thrust for knowledge

Discrete Mathematics 4 - 41 Relations Discrete Mathematics 4 - 42 Relations

Example 4.9.1 Let A = {1, 2, 3}. R 1 , R 2 and R 3 are relations on set A. Find the reflexive 4.9.3 Transitive Closure
closures of R 1 , R 2 and R 3 .
Let R be a relation on a set A which is not transitive relation. The transitive closure
Where R 1 = {(1, 1) (2, 1)}, R 2 = {(1, 1) (2, 2), (3, 3)},
of a relation R is the smallest transitive relation containing R. It is denoted by R * .
R 3 = {(3, 1) (1, 3), (2, 3)}.
The following theorem gives a procedure to find the transitive closure of a given
Solution :
relation.
We have A = {1, 2, 3} \ D = {(1, 1), (2, 2), (3, 3)}
Theorem : Let A be any non empty set and |A| = n. Let R be a relation on A. Then
Then
the transitive closure of R is R * = R È R 2 È R 3 È ... È R n .
i) The reflexive closure of R 1 is R = R 1 È D
Example 4.9.3 If A = {1, 2, 3, 4, 5} and R = {(1, 2), (3, 4), (4, 5), (4, 1), (1, 1)}. Find it's
\ R = {(1, 1) (2, 2) (3, 3) (2, 1)} transitive closure.
ii) The reflexive closure of R 2 is R = R 2 È D = R 2 Solution : Let R * be the transitive closure of given relation R.
iii) The reflexive closure of R 3 is R = R 3 È D \ R* = R È R 2 È R 3 È R 4 È R 5
\ R = {(1, 1) (2, 2) (3, 3) (3, 1) (1, 3), (2, 3)} Now R 2 = R × R = {(3, 5) (3, 1) (4, 2) (4, 1) (1, 2) (1, 1)}
R 3 = R × R 2 = {(3, 2) (3, 1) (4, 2) (4, 1) (1, 1) (1, 2)}
4.9.2 Symmetric Closure
R 4 = R × R 3 = {(3, 1) (3, 2) (4, 1) (4, 2) (1, 1) (1, 2)} = R 3
Let R be a relation on a set A and R is not symmetric relation.
R 5 = R × R 4 = {(3, 1) (3, 2) (4, 1) (4, 2) (1, 1) (1, 2)} = R 4 = R 3
A relation R 1 = R È R –1 is called the symmetric closure of R if R È R –1 is the smallest
symmetric relation containing R. R * = {(1, 2) (3, 4) (4, 5) (4, 1) (1, 1) (3, 5) (3, 1) (4, 2) (3, 2)}

Example 4.9.2 Find the symmetric closure of the following relations. On A = {1, 2, 3}. Which is the transitive closure of R.
R 1 = {(1, 1) (2, 1)}
4.9.4 Warshall's Algorithm to Find Transitive Closure
R 2 = {(1, 2) (2, 1) (3, 2) (2, 2)}
To find the transitive closure of a relation by computing various powers of R or
R 3 = {(1, 1) (2, 2) (3, 3)} SPPU : Dec.-12
product of the relation matrix is quite impractical for large relations. Warshall's
Solution : Given that algorithm gives an alternate method for finding transitive closure of R. Warshall's
We have A = {1, 2, 3} algorithm is practical and efficient method.

i) R 1–1 = {(1, 1) (1, 2)} Consider the following steps to find transitive closure of the relation R on a set A.

\ R = R 1 È R 1–1 = {(1, 1) (1, 2) (2, 1)} is the symmetric closure Step 1 : We have |A| = n
of R 1 \ We require Wo , W1 , W2 , ... Wn . Warshall sets
ii) R –1
2
= {(2, 1) (1, 2) (2, 3) (2, 2)} Wo = Relation Matrix of R = M R .
\ R = R2 È R –1
= {(1, 2) (2, 1) (3, 2) (2, 3) (2, 2)} is the
2
symmetric closure of R 2 Step 2 : To find the transitive closure of relation R on set A, with |A| = n
Procedure to compute Wk from Wk–1 is as follows
iii) R 3 is the symmetric relation.
i) Copy 1 to all entries in Wk from Wk–1 , where there is a 1 in Wk–1 .
\ R 3 itself is the symmetric closure.
ii) Find the row numbers p1 , p 2 , p 3 ... for which there is 1 in column k in Wk–1 and
the column numbers q 1 , q 2 , q 3 ... for which there is 1 in row k of Wk–1 .

TECHNICAL PUBLICATIONS® - An up thrust for knowledge TECHNICAL PUBLICATIONS® - An up thrust for knowledge
Discrete Mathematics 4 - 43 Relations Discrete Mathematics 4 - 44 Relations

iii) Mark entries in Wk as 1 for ( p i , q i ). If there are not already 1. Step 3 : To find W2 :
To find W2 from W1 , we consider the second column C 2 and second row R 2 .
Step 3 : Stop the procedure when Wn is obtained and it is the required transitive
closure of R. In C 2 , 1 is present at R 4

Examples
In R 2 , 1 is present at C 4
Thus add new entry in W2 at (R 4 , C 4 ), which is given below
Example 4.9.4 Find the transitive closure of R by Warshall's algorithm.
Where A = {1, 2, 3, 4, 5, 6} and R = {(x, y)/(x – y) = 2} SPPU : Dec.-05, 12, 13, 14 0 0 1 0 0 0
Solution : 0 0 0 1 0 0
W2 = 1 0 1 0 1 0
0 1 0 1 0 1
Step 1 : We have |A| = 6,
0 0 1 0 0 0
R = {(1, 3), (3, 1), (2, 4), (4, 2), (4, 6), (6, 4), (3, 5), (5, 3)} 0 0 0 1 0 0

Thus we have to find Warshall's sets, Wo , W1 , W2 , W3 , W4 , W5 and W6 . Step 4 : To find W3 :


The first set Wo is same as M R . Which is shown below To find W3 from W2 , we consider the third column and third row.
In C 3 , 1 is present at R 1 , R 3 , R 5
1 2 3 4 5 6 In R 3 , 1 is present at C1 , C 3 , C 5
1 0 0 1 0 0 0 Thus add new entries in W3 at ( R 1 , C1 ), ( R 1 , C 3 ), ( R 1 , C 5 )
2 0 0 0 0 1 0
( R 2 , C1 ), ( R 2 , C 3 ), ( R 2 , C 5 ) ( R 3 , C1 ), ( R 3 , C 3 ), ( R 3 , C 5 ) which is given below
3 1 0 0 0 1 0
W0 = MR =
4 0 1 0 0 0 1
1 0 1 0 1 0
5 0 0 1 0 0 0
0 0 0 1 0 0
6 0 0 0 1 0 0 1 0 1 0 1 0
W3 =
0 1 0 1 0 1
1 0 1 0 1 0
Step 2 : To find W1 : 0 0 0 1 0 0

To find W1 from Wo , we consider the first column and first row. Step 5 : To find W4 :
In a column C1 , 1 is present at R 3 To find W4 from W3 , we consider the fourth column and fourth row.
In a row R 1 , 1 is present at C 3 In C 4 , 1 is present at R 2 , R 4 , R 6
Thus add new entry in W1 , at (R 3 , C 3 ) which is given below In R 4 , 1 is present at C 2 , C 4 , C 6
Thus add new entries in W4 at (R 2 , C 2 ), (R 2 , C 4 ), (R 2 , C 6 ), (R 4 , C 2 ), (R 4 , C 4 ),
(R 4 , C 6 ), (R 6 , C 2 ), (R 6 , C 4 ), (R 6 , C 6 ) which is given below
0 0 1 0 0 0
0 0 0 1 0 0
1 0 1 0 1 0
1 0 1 0 1 0
W1 = 0 1 0 1 0 1
0 1 0 0 0 1
W4 = 1 0 1 0 1 0
0 0 1 0 0 0
0 1 0 1 0 1
0 0 0 1 0 0 1 0 1 0 1 0
0 1 0 1 0 1

TECHNICAL PUBLICATIONS® - An up thrust for knowledge TECHNICAL PUBLICATIONS® - An up thrust for knowledge

Discrete Mathematics 4 - 45 Relations Discrete Mathematics 4 - 46 Relations

Step 6 : To find W5 : a b c d
a
To find W5 from W4 , we consider the 5 th column and 5 th row.
0 0 0 1
b 1 0 0 1
W 0 = MR =
In C 5 , 1 is present at R 1 , R 3 , R 5 c 0 1 0 1
d 0 0 1 0
In R 5 , 1 is present at C1 , C 3 , C 5
Thus add new entries in W5 at (R 1 , C1 ), (R 1 , C 3 ), (R 1 , C 5 ), (R 3 , C1 ), (R 3 , C 3 ), Step 2 : To find W1 :
(R 3 , C 5 ), (R 5 , C1 ), (R 5 , C 3 ), (R 5 , C 5 ) which is given below To find W1 from Wo , we consider the first column and first row.
In C1 , 1 is present at R 2
1 0 1 0 1 0
0 1 0 1 0 1
In R 1 , 1 is present at C 4

W5 = 1 0 1 0 1 0
= W4 Thus add new entry in W1 at (R 2 , C 4 )
0 1 0 1 0 1
1 0 1 0 1 0
0 0 0 1
0 1 0 1 0 1
1 0 0 1
W1 =
0 1 0 1
0 0 1 0
Step 7 : To find W6 :
Step 3 : To find W2
To find W6 from W5 we consider the 6 th column and 6 th row.
To find W2 , from W1 , we consider the second column and second row.
In C 6 , 1 is present at R 2 , R 4 , R 6
In C 2 , 1 is present at R 3
In R 6 , 1 is present at C 2 , C 4 , C 6
In R 2 , 1 is present at C1 and C 4
Thus add new entries in W6 at (R 2 , C 2 ), (R 2 , C 4 ), (R 2 , C 6 ), (R 4 , C 2 ), (R 4 , C 4 ),
(R 4 , C 6 ), (R 6 , C 2 ), (R 6 , C 4 ), (R 6 , C 6 ) which is given below Thus add new entries in W2 at (R 3 , C1 ), (R 3 , C 4 ) which is given below.

W6 = W5 = W4
0 0 0 1
Hence W6 is the relation matrix of R * W2 =
1 0 0 1
1 1 0 1
ì (1, 1), (1, 3) (1, 5) ( 2, 2) ( 2, 4) ( 2, 6) ( 3, 1) ( 3, 3) ( 3, 5) ü
\ R* = í ý
0 0 1 0
î( 4 , 2) ( 4 , 4) ( 4 , 6) ( 5, 1) ( 5, 3) ( 5, 5) ( 6, 2) ( 6, 4) ( 6, 6)þ
Step 4 : To find W3 :
Example 4.9.5 Let R = {(a, d), (b, a), (b, d), (c, b), (c, d), (d, c)}
To find W3 from W2 , we consider the 3 rd column and 3 rd row.
Use Warshall's algorithm to find the matrix of transitive closure where A = {a, b, c, d}
In C 3 , 1 is present at R 4
SPPU : Dec.-15
In R 3 , 1 is present at C1 , C 2 , C 4
Solution : Step 1 : We have
Thus add news entries in W3 at (R 4 , C 1 ), (R 4 , C 2 ), (R 4 , C 4 )
A = {a, b, c, d} \ |A| = 4
Which is given below
R = {(a, d) (b, a), (b, d) (c, b), (c, d), (d, c)}

Thus we have to find Warshall's sets Wo , W1 , W2 , W3 , W4 0 0 0 1


1 0 0 1
The first set Wo = M R W3 =
1 1 0 1
1 1 1 1

TECHNICAL PUBLICATIONS® - An up thrust for knowledge TECHNICAL PUBLICATIONS® - An up thrust for knowledge
Discrete Mathematics 4 - 47 Relations Discrete Mathematics 4 - 48 Relations

Step 5 : To find W4 :
To find W4 from W3 , we consider the 4 th column and 4 th row.
0 1 1 1
1 1 1 1
W1 =
In C 4 , 1 is present at R 1 , R 2 , R 3 , R 4 0 1 0 1
In R 4 , 1 is present at C1 , C 2 , C 3 , C 4 0 1 1 0

Thus we add new entries in W4 at (R1 , C1 ), (R1 , C2 ), (R1 , C3 ), (R1 , C4 ), (R 2 , C1 ),


Step 3 : To find W2 :
(R 2 , C2 ), (R 2 , C3 ), (R 2 , C4 ), (R 4 , C3 ), (R 3 , C1 ), (R 3 , C2 ), (R 3 , C4 ), (R 4 , C1 ), (R 4 , C2 ),
(R 4 , C3 ), (R 4 , C4 ) To find W2 from W1 , we consider the 2 nd column and 2 nd row.
Which is given below In C 2 , 1 is present at R 1 , R 2 , R 3 , R 4
é1 1 1 1ù In R 2 , 1 is present at C1 , C 2 , C 3 , C 4
ê1 1 1 1ú
W4 = ê ú Thus we add new entries in W2 at (R1 , C1 ), (R1 , C2 ), (R1 , C3 ), (R1 , C4 ), (R 2 , C1 ),
ê1 1 1 1ú
(R 2 , C2 ), (R 2 , C3 ), (R 2 , C4 ), (R 3 , C1 ), (R 3 , C2 ), (R 3 , C2 ), (R 3 , C4 ), (R 4 , C1 ), (R 4 , C2 ),
ê1 1 1 1úû
ë (R 4 , C3 ), (R 4 , C4 )
Hence W4 is the relation matrix of R * . Which is given below
ì(a, a), (a, b) (a, c) (a, d) (b, a) (b, b) (b, c) (b, d)ü é1 1 1 1ù
and R* = í ý
î(c, a), (c, b) (c, c) (c, d) (d, a) (d, b) (d, c) (d, d)þ ê1 1 1 1ú
W2 = ê ú
ê1 1 1 1ú
Example 4.9.6 Find the transitive closure of the relation R on ê1
ë 1 1 1úû
A = {1, 2, 3, 4} defined by
R = {(1, 2), (1, 3), (1, 4), (2, 1), (2, 3), (3, 4), (3, 2), (4, 2), (4, 3)} All entries in W2 are 1
SPPU : Dec.-07, May-15, 18, Marks 4 Hence W2 is the relation matrix of transitive closure of R.
Solution : Step 1 :
ì(1, 1), (1, 2) (1, 3) (1, 4) (2, 1) (2, 2) (2, 3) (2, 4)ü
and R* = í ý
We have |A| = 4, Thus we have to find î(3, 1), (3, 2) (3, 3) (3, 4) (4, 1) (4, 2) (4, 3) (4, 4)þ
Warshall's sets, W0 , W1 , W2 , W3 , W4
Example 4.9.7 If R = {(a, b), (b, a), (b, c), (c, d), (d, a)} be a relation on the set A = {a, b, c,
The first set W0 = M R d}. Find the transitive closer of R using Warshall's algorithm. SPPU : Dec.-16, Marks 6
Solution :
1 2 3 4
1 0 1 1 1 Step 1 : We have A = {a, b, c, d}
0 1
W0 = MR = 2 1 0
R = {(a, b), (b, a), (b, c), (c, d), (d, a)}
3 0 1 0 1
4 0 1 1 0 Thus we have to find Warshall's sets W 0 , W 1 , W 2 , W 3 , W 4 .
The first set W0 = MR a b c d
Step 2 : To find W1 :
a 0 1 0 0
To find W1 from W0 , we consider the first column and first row. b 1 0 1 0
\ W0 = MR = c 0 0 0 1
In C1 , 1 is present at R 2 d 1 0 0 0
In R 1 , 1 is present at C 2 , C 3 , C 4
Thus add new entries in W1 at (R 2 , C2 ), (R 2 , C3 ), (R 2 , C4 ) which is given below

TECHNICAL PUBLICATIONS® - An up thrust for knowledge TECHNICAL PUBLICATIONS® - An up thrust for knowledge

Discrete Mathematics 4 - 49 Relations Discrete Mathematics 4 - 50 Relations

Step 2 : To find W 1 In R 4 , 1 is present at C 1 , C 2 , C 3


To find W 1 from W 0 , we consider the first column and first row. In C 1 , 1 is present \ Add new entries in W 3 at (R 1 , C 1 ), (R 1 , C 2 ), (R 1 , C 3 ), (R 2 , C 1 ), (R 2 , C 2 ) ,(R 2 , C 3 ),
R 2 and R 4 (R 3 , C 1 ), (R 3 , C 2 ), (R 3 , C 3 )
In R 1 , 1 is present at C 2 é1 1 1 1ù
ê1 1 1 1ú
Thus add new entries in W 1 at (R 2 , C 2 ) (R 4 , C 2 ) \ W4 = ê ú
ê1 1 1 1ú
ê1 1 0 0úû
0 1 0 0 ë
\ W1 = 1 1 1 0 Hence the W 4 is the relation metrix of R*
0 0 0 1
1 1 0 0 ì(a, a) (a, b) (a, c) (a, d) (b, a) (b, b) (b, c)ü
And R* = í ý
Step 3 : To find W 2 î(b, d) (c, a) (c, b) (c, c) (c, d) (d, a) (d, b)þ
To find W 2 from W 1 , we consider second column and second row of W 1 . Example 4.9.8 Find the transitive closure of R by Warshall's algorithm. A = {Set of positive
In C 2 , 1 is present at R 1 , R 2 , R 3 integers £ 10}.
In R 2 , 1 is present at C 1 , C 2 , C 3 R = {(a, b) / a divides b} SPPU : May-07

\ Add new entries in W 2 at (R 1 , C 1 ), (R 1 , C 2 ), (R 1 , C 3 ) (R 2 , C 1 ), (R 2 , C 2 ), (R 2 , C 3 ) Solution :

(R 3 , C 1 ), (R 3 , C 2 ), (R 3 , C 3 ) Step 1 : We have |A| = 10. Thus we have to find


Warshall's sets W0 , W1 , W2 , .... W10
1 1 1 0
1 1 1 0 R = {(1, 1), (1, 2), (1, 3), (1, 4), (1, 5), (1, 6), (1, 7), (1, 8), (1, 9), (1, 10),
\ W2 = 1 1 1 1
(2, 2), (2, 4), (2, 6), (2, 8), (2, 10), (3, 3), (3, 6), (3, 9), (4, 4), (4, 8),
1 1 0 0
(5, 5), (5, 10), (6, 6), (7, 7), (8, 8), (9, 9), (10, 10)}
Step 4 : To find W 3
The first set W0 = M R
To find W 3 from W 2 , we consider the III rd row and III rd column of W 2 . 1 2 3 4 5 6 7 8 9 10
In C 3 , 1 is present at R 1 , R 2 , R 3 1 é1 1 1 1 1 1 1 1 1 1ù
In R 3 , 1 is present at C 1 , C 2 , C 3 , C 4 2 ê0 1 0 1 0 1 0 1 0 1ú
ê ú
3 ê0 0 1 0 0 1 0 0 1 0ú
\ Add new entries in W 2 at (R 1 , C 1 ), (R 1 , C 2 ), (R 1 , C 3 ), (R 1 , C 4 ), (R 2 , C 1 ), (R 2 , C 2 ),
4 ê0 0 0 1 0 0 0 1 0 0ú
(R 3 , C 3 ), (R 2 , C 4 ), (R 3 , C 1 ), (R 3 , C 2 ), (R 3 , C 3 ), (R 3 , C 4 ) ê ú
\ W0 = 5 ê0 0 0 0 1 0 0 0 0 1ú
ê ú
6 ê0 0 0 0 0 1 0 0 0 0ú
1 1 1 1
\ W3 = 1 1 1 1 7 ê0 0 0 0 0 0 1 0 0 0ú
ê ú
1 1 1 1 8 ê0 0 0 0 0 0 0 1 0 0ú
1 1 0 0
9 ê0 0 0 0 0 0 0 0 1 0ú
ê ú
10 ë 0 0 0 0 0 0 0 0 0 1û
Step 5 : To find W 4
The relation R itself is a transitive relation on the set of positive integers. Hence
To find W 4 from W 3 , we consider 4 th row and 4 th column of W 3 .
R = R * and
In C 4 , 1 is present at R 1 , R 2 , R 3
W0 = M R is the relation matrix of R *
TECHNICAL PUBLICATIONS® - An up thrust for knowledge TECHNICAL PUBLICATIONS® - An up thrust for knowledge
Discrete Mathematics 4 - 51 Relations Discrete Mathematics 4 - 52 Relations

Example 4.9.9 Use Warshall's Algorithm to find transitive closure of R where Thus add new entries in W3 at (R1 , C1 ), (R1 , C2 ), (R1 , C3 ), (R 3 , C1 ), (R 3 , C2 ),
é1 0 1ù (R 3 , C3 ) which is given below
M R = ê 0 1 0ú and A = {1, 2, 3} é1 1 1ù
ê ú
êë1 1 0úû SPPU : May-06, May-08 W3 = ê 0 1 0ú
ê ú
Solution : ë1 1 1û

Step 1 : We have |A| = 3. Thus we have to find Warshall's sets W0 , W1 , W2 , W3 Hence W3 is the relation matrix of R *
The first set is and R * = {(1, 1) (1, 2) (1, 3) (2, 2) (3, 1) (3, 2) (3, 3)}

1 0 1 4.10 Partially Ordered Set SPPU : Dec.-06


W0 = MR = 0 1 0
1 1 0 A relation R on a set A is called a partially ordered relation iff R is reflexive,
anti-symmetric and transitive relation.
Step 2 : To find W1 :
The set A together with partially ordered relation is called a partially ordered set or
To find W1 from W0 , we consider the first column and the first row. POSET.
In C1 , 1 is present at R 1 , R 3 It is denoted by (A, R) or (A, £) where £ is a partially ordered relation.
In R 1 , 1 is present at C1 and C 3
Examples :
Thus add new entries in W1 at (R1 , C1 ), (R1 , C3 ), (R 3 , C1 ), (R 3 , C3 )
1) (Â, £) (N, £) are Posets.
Which is given below
where '£' is reflexive, antisymmetric and transitive relation.
é1 0 1ù
W1 = ê 0 1 0ú 2) If A = P(S) where S = (a, b, c) and for X, Y Î A, Define X £ Y or XRY iff X Í Y.
ê ú
êë1 1 1úû As X £ X Þ X £ X. \ ' £ ' is reflexive.
If X £ Y, Y £ Z Þ X Í Y and Y Í X Þ X = Y
Step 3 : To find W2 :
\ ' £ ' is antisymmetric relation.
To find W2 from W1 , we consider the 2 nd column and 2 nd row.
If X £ Y, Y £ Z Þ X Í Y and Y Í Z Þ X Í Z Þ X £ Z
In C 2 , 1 is present at R 2 , R 3
Þ ' £' is transitive relation.
In R 2 , 1 is present at C 2
\ (P(S), Í) or (P(S), £) is a poset.
Thus add new entries in W2 at (R 2 , C2 ), (R 3 , C2 ) which is given below
é1 0 1ù
I) Comparable elements : Let (A, £) be a poset. Two elements a, b in A are said to be
W2 = ê 0 1 0ú = W1 comparable elements if a £ b or b £ a. Two elements a and b of a set A are said to be
ê ú
êë1 1 1úû non-comparable if neither a £ b nor b £ a.
In above example (2),
Step 4 : To find W3
The comparable elements are
To find W3 from W2 , we consider the 3 rd column and 3 rd row.
In C 3 , 1 is present at R 1 , R 3 {a} = {a, b}, {b} = {a, b, c} {b, c} Í {a, b, c}
In R 3 , 1 is present at C1 , C 2 , C 3 Non comparable elements are
{a} Ë {b } {a} Ë {a, c}

®
TECHNICAL PUBLICATIONS® - An up thrust for knowledge TECHNICAL PUBLICATIONS - An up thrust for knowledge

Discrete Mathematics 4 - 53 Relations Discrete Mathematics 4 - 54 Relations

II) Totally ordered set : Let A be any nonempty set. The set A is called linearly {a,b,c}
Level 4
ordered set or totally ordered set if every pair of elements in A are comparable.
i.e. for any a, b Î A either a £ b or b £ a .
Level 3 {a,b} {a,c}
{b,c}
4.10.1 Hasse Diagram

It is useful tool, which completely describes the associated partially ordered relation. Level 2 {a}
{b} {c}
It is also known as ordering diagram.
A diagram of graph which is drawn by considering comparable and non-comparable Level 1
elements is called Hasse diagram of that relation. Therefore while drawing Hasse 
diagram following points must be followed. Fig. 4.10.1
4.10.2 Chains and Antichains
1) The elements of a relation R are called vertices and denoted by points.
Let (A, £) be a poset. A subset of A is called a chain if every pair of elements in the
2) All loops are omitted as relation is reflexive on poset.
subset are related.
3) If aRb or a £ b then join a to b by a straight line called an edge the vertex b A subset of A is called antichain if no two distinct elements in a subset are related.
appears above the level of vertex a. Therefore the arrows may be omitted from e.g. In above example (4.10.1)
the edges in Hasse diagram. 1) The chains are {f, {a }, {a, b}, {a, b, c}}, {{a}, {a, c}, {a, b, c}}, {{b, c}, {a, b, c}}
4) If a £/ b and b £/ a i.e. a and b are non - comparable elements, then they lie on same 2) Antichains are {{a}, {b}, {c}}
level and there is no edge between a and b.
Note : 1) The number of elements in the chain is called the length of chain.
5) If a £ b and b £ c then a £ c. So there is a path a ® b ® c. Therefore do not join a 2) If the length of chain is n in a poset (A, £ ) then the elements in A can be
to c directly i.e. delete all edges that are implied by transitive relation. partitioned into n disjoint antichains.
Example 4.10.1 Draw Hasse diagram of a poset (P(s), Í) where S = {a, b, c}.
4.10.3 Elements of Poset
Solution : P (s) = { f, {a}, {b}, {c}, {a, b} {a, c} {b, c} {a, b, c}}
1) Let (A, £ ) be a poset. An element a Î A is called a maximal element of A if there
Now find the comparable and non comparable elements. is no element c Î A such that a £ c.
f Í {a}, fÍ {b}, fÍ {c}, \ {a}, {b}, {c} lie above the level of f. 2) An element b Î A is called a minimal element of A if there is no element c Î A
such that c £ b
{a} Í {a, b}, {b}Í {a, b}, {c} Í {a, c} \ {a, b}, {b, c} {a,c} lies above the level of {a}, {b}, {c}.
3) Greatest element : An element x Î A is called a greatest element of A if for all
{a, b} Í S, {b, c} Í S, {a, c} Í S \ S lies above the level of {a, b}, {a, c}, {b, c}
a Î A, a Î x. It is denoted by 1 and is called the unit element.
But {a}, {b} {c{ are non comparable \ {a}, {b}, {c} lie on same level. 4) Least element : An element y Î A is called a least element of A if for all a Î A,
{a, b}, {a, c}, {b, c} are non comparable \ lie on same level. y £ a.

By considering the above observations, the Hasse diagram is as follows : It is denoted by 0 and is called as zero element.
5) Least upper bound (lub) : Let (A, £ ) be a poset. For a, b, c Î A, an element C is
called upper bound of a and b if a £ c and b £ c C is called as least upper bound
of a and b in A if C is an upper bound a and b there is no upper bound d of a
and b such that d £ c. It is also known as supremum.

® ®
TECHNICAL PUBLICATIONS - An up thrust for knowledge TECHNICAL PUBLICATIONS - An up thrust for knowledge
Discrete Mathematics 4 - 55 Relations Discrete Mathematics 4 - 56 Relations

6) Greatest lower bound (glb) : Let (A, £ ) be a poset. for a, b, l Î A, an element l is 2) 5, 15, 3 are lower bounds of 30 and 45. But 15 is the greatest lower bound of 30
called the lower bound of a and b if l £ a and l £ b. to 45.
An element l called the greatest lower bound of a and b if l is the lower bound of a 3) This Poset has neither greatest element nor least element.
and b and there is no lower bound f of a and b such that · l £ f . 4) This poset has two maximal elements 45 and 30 as there is no element c such
glb is also called as infimum. that 45 £ C and 30 £ C.

Example 4.10.2 Determine the greatest and least elements of the poset whose Hasse diagrams 5) This poset has three minimal elements 2, 3 and 5. because there is no element
are shown below. x Î A such that x £ 2, x £ 3 and x £ 5

d 4.10.4 Types of Lattices


d e d c d
c A lattice is a poset in which every pair of elements has a least upper bound (lub) and
c b b c
b a greatest lower (glb).
a b a a
a
Let (A, £) be a poset and a, b Î A then lub of a and b is denoted by a Ú b. It is called
the join of a and b.
I II III IV
i.e. a Ú b = lub (a, b)
Fig. 4.10.2
The greatest lower bound of a and b is called the meet of a and b and it is denoted
Solution : The Poset shown in Fig. 4.10.2 (I) has neither greatest not least element. by a Ù b

The Poset shown in Fig. 4.10.2 (II), has greatest and a as least element. \ a Ù b = glb (a, b)

The Poset shown in Fig. 4.10.2 (III), has no greatest element but a is the least element. From the above discussion, it follows that a lattic is a mathematical structure with
The Poset shown in Fig. 4.10.2 (IV), has greatest and a as least element. two binary operations Ú (join) and Ù (meet). It is denoted by {L, Ú , Ù }.

Example 4.10.3 Find glb, lwb, ub, lb, maximal, minimal, of the poset (A, R), Here aRb if Examples :
a / b where. A = { 2, 3, 5, 6, 10, 15, 30, 45} Example 4.10.4 Let A = {1, 2, 3} P (A) = { f, {1}, {2}, {3}, {1, 2}, {1, 3}, {2, 3}, {1, 2, 3 } show that
Solution : We have A = {2, 3, 5, 6, 10, 15, 30, 45} and aRb iff a|b. P(A), Í ) is a lattice
Hasse diagram is as follows : Solution : The Hasse diagram of the poset (P(A), Í) is given below :

30 {a,b,c}
Level 3 45

{a,b} {a,c}
Level 2 6 10 {b,c}
15

Level 1 {a}
2 {b} {c}
3 5

Fig. 4.10.3

1) Here 10 and 30 are upper bounds of 2 and 5, But 10 is the least upper bound of
Fig. 4.10.4
2 and 5.
Here every pair of elements of a poset has lub and glb. Hence (P(A), Í) is a lattice.
® ®
TECHNICAL PUBLICATIONS - An up thrust for knowledge TECHNICAL PUBLICATIONS - An up thrust for knowledge

Discrete Mathematics 4 - 57 Relations Discrete Mathematics 4 - 58 Relations

Example 4.10.5 Determine which of the following posets are lattice. 4.10.5 Properties of a Lattice

f Let (L, Ù , Ú) be a lattice and a, b, c Ï L. Then L satisfies the following properties.


6 4
5 d c
3
1) Commutative property
d e
2
2 b a Ù b = b Ù a and a Ú b = b Ú a
4 c
3 1
a 2) Associative law
1 a b

I II III IV
a Ú (b Ù c) = (a Ú b) Ù (a Ú c)

Fig. 4.10.5 a Ù (b Ú c) = (a Ù b) Ú (a Ù c)

Solution : I) In Fig. 4.10.5 (I), every pair of elements has lub and glb. 3) Absorption law
\ It is a lattice. a Ù (a Ú b) = a and av (a Ù b) = a
II) In Fig. 4.10.5 (II), every pair of elements has lub and glb. 3) a Ù a = a, a Ú a = a
\ It is a lattice. 4) a Ù b = a iff a Ú b = b
II) In Fig. 4.10.5 (III), a Ù b does not exist.
4.10.6 Types of Lattices
\ It is not a lattice.
I) Bounded lattice : A lattice L is called a bounded lattice if it has a greatest element 1
IV) In Fig. 4.10.5 (IV), c Ú d does not exist.
and least element 0.
\ It is not a lattice.
II) Sublattice : Let, (L, Ú , Ù ) be a lattice. A non empty subset L 1 of L is called a
Example 4.10.6 Let A be the set of positive factors of 15 and R be a relation on A s.t.
sublattice of L if L 1 itself is a lattice w.r.t. the operations of L.
R = {xRy| x divides y, x y Î A}. Draw Hasse diagram and give and Ù and Ú for lattice.
SPPU : Dec.-06 III) Distributive lattice : A lattice (L, Ú , Ù ) is called a distributive lattice if for any
Solution : We have A = {1, 3, 5, 15} elements a,b,c Î L, it satisfies the following properties,
i) a Ú (b Ù c) = ( a Ú b) Ù (a Ú c)
R = {(1,1) (1,3) (1,5) (1, 15) (3,15) (5,15) (15,15)}
ii) a Ù (b Ú c) = ( a Ù b) Ú (a Ù c)
Hasse diagram of R is : Table for Ù and Ú
If the lattice does not satisfy the above properties then it is called a non distributive
Ú 1 3 5 15 Ù 1 3 5 15 lattice.
15 1 1 3 5 15 1 1 1 1 1
Theorem : Let, (L, Ù , Ú) be a lattice with universal bounds 0 and 1 then for any
3 3 3 15 15 3 1 3 1 3 a Î L, a Ú 1 =1, a Ù 1 = a, 0 Ú a = a, 0 Ù a = 0.
5 3
5 5 15 5 15 5 1 1 5 5 30  1
III) Complement lattice : Let (L, Ù , Ú) be a lattice with
1
15 15 15 15 15 15 1 3 5 15 universal bounds 0 and 1 for any a Î L, b Î L is said to be
2 3 5
complement of a if a Ú b = 1 and a Ù b = 0.
Fig. 4.10.6 A Lattice in which every element has a complement in that 10
lattice, is called the complemented lattice.
Every pair of elements has lub and glb. \ It is a lattice. Fig. 4.10.7
e.g. 1) The Hasse diagram is here 0 » 1 and 1 » 30.

® ®
TECHNICAL PUBLICATIONS - An up thrust for knowledge TECHNICAL PUBLICATIONS - An up thrust for knowledge
Discrete Mathematics 4 - 59 Relations Discrete Mathematics 4 - 60 Relations

i) 2 Ù 3 = 0 and 2 Ú 3 = 1, 2 Ù 5 = 0 and 2 Ú 5 = 1 i) Chains are


{1, 2, 4, 12}
\ 2 has two compliments 3 and 5
{1, 2, 6, 12}
Hence the complement is not unique.
{1, 3, 6, 12}
4.11 Principle of Duality SPPU : Dec.-10, 11, 13, 14, 15, 16, May-14, 15, 19 ii) Antichains are
{6, 9} {9, 12}
Any statement about lattice involving Ù , Ú £ , ³ remains true if 'Ù' is replaced by 'Ú',
{4, 9}
'Ú by Ù ', ' £ by ³', ' ³ by £', '0 by 1' and '1 by 0'.
e.g. 1) a Ú (b Ù c) = a Ù (b Ú c) Example 4.11.2 Determine whether the poset represented by each of the Hasse diagram are
lattices. Justify your answer.
2) a Ù (b Ú 1) = a Ú (b Ù 0)
6 h
Examples f

e 4 5 c g
Example 4.11.1 Let A = {1, 2, 3, 4, 6, 9, 12} Let a relation R on a set A is R = {(a,b)/ a
f
divides b " a, b Î A}. Give list of R. Prove that it is a partial ordering relation. Draw c d
c
Hasse diagram of the same. Prove or disprove it is a lattice. Give two examples of chain b
2 3 b d
and antichains. SPPU : Dec.-11, May-19, Marks 6 a
1 a
Solution : We have A = {1, 2, 3, 4, 6, 9, 12}
  
ì(1,1), (1,2), (1,3), (1,4) (1,6), (1,9), (1,12), (2,2), (2,4), (2,6),(2,12) (3,3), (3,6), (3,9) ü Fig. 4.11.2 SPPU : Dec.-10
and R = í ý
î(3,12), (4,4), (4,12), (6,6), (6,12), (9,9), (12,12) þ Solution :

We know that for any a Î A, a | a \ aRa I) In Fig. 4.11.2 (I), every pair of element has glb and lub. \ It is a lattice.

\ R is a reflexive relation. II) In Fig. 4.11.2 (II), every pair of elements has lub and glb. \ It is a lattice.
III) In Fig. 4.11.2 (III), every pair of elements gas lub and glb. \ It is a lattice.
As a | b and b | a Þ a = b \ R is antisymmetric relation.
As a | b and b | c Þ a | c Þ R is a transitive relation. Example 4.11.3 Show that the set of all divisors of 36 forms a lattice. SPPU : Dec.-14

\ R is reflexive antisymmetric and transitive Solution : Let A = {1, 2, 3, 4, 6, 9,12, 18, 36 } and Let '£' is a divisor of.
\ (A, R) is a poset and R is a partial ordering relation. It's Hasse diagram is as follows.
Hasse diagram is as follows : 36  1
12
12 18

4 6 9
4 6 9

3
2
2 3
1
10
Fig. 4.11.1 Fig. 4.11.3
In above diagram 6 Ú 9 does not exist. \ It is not a lattice.
® ®
TECHNICAL PUBLICATIONS - An up thrust for knowledge TECHNICAL PUBLICATIONS - An up thrust for knowledge

Discrete Mathematics 4 - 61 Relations Discrete Mathematics 4 - 62 Relations

The universal upper bound 1 is 36 and lower bound 0 is 1. Every pairs of elements It's Hasse diagram is as follows :
of this poset has lub and glb.
70
\ It is a lattice.
Example 4.11.4 SLet n be a positive integer, S n be the set of all divisors of n, Let D denote
10 14
the relation of divisor. Draw the diagram of lattices for n = 24, 30, 6. SPPU : May-15 35

Solution : Given that


2
5 7
i) We have S 6 = {1,2,3,6}, D is the relation of divisor.
ii) S 24 = {1, 2, 3, 4, 6, 8, 12, 24}
1
iii) S 30 = {1, 2, 3, 5, 6, 10, 15, 30} Fig. 4.11.5
Diagrams of Lattices are as follows. Every pair of elements of A has Ù and Ú.
\ It is a lattice [write table of Ù and Ú].
24
Example 4.11.6 Let A = {1, 2, 3, 4 , 5, 6, 7, 8 ,9, 12, 18, 24} be ordered by the relation x
12 30
divides y. Show that the relation is a partial ordering and draw Hasse diagram.
8
SPPPU : Dec-15

6 6 6 10 Solution : We have A = {1, 2, 3, 4, 5, 6, 7, 8, 9, 12, 18, 24}


4 15
R = {(x,y) | x divides y, for x, y Î A}
2 R = {(1, 2) (1, 3), (1, 4), (1, 5) (1, 6) (1, 7) (1, 8) (1, 9) (1, 12) (1, 18) (1, 24)
2 3 2 3 3 5
(2, 2) (2, 4) (2, 6) (2, 8) (2, 12) (2, 18) (2, 24) (5, 5) (6, 6) (6, 12) (6, 18)
(6, 24), (7, 7) (8, 8) (8, 24) (9, 9) (9, 18) (12, 12) (12, 24) (18, 18) (4, 24)}
1 1 1
We have for any x Î A, x|x Þ R is a reflexive for x|y and y|x Þ x = 0 Þ R is
S6 S24 S30
antisymmetric. If x|y and y|z Þ x|z \ R is a transitive relation.
\ R is a partial ordering relation, It's Hasse diagram is as follows.
Fig. 4.11.4
24
Example 4.11.5 Show that the set of all divisors of 70 forms a lattice.
SPPU : Dec.-13, May-19, Marks 3 8 18
12
Solution : Let A = {1, 2, 5, 7, 10, 14, 35, 70}

and Let ' £' is "a divisor of". 4 6 9

The universal upper bound 1 is 70 and the lower bound 0 is 1.


5 7
3
2

1
Fig. 4.11.6

® ®
TECHNICAL PUBLICATIONS - An up thrust for knowledge TECHNICAL PUBLICATIONS - An up thrust for knowledge
Discrete Mathematics 4 - 63 Relations Discrete Mathematics 4 - 64 Relations

Example 4.11.7 Let x = {2, 3, 6, 12, 24, 36} and x £ y iff x divides y find Solution : Let
i) Maximal element ii) Minimal element iii) Chain iv) Antichain v) Is Poset lattice
SPPU : May-14 A = {a, b, c, d, e}
Solution : We have x = {2, 3, 6, 12, 24, 36} From the given Hasse diagram, every pair of element has glb and lub. Hence it is a
The relation 'R' = '£' lattice.
\ R = {(2, 2) (2, 6) (2, 12) (2, 24) (2, 36) (3, 3) (3, 6) (3, 12) (3, 24) (3, 36),
qqq
(6, 6) (6, 12) (6, 24) (6, 36) (12, 12)(12, 24) (12, 36) (24, 24) (36, 36)}.
It's Hasse diagram is as follows .

36 24

12

2 3

Fig. 4.11.7
i) Maximal elements are 24, 36
ii) Minimal elements are 2, 3
iii) Chain { 2, 6, 12, 24}, { 2, 6, 12, 36}, {3, 6, 12, 24}, {3, 6, 12, 36}
iv) Antichain : {2, 3} {24, 36}
v) The given poset is not a lattice as 2 Ù 3 does not exist.
Example 4.11.8 The following is the Hasse diagram of the poset :
{(a, b, c, d, e), <} Is it a lattice ? Justify SPPU : Dec.-16, Marks 2

b c d

e
Fig. 4.11.8

® ®
TECHNICAL PUBLICATIONS - An up thrust for knowledge TECHNICAL PUBLICATIONS - An up thrust for knowledge

Discrete Mathematics 5-2 Functions

5.1 Introduction
The concept of relation was defined very generally in the preceeding chapter. We
shall now discuss a particular class of relations called functions. Many concepts of
computer science can be conveniently stated in the language of functions.

5.2 Function SPPU : May-07, 08, 15, Dec.-07, 12,

Let A and B be non-empty sets. A function f from A to B is denoted by f : A ® B


and defined as a relation from A to B such that for every a Î A, there exists a unique
b Î B such that (a, b) Î f .
If (a, b) Î f then it can be written as f(a) = b. Functions are also called as Mappings or
Transformations.
e.g.
1) f = {(a, b) (b, c) (c, a)} is a function on set A = {a, b, c}
2) f = {(a, b) (b, c) (b, a)} is not a function as (b, c) and (b, a) Î f .
3) f = {(a, b) (b, c)} is not a function on set A = {a, b, c} as for C Î A $/ any element x
in A such that (c, x) Î f .

5.2.1 Important Definitions

I) Let f : A ® B be a function such that f(a) = b then b Î B is called the image of


a Î A and a is called the pre-image of b. The element a is called argument of f.
II) Let f : A ® B be a function. The set A is known as domain set of f and the set B
is known as co-domain set of f.
III)The range set of a function f : A ® B is denoted by R(f) and defined as
R(f) = {b b Î B and f(a) = b for some a Î A }

In other words, range of f is the set of all images of the elements of A under f.

Examples

Example 5.2.1 If f is a function such that f(x) = x 2 – 1 where x Î R find the values of f(–1),
f(0), f(1), f(3) and range set of f.
Solution : We have f(x) = x 2 – 1

\ f(–1) = (–1) 2 – 1 = 1 – 1 = 0
f(0) = 0 – 1 = –1
f(1) = 1 2 – 1 = 0
f(3) = 3 2 – 1 = 8

TM TECHNICAL PUBLICATIONS® - An up thrust for knowledge


TECHNICAL PUBLICATIONS - An up thrust for knowledge
Discrete Mathematics 5-3 Functions Discrete Mathematics 5-4 Functions

For any x Î Â, x 2 – 1 Î Â 5.2.3 Equality of Two Functions


\ The range of f is R(f) = {x –1 £ x < ¥} Two functions f : A ® B and g : A ® B are said to be equal functions or identical
functions iff.
Example 5.2.2 Let f : Â ® Â where f is defined by
a) f(x) = x f(x) = g(x) " x Î A
b) f(x) = x 2 Note : Two functions f : Z ® Z and g : Â ® Â defined as f(x) = x, " x Î Z and
c) f(x) = sin x " x ÎÂ g(x) = x, " x Î Â are not identical or equal functions because their domains are not
Solution : same.
a) We have f(x) = x \ f(4) = 4 = ±2 5.2.4 Identity Function
\ f is not a function. Therefore we can not find range of f. Let A be any non empty set and function f : A ® A is said to be the identity function
b) We have f(x) = x 2 , x 2 is positive real number if f(x) = x, " x Î A.

Hence range of f is R(f) = {x |0 £ x < ¥} e.g.


f
c) " x Î Â, sin x lies between –1 to +1 A A

\ R(f) = {x | – 1 £ x £ 1} 1 1
2 2 A is the identity function
f:A
3 3
5.2.2 Partial Functions 4 4

Let A and B be two non empty sets. A partial function f with domain set A and
Fig. 5.2.2
codomain set B is any function from A' to B where A' CA. For any x Î A – A' , f(x) is not
defined. 5.2.5 Constant Function
To make the distinction more clear, the function which is not partial is called as a A function f : A ® B is said to be a constant function if f(x) = constant = k ; " x Î A.
total function.
The range set of a constant function consists of only one element.
e.g.
5.2.6 Composite Function
1)
Let f : A ® B and g : B ® C be two functions.
A f B The composite function of f and g is denoted by gof and defined as gof : A ® C is a
1 a function such that (gof) (a) = g[f(a)] " a Î A.
A' 2 b f : A' B is a partial function
3 c Note : gof is defined only when the range of f is a subset of the domain of g.
4 d
5 e e.g. 1)
2
Fig. 5.2.1 A f(x) = x B g(x) = x C gof
Therefore gof : A C A C
1 1 1 1 gof (1) = g[f(1)] = g(1) = 1 1 1
2) The function f : Â ® Â defined as f(x) = is a partial function. As it is not
x 2 4 4 gof (2) = g[f(2)] = g(4) = 4 2 4
9
3 9 9 3
defined for x = 0. gof (3) = g[f(3)] = g(9) = 9 16
4 16 16 4
25 gof (4) = g[f(4)] = g(16) = 16 25
25
3) The function f : Â ® Â defined as f(x) = x is a partial function, as x is not 36 36
defined for x < 0, in Â.
Fig. 5.2.3 Fig. 5.2.4

TECHNICAL PUBLICATIONS® - An up thrust for knowledge TECHNICAL PUBLICATIONS® - An up thrust for knowledge

Discrete Mathematics 5-5 Functions Discrete Mathematics 5-6 Functions

Example 5.2.3 Let f(x) = x+2, g(x) = x – 2, h(x) = 3x, for x Î Â Where  is the set of real c) fog(5) = f[g(5)] = f[25 – 2] = f(23) = 2(23) + 1 = 47
numbers
d) gof(a + 3) = g[f(a + 3)] = g[2(a + 3) + 1] = g[2 a + 7]
Find i) gof ii) fog iii) fof iv) hog v) gog vi) foh vii) hof viii) fohog ix) gofoh.
SPPU : May-08, 15, Dec.-12, Marks 4 = [2 a + 7] 2 – 2 = 4 a 2 +28 a + 47
Solution : Let x Î Â be any real number. 5.3 Special Types of Functions SPPU : Dec.-10, 11, 16, 17, May-18

i) gof(x) = g[f(x)] = g[x + 2] = x + 2 – 2 = x I) Let f : A ® B be a function.


ii) fog(x) = f[g(x)] = f[x – 2] = x – 2 + 2 = x i) Function f is said to be one to one (or Injective) function if distinct elements of A
iii) fof(x) = f[f(x)] = f[x + 2] = x + 2 + 2 = x + 4 are mapped into distinct elements of B.
i.e. f is one to one if
iv) hog(x) = h[g(x)] = h[x – 2] = 3(x – 2) = 3x – 6
a 1 ¹ a 2 Þ f(a 1 ) ¹ f(a 2 )
v) gog(x) = g[g(x)] = g[x – 2] = x – 2 – 2 = x – 4
OR f(a 1 ) = f(a 2 ) Þ a 1 = a 2
vi) foh(x) = f[h(x)] = f[3 x] = 3x + 2
ii) Function f is said to be onto function if each element of B has at least one
vii) hof(x) = h[f(x)] = h[x + 2] = 3(x + 2) = 3x + 6
preimage in A.
viii) fohog(x) = f[h(g(x))] = f[h(x – 2)]
OR
= f[3(x – 2)] = f(3 x – 6)
A function f : A ® B is called onto (or surjective) function if the range set of f is equal
= 3x – 6 + 2 = 3 x – 4 to B.

ix) gofoh(x) = g[f(h(x))] = g[f(3 x)] iii) A function f is called a objective function if it is both one to one and onto.

= g[3 x + 2] = 3 x + 2 – 2 = 3 x iv) A function f is called into function if $ at least one element in B which has no
preimage in A.
Example 5.2.4 Let functions f and g be defined by f(x) = 2x + 1, g(x) = x 2 – 2
i.e. A function f is called into function if R(f) ¹ B
Find a) gof(4) and fog (4)
2) Let a function f : A ® B be a bijective function then f –1 : B ® A is called the
b) gof(a + 2) and fog(a + 2)
inverse mapping of f and defined as f(b) –1 = a iff f(a) = b
c) fog(5)
It is also known as invertible mapping.
d) gof(a + 3) SPPU : May-07, Dec.-07, Marks 4
Solution : 3) Characteristic function of a set :
Let U be a universal set and A be a subset of U.
a) gof(4) = g[f(4)] = g[2(4) + 1] = g[9] = 9 2 – 2 = 79
Then the function y A : U ® {0, 1} defined by
fog(4) = f[g(4)] = f(4 2 – 2) = f(14) = 2(14) + 1 = 29
ì1 if x Î A
y A (x) = í
î0 if x Ï A
b) gof(a + 2) = g[f(a + 2)] = g[2(a + 2) + 1] = g[2a + 5]
2
= ( 2 a +5) – 2 = 4 a +20a + 23
2
is called a characteristic function of the set A.
fog(a + 2) = f[g(a + 2)] = f[(a +2) 2 – 2] = f[a 2 + 4a +2]

= 2[(a 2 + 4 a +2] +1 = 2 a 2 +8 a +5

TECHNICAL PUBLICATIONS® - An up thrust for knowledge TECHNICAL PUBLICATIONS® - An up thrust for knowledge
Discrete Mathematics 5-7 Functions Discrete Mathematics 5-8 Functions

Examples d) Injective as well as surjective i.e. bijective


Example 5.3.1 Give examples of functions of the following types by diagrams. A f B
a) Injective function but not surjective
1 1
b) Surjective but not injective 2 4
3 9
c) Neither injective nor surjective 4 16
5 25
d) Injective as well as surjective
e) Into function
Fig. 5.3.4
f) Inverse function
e) Into function
Solution :
a) Injective but not surjective A f B

2
f(x) = x 1 1
A B
2 2
1 3 3
1
2 4
3 9
4 16 Fig. 5.3.5
25
f) Inverse function
Fig. 5.3.1
–1
A f B f A
b) Surjective but not injective
1 1 1
A B 2 4 2
3 9 3
–2 4 16 4
–1 0
0 1
1 4 Fig. 5.3.6
2

Example 5.3.2 Determine if each is a function. If yes is it injective, surjective, bijective ?


Fig. 5.3.2
a) Each person in the earth is assigned a number which corresponds to his age.
c) Neither injective nor surjective b) Each student is assigned a teacher.
c) Each country has assigned it's capital. SPPU : Dec.-11, Marks 4
A f B
Solution :
–2
0 a) Every person has unique age
–1
1
0
1
4 \ It is a function. Two person's may have same age. \ It is not injective. There is no
9
2 person whose age is 300 years. \ It is not surjective. \ Function is not bijective.
b) It is a function. It is not injective. It is not surjective. \ It is not bijective.
Fig. 5.3.3
c) It is a function. It is injective as well as surjective. \ It is bijective.

TECHNICAL PUBLICATIONS® - An up thrust for knowledge TECHNICAL PUBLICATIONS® - An up thrust for knowledge

Discrete Mathematics 5-9 Functions Discrete Mathematics 5 - 10 Functions

Example 5.3.3 Let A = {a, b, c, d} and B = {1, 2, 3}. Determine whether the relation R from R 4 = {(1, 6), (2, 2), (3, 2), (4, 4), (5, 10)}
A to B is a function. Justify. If it is function give the range :
Every element of a domain set has unique image in codomain set so R 4 is a function.
i) R = [(a, 1), (b, 2), (c, 1), (d, 2)]
Example 5.3.5 Let A = B be the set of real numbers.
ii) R = [(a, 1), (b, 2), (a, 2), (c, 1), (d, 2)] SPPU : Dec.-16, Marks 4
f : A ® B given by f(x) = 2x 3 – 1
Solution : Given that
1 1
g : B ® A given by g(y) = 3 y +
A = {a, b, c, d}, B = {1, 2, 3} 2 2
i) R = {(a, 1), (b, 2), (c, 1), (d, 2)} Show that f is a bijective function and g is also bijective function :
SPPU : Dec.-10, Marks 4
Every element of the domain set A is related to the unique element of set B.
Solution :
\ R is a funtion from A ® B.
1) Suppose f(x 1 ) = f(x 2 )
Range of R is {1, 2}.
Þ 2x 13 – 1 = 2x 32 – 1
ii) R = {(a, 1), (b, 2), (a, 2), (c, 1) (d, 2)}
Þ x 13 = x 32
Here a ® 1 and a ® 2 hence a has two images 1 and 2 in set B. i.e. a has no unique
image in set B. Thus given relation R is not a function. Þ x1 = x2

Example 5.3.4 What are relations and functions. Given a relation R = {(1, 4), (2, 2), (3, 10), \ f is injective mapping
(4, 8), (5, 6)} and chek whether the following relations R 1 , R 2 , R 3 and R 4 is a function Let y Î B and f(x) = y Þ 2x 3 – 1 = y
or not. 1+ y
\ 2x 3 = 1+ y Þ x 3 =
R 1 = {(1, 4), (2, 4), (3, 4), (4, 4), (5, 4)} 2
R 2 = {(1, 2), (2, 4), (2, 10), (3, 8), (4, 6), (5, 4)} 3
1+ y 3
y 1
Þ x = = + Î A = Â for any y Î B
R 3 = {(1, 6), (2, 2), (4, 4), (5, 10)} 2 2 2
R 4 = {(1, 6), (2, 2), (3, 2), (4, 4), (5, 10} SPPU : Dec.-17, Marks 6 \ f is a surjective mapping.
Solution : Refer sections 4.3 and 5.2
Thus f is injective as well as surjective function.
Domain set = {1, 2, 3, 4, 5} Hence f is a bijective function.
Codomain = {2, 4, 6, 8, 10}. 1 1 1 1
We have f(x) = y Þ x = 3 y + Þ f –1 ( y) = x = 3 y + = g(y)
Given that R 1 = {(1, 4), (2, 4), (3, 4), (4, 4), (5, 4)} 2 2 2 2
All elements are mapped to 4. –1
Thus f = g. We know that if f is bijective function then f –1 is also bijective. Hence g
\ R 1 is a constant function. is bijective function.
R 2 = {(1, 2), (2, 4), (2, 10), (3, 8), (4, 6), (5, 4)} Example 5.3.6 Explain classification of functions with example.
In R 2 2 ® 4 and 2 ® 10 which is not possible in function.
Solution : Depending upon the nature of function, there are mainly two functions.
\ R 2 is not a function
1. Algebraic function : A function which consists of a finite number of terms
R 3 = {(1, 6), (2, 2), (4, 4), (5, 10)} involving powers and roots of the independent variable and four fundamental
As 3 Î D but 3 has no any image in R 3 operations addition, subtraction, multiplication and division, is called an algebraic
\ R 3 is not a function. function.

TECHNICAL PUBLICATIONS® - An up thrust for knowledge TECHNICAL PUBLICATIONS® - An up thrust for knowledge
Discrete Mathematics 5 - 11 Functions Discrete Mathematics 5 - 12 Functions

There are three types of algebraic functions. \ x 1 = x2


(a) Polynomial function : A function of the form Þ g(y) = h (y) " y
a 0 x n + a 1 x n - 1 + a 2 x n - 2 + ..... + a n , where n is positive integer, a0, a1, a2, .... ,an are real Þ g º h i.e. g and h are equal function.
numbers and a0 ¹ 0, is called polynomial function of x in degree n.
Hence universe of f is unique.
e.g. f(x) = x3 - 3x 2 + 2x - 5
Example 5.3.8 If f : A ® B and g : B ® C are bijective functions the gof is also bijective.
f(x)
(b) Rational function : A function of the form where f(x) and g(x) are
g(x) Solution : Let f : A ® B and g : B ® C be two bijective functions then gof A ® C is a
x 2 - 3x + 5 function.
polynomial functions and g(x) ¹ 0 is called rational function e.g.
x2 + 1 (i) Let x1, x2 Î A and suppose gof(x1) = gof(x2)
(c) Irrational function : A function involving radicals is called irrational function. g[f (x1)] = g[f(x2)]
e.g. f(x) = x 2/ 3 + 5x 2 + 1, 2 x + 1
Þ f(x1) = f(x2) ... (Q g is 1 – 1 function)
2. Transcendental function : A function which is not algebraic is called
transcendental function. Þ x 1 = x2 ... (Q f is 1 – 1 function)

e.g. f(x) = sin x + x3 + 5 x Þ gof is 1 – 1 function.

(a) Trigonometric function : The six functions sin x, cos x, tan x, sec x, cosec x, cot x, (ii) Let z Î C.
where x is in radians are called trigonometric functions. \ $ y in B such that g (y) = z and $ x in A such that f(x) = y.

e.g. f(x) = sin x + tan x. \ g(y) = z Þ z = g (f (x)) = gof (x) where x Î A


Þ gof is onto function.
(b) Inverse trigonometric function : The six functions sin–1 x, cos–1 x, tan–1 x,
cosec–1 x, sec–1 x and cot–1 x are called inverse trigonometric functions. Hence gof is bijective function.

e.g. f(x) = cos–1 x + 5 tan–1 x Example 5.3.9 Determine whether the function is bijective.
i
(c) Exponential function : A function of the form f (x) = ax (a > 0) satisfying f : I ® I such that f(i) = if i is even
2
a x × a y = ax + y and a' = a is called exponential fucntion. e.g. f(x) = 5x. ( i - 1)
= if i is odd
(d) Logarithmic function : The inverse function of the exponential function is called 2
logarithmic function. e.g. f(x) = log x. Solution : Let a and b be any integers such that a ¹ b.

Example 5.3.7 If f : A ® B is bijective function then f–1 is unique. There are three possibilities

Solution : Let f A ® B is a bijective function. Case 1 : If a and b are odd


a -1 b -1
Claim : Show that f–1 is unique. \ f(a) = , f(b) =
2 2
Suppose f–1 is not unique, so there are two inverse functions say, g and h. a -1 b -1
f(a) = f(b) Þ =
Let x1, x2 Î A, $ y in B such that 2 2
Þ a=b
f–1 (y) = x1 Þ g(y) = x1 Þ f(x1) = y
Case 2 : If a and b are even
and f–1 (y) = x1 Þ h(y) = x2 Þ f(x2) = y
a a
then f(a) = , f(b) =
This implies f(x1) = f (x2) , but f is 1 – 1 function. 2 2

TECHNICAL PUBLICATIONS® - An up thrust for knowledge TECHNICAL PUBLICATIONS® - An up thrust for knowledge

Discrete Mathematics 5 - 13 Functions Discrete Mathematics 5 - 14 Functions

a b Þ y3 = x – 1
f(a) = f(b) Þ = Þ a=b
2 2
Þ x = y3 + 1 ... (Which is f (x))
Case 3 : If a is odd and b is even
Hence f (x) and g (x) are converse to each other.
a -1 b
then f(a) = and f(b) =
2 2 Example 5.3.12 Let f : X ® Y and X and Y are set of real numbers. Find f–1 if
a -1 b 2x - 1
Now, f(a) = f(b) Þ = Þ
a b 1
- = (i) f(x) = x2 ; (ii) f(x) =
2 2 2 2 2 5
Solution : Let f : X ® Y and X, Y Í Â
Þ a–b = 1 \ a¹b
(i) f(x) = x2
In particular a = 7, b = 6
7 -1 6 Let f(x) = y
f(7) = =3 f(6) = = 3
2 2 x2 = y
f(7) = f(6) but 6 ¹ 7 Þ y = x2
Thus f is not one one function. Hence f is not bijective function. Þ y = ± x
Example 5.3.10 Let f (x) = ax2 + b and g(x) = cx2 + d, where a, b, c, d are constants. Therefore, given function is not one to one. Hence f–1 does not exist.
Determine for which values of constants gof (x) = fog (x).
(ii) Let f(x) = y
Solution : Given that, f(x) = ax2 + b and g(x) = cx2 + d
2x - 1
= y
Suppose fog (x) = gof (x) 5

f [g (x)] = g [f (x)] Þ 2x = 5y + 1
5y + 1
f [ cx2 + d] = g [ax2 + b] x = " yÎÂ
2
Þ a [ cx2 + d]2 + b = c [ax2 + b]2 + d
Function f is 1 – 1 and onto.
Þ a [c2 x4 + 2cdx2 + d2] + b = c (a2 x4 + 2abx2 + b2) + d
\ The inverse function of f is given as,
Þ ac2 x4 + 2acdx2 + ad2 + b = ca2 x4 + 2cabx2 + cb2 + d 1 + 5y
f–1 (x) =
Þ Coefficient of x4 = coefficient of x4 2

and Coefficient of x2 = coefficient of x2 Example 5.3.13 Let X = {a, b, c}. Define f : X ® X such that f = {(a, b) (b, a) (c, c)}.

Þ ac2 = ca2 Þ a=c Find (i) f –1 ; (ii) f 2 ; (iii) f 3 ; (iv) f 4


Solution : Given fucntion f is bijective function.
and 2acd = 2acb Þ b=d
(i) f–1 = {(b, a) (a, b) (c, c)}
Thus a = c and b = d
(ii) f2 = fof = {(a, a) (b, b) (c, c)}
Example 5.3.11 Show that the functions f(x) = x3 + 1 and g(x) = (x – 1)1/3 are converse to
each other. fof (a) = f[f (a)] = f(b) = a
Solution : We have, g(x) = (x - 1) 1 3 fof (b) = f[f (b)] = f (a) = b

Let y = g (x) = (x - 1) 1 3 fof (c) = f [f (c)] = f (c) = c

TECHNICAL PUBLICATIONS® - An up thrust for knowledge TECHNICAL PUBLICATIONS® - An up thrust for knowledge
Discrete Mathematics 5 - 15 Functions Discrete Mathematics 5 - 16 Functions

(iii) f3 (a) = f [f2 (a)] = f (a) = b It's pictorial representation is


f3 (b) = f [f2 (b)] = f (b) = a f g
X Y Z
f3 (c) = f [f2 (c)] = f (c) = c
1 p a
f2 = {(a, b), (b, a), (c, c)}
2
q b
(iv) f4 = f2 o f2 3

f4 (a) = f2 [f2 (a)] = f2 (a) = a


gof
f4 (b) = f2 [f2 (b)] = f2 (b) = b Fig. 5.3.7
f4 (c) = f2 [f2 (c)] = f2 (c) = c Example 5.3.16 Let R be the set of real numbers and f, g, h : Â ® Â such that
\ f4 = {(a, a) (b, b) (c, c)} 1
f(x) = x + 2 , g (x) = h (x) = 3. Compute
x 2 +1
Example 5.3.14 Let A = {1, 2, 3}, B = {1, 2, 3} and f 1 and f 2 are functions from A to B.
(i) f–1 g (x) ; (ii) hf (gf–1) [h f (x)]
f 1 = {(1, 2), (2, 3), (3, 1)}
Solution : (i) Given that f (x) = x + 2
f 2 = {(1, 2), (2, 1), (3, 3)}
Compute f 1 o f 2 and f 2 o f 1 SPPU : May-18, Marks 4 Let f(x) = y = x + 2 Þ x = y–2
Solution : We have f1 o f2 ( x) = f1 [ f2 ( x)] f–1 (y) = y – 2
é 1 ù 1
\ f1 [ f2 (1)] = f1 [ 2] = 3 f–1[g (x)] = f -1 ê ú = 2 -2
ë x 2 + 1û x +1
f1 [ f2 ( 2)] = f1 [1] = 2
f1 [ f2 ( 3)] = f1 [ 3] = 1 =
1 - 2x 2 - 2
=
(
- 2x 2 + 1 )
Now, f2 o f1 ( x) = f2 [f1 (x)] (x 2 + 1) x2 + 1

f2 [ f1 (1)] = f2 [ 2] = 1 (ii) We have f(x) = x + 2


f2 [ f1 ( 2)] = f2 [ 3] = 3 h f (x) = h [f (x)] = h [x + 2] = 3
f2 [ f1 ( 3)] = f2 [1] = 2 g f–1 (x) = g [f–1 (x)] = g [x – 2]

Example 5.3.15 Let X = {1, 2, 3} , Y = {p, q} and z = {a, b}


1 1
= =
Let f : X ® Y be {(1, p), (2, p), (3, q)} and g : Y ® Z be ( x - 2) 2 + 1 x 2 - 2x + 5

g = {(p, b) (q, b)}. Find gof and show it pictorically. 1


(g f–1) [h f (x)] = g f–1 (3) =
Solution : Given that, f = {(1, p), (2, p), (3, q)} 3 2 - 2(3) + 5
g = {(p, b) (q, b)} 1 1
= =
and gof (x) = g [ f (x) ] and gof : X ® Z 9-6+6 8
é 1ù é æ 1 öù
gof (1) = g [f (1)] = g (p) = b \ h f (g f–1) (h f (x)) = h f ê ú = h ê f ç ÷ ú
8
ë û ë è 8 øû
gof (2) = g [f (2)] = g (p) = b
é1 ù é 17 ù
= h ê + 2ú = h ê ú = 3
gof (3) = g [f (3)] = g (q) = b ë8 û ë 8û
gof = {(1, b) (2, b) (3, b)} i.e. h f (g f–1) (h f (x)) = 3

TECHNICAL PUBLICATIONS® - An up thrust for knowledge TECHNICAL PUBLICATIONS® - An up thrust for knowledge

Discrete Mathematics 5 - 17 Functions Discrete Mathematics 5 - 18 Functions

Example 5.3.17 Show that f, g : N ´ N ® N as f (x, y) = x + y. g(x, y) = xy are onto but not i.e. f(x) ³ 2 and for x < 0, f(x) = x < 0
one-one
\ Range set = f(Â) = {y Î Â/f(x) ³ 2 or f(x) < 0}
Solution : (i) Suppose f (x 1 , y 1 ) = f (x 2 , y 2 )
\ 1 Ï f (Â)
Þ x 1 + y 1 = x2 + y 2 Ì
Thus f (Â) ¹ Â
Þ x1 ¹ x2 and y1 ¹ y2
Hence  is an infinite set.
e.g. x1 = 5 , x2 = 2 , y1 = 4 , y2 = 7.
I) Properties of an infinite sets
\ f is not one to one.
1) If A is an infinite set the A ´ A, P (A) are infinite sets.
Now, g (x1, y1) = g (x2, y2)
2) If A and B are non empt sets and either A or B is an infinite set then A ´ B is an
Þ x 1 y 1 = x 2 y2 infinite set.
Þ x1 may or may not be equal to x2 3) If either A or B is an infinite set then A È B is an infinite set.

and y1 may or may not be equal to y2 4) If A Í B and A is an infinite set then B is also an infinite set.

\ g is not one to one. i.e. the superset of an infinite set is an infinite.

(ii) f(x, y) = x + y 5.4.2 Countable Sets


\ Every element of N can be written as the sum of two elements of N. Hence f is We know that the codinality of a set is the number of elements of that set. If set is
onto.
finite then, we can list elements as 1, 2, 3, …. Therefore every finite set is countable. As
Now, g (x, y) = xy |f| = 0, the null set is also countable. A question remains same for an infinite sets. Let
us define countable infinite sets.
Every element of can be written as the product of two elements of N.
\ g is onto. Definition :
An infinite set A is said to be countable if there exists a bijection f : N®A
5.4 Infinite Sets and Countability SPPU : May-18, 19
\ A = { f (1), f (2), f (3), …}

5.4.1 Infinite Set A countably infinite set is also known as a denumarable set.
i.e. If A is a denumerable set then we can least elements of A as a 1 , a 2 , a 3 , … a n …
A set A is said to be an infinite set if there exists an injective mapping (function)
or f (1), f (x) … f (n) … .
f : A ® A such that f(A) is a proper subset of A.
e.g. 1) f is countable
If no such injective function exists, then set is finite.
2) A = {1, 2, 3, 4, … 1000} is countable as |A| = 1000.
Examples
3) As f : N ® N defined as f (x) = n, \ N is countable
1) Let f : N®N such that f(n) = 2 n, " n Î N = Natural number set.
4) The set of integers is countable
There range set = f(N) = {Set of positive even natural numbers} C N n+ 1
As f : N ® Z such that f (n) = if n = 1, 3, 5, …
\ N is an infinite set. 2
ìx + 2 if x ³ 0 -n
2) Define f : Â ® Â such that f(x) = í = if n = 0, 2, 4, 6, …
2
î x if x < 0
is bijective mapping.
f is an injective mapping and for x ³ 0, f (x) = x + 2
TECHNICAL PUBLICATIONS® - An up thrust for knowledge TECHNICAL PUBLICATIONS® - An up thrust for knowledge
Discrete Mathematics 5 - 19 Functions Discrete Mathematics 5 - 20 Functions

5) The set of rational numbers is countable. 4) The set of irrational numbers is uncountable.
6) The set of real numbers is not countable
Proof : We know that  = Q È Q where Q = set of rational numbers, Q = set of
7) The set of complex numbers is not countable. irrational numbers.
8) The set of real numbers in [a, b], a < b is not countable. Suppose Q is countable Þ Q È Q = Â is countable which is contradiction.
9) The countable union of countable sets is countable. \ Q is not countable i.e. uncountable.

Properties of countable sets : 5) The set of real numbers in (0, 1) is not countable. Assume that the set is
countable. \ A = {x 1 , x 2 , x 3 , L x n , L}.
1) A subset of a countable set is countable.
2) Let A and B be countable sets then A È B is countable. Proof : Any real number in (0, 1) can be written in a unique decimal without an
infinite string of 9's at the end. i.e. 0.3459999 will be represented as 0.345000. Let the
Þ Define f : A È B ® N as f (a i ) = 2 i – 1 infinite sequence be given by,
and f (b i ) = 2 i 1 ® x 1 = 0 × a 11 a 12 a 13 …
f is bijective \ A È B is countable 2 ® x 2 = 0 × a 21 a 22 a 23 …
3) Prove that the set of rational numbers is countably infinite. 3 ® x 3 = 0 × a 31 a 32 a 33 …
SPPU : May-18, 19, Marks 4 M M
Proof : We know that the countable union of countable sets is countable. Therefore it is n ® x n = 0.a n1 a n2 a n3 …
sufficient to prove that the set of rational numbers in [0, 1] is countable. M M
We have to prove that $ atleast one function f. Construct a new number y = 0 × b1 b 2 b 3 …
f : [0, 1] ® N such that f is injective.
Where b i = 0 if a ii ¹ 0
We arrange the rational numbers of the interval according to increasing denominators
b i = 1 if a ii = 0
as
1 1 2 1 3 1 2 3 4 Hence b1 ¹ a 11 , b 2 ¹ a 22 , b 3 ¹ a 33 … b n ¹ a nn
0, 1, , , , , , , , , ,L
2 3 3 4 4 5 5 5 5 \ b i ¹ a ii , " i
then the one to one correspondence is as follows
\ y ¹ x1 , y ¹ x i " i
0«1 Hence y is not in the list of numbers {x 1 , x 2 , L x n L} Thus y Î(0, 1) and it is
1«2 different from elements in {x 1 , x 2 , L x n L} which is contradiction that A is countable.
1 Hence A is not countable.
«3
2
Thus the set of real numbers is not countable.
1
«4
3 5.5 Pigeon Hole Principle SPPU : Dec.-09, 11, 12
2
«5 I) This principle states that if there are n + 1 pigeons and only n pigeon holes then
3
1 two pigeons will share the same whole.
« 6 and so on
4 This principle is stated by using the analogy of the bijective mapping i.e. If A and B
Hence set of rationals in [0, 1] is countable. Thus the set of rational numbers is are any two sets such that |A| > |B| then there does not exist bijective mapping from A
countable and as the set is infinite, it is countably infinite. to B.

TECHNICAL PUBLICATIONS® - An up thrust for knowledge TECHNICAL PUBLICATIONS® - An up thrust for knowledge

Discrete Mathematics 5 - 21 Functions Discrete Mathematics 5 - 22 Functions

Examples Example 5.5.4 Write generalized pigeonhole principle. Use any form of pigeonhole principle to
solve the given problem.
Example 5.5.1 If 11 shoes are selected from 10 pairs of shoes then there must be a pair and
i) Assume that there are 3 mens and 5 womens in a party show that if these people are
matched shoes among the selection.
lined up in a row at least two women will be next to each other.
Solution : In the pigeonhole principle, 11 shoes are pigeons and the 10 pairs are the
pigeon holes. ii) Find the minimum number of students in the class to be sure that three of them are
born in the same month. SPPU : Dec.-11, Marks 4
Example 5.5.2 Show that if seven numbers from 1 to 12 are chosen then two of them will add
Solution : Please refer section 5.5 (II) for definition.
upto 13.
i) By using analogy of pigeon hole principle, we get
Solution : We have A = {1, 2, 3, 4, 5, … 12}
3 men = pigeonholes and 5 women = pigeon
We form the six different sets each containing 2 numbers that add uptot 13.
A1 = {1, 12}, A 2 = {2, 11}, A 3 = {3, 10}, A 4 = {4, 9}, A 5 = {5, 8}, A 6 = {6, 7} Pigeons are more than pigeon holes.

Each of the seven numbers chosen must belong to one of these sets. As there are only \ At least two pigeons share the same pigeon hole i.e. at least two women in a row
six sets, by pigeonhole principle two of the chosen numbers must belong to the same set will be next to each other.
and their sum is 13. ii) Let h = Number of pigeons = Number of students
II) The extended pigeon hole principle n = Number of pigeon holes = Number of months = 12
If n pigeons are assigned to m pigeon holes, then one of the pigeon holes must be
Given that three students in the class are born in the same month.
é n - 1ù
occupied by at least ê + 1 pigeons. It is also known as generalized pigeon hole
ë m ûú \
é n - 1ù
n - 1 êë m úû + 1 = 3
é ù é 9ù é 16ù
principle. Here ê is the integer division of n – 1 by m. e.g. ê ú = 4, ê ú = 3,
ë m ûú ë 2û ë 5û n- 1
Þ = 3–1=2
é 8ù 12
= 2.
ëê 3ûú
n = 2 ´ 12 + 1 = 25
Examples Therefore there are 25 minimum number of students in the class.
Example 5.5.3 Show that 7 colours are used to paint 50 bicycles, then at least 8 bicycles will
5.6 Discrete Numeric Functions
be of same colour. SPPU : Dec.-09, 12, Marks 4
é n - 1ù A function whose domain is a set of natural numbers including zero and whose
Solution : By the extended pigeonhole principle, at least ê + 1 pigeons will occupy
ë m ûú range set is the set of real numbers, is called a discrete numeric function or numeric
one piegeonhole. function. It is also known as a sequence. If f : N È {0} ® Â is a discrete numeric
function then f(0, f(1), f(2), f(3), … denote the value of function at 0, 1, 2, 3, …
Here n = 50, m = 7 and m < n then
The numeric function f is written as
é 50 - 1ù
ê 7 ú +1 = 7+1=8 f = {f 0 , f 1 , f 2 , L }
ë û

Thus 8 bicycles will be of the same colour. Hereafter, to denote numeric function, we use
< a r > = {a 0 , a 1 , a 2 , a 3 L a r , L }
e.g. 1) a r = 2 k if 0 £ k £ 3
= k if k > 3

TECHNICAL PUBLICATIONS® - An up thrust for knowledge TECHNICAL PUBLICATIONS® - An up thrust for knowledge
Discrete Mathematics 5 - 23 Functions Discrete Mathematics 5 - 24 Functions

This discrete function can be written as Example 2 :

a = {1, 2, 4, 8, 4, 5, 6, 7, 8, …} ì 1 if 0 £ r £ 2 ì2 r + 1 , 0 £ r £ 1
If ar = í and br = í
î3 r if r³ 3 î r- 5 , r³ 2
ì100 + r, 0 £ r £ 5
2) ak = í 2
î r r³ 6
then a+b = c
\ This discrete function can be written as ì2r + 1 + 2 , 0 £ r £ 1
ï
a = {100, 101, 102, 103, 104 , 105, 6 2 , 7 2 , 8 2 , 9 2 , L } \ c r = a r + b r = í 1 + ( - 3) , r= 2
ï3 r + r - 5 , r³ 3
î
5.6.1 Basic Operations on Numeric Functions
ì2 r + 1 + 2 , 0 £ r £ 1
Let a and b be two numeric functions. Then ï
= í -2 , r= 2
i) Addition : c = a + b is also a numeric function defined as ï 4 r- 5 , r³ 3
î
c r = a r + br
ì 1 (2 r + 1) , 0 £ r £ 1
i.e. c 0 = a 0 + b 0 , c 1 = a 1 + b1 , c 2 = a 2 + b 2 and so on. ï
and d = ab and d r = a r b r = í 1 ( - 3) , r= 2
ï3 r ( r - 5) , r³ 3
ii) Multiplication : d = ab is also a numeric function defined as d r = a r b r , î
" Î N È {0}. ì 2r + 1 , 0£ r£ 1
iii) Scaling : Let k be any real number then b = ak is a numeric function and k is ï
dr = í -3 , r= 2
called as scalling factor. ï3 r 2 - 15 r , r³ 3
î
iv) Linearity : Let p and q be any real numbers, then x = pq + qb is a numeric
function where Example 3 :
x r = p a r + q br If a r = 3r , r ³ 0

v) The convolution of two numeric functions : Let a and b be two numeric br = 5r , r ³ 0


functions. The convolution of two functions a and b is denoted by c = a * b and r
defined as then c = a * b Þ cr = å a n br - n = a 0 br + a 1 br - 1 + a 2 br - 2 +L + a r b0
n= 0
r
c r = a 0 br + a1 br -1 + a 2 br - 2 + … + a r b0 = åan br - n
5.6.2 Finite Differences of a Numeric Functions
n= 0

Example 1 : I) Shift of a numeric function

1 ì 1 1 1 ü The shift of a numeric function of a r is denoted by E [a r ] and defined as


If ar = , r ³ 0 then a = í1, , , , Ký
3r î 3 32 33 þ E [a r ] = a r +1

and
ì 5 5 5
5a = í5, , ,
ü
, Lý E - 1 [a r ] = a r - 1
î 3 32 33 þ
E n [a r ] = a r +n , E - n [a r ] = a r - n
1
\ 5 a r = 5× Note : It is also denoted by s n [a r ] = E n [a r ]
3r

TECHNICAL PUBLICATIONS® - An up thrust for knowledge TECHNICAL PUBLICATIONS® - An up thrust for knowledge

Discrete Mathematics 5 - 25 Functions Discrete Mathematics 5 - 26 Functions

Example 1 : ì0 ; 0 £ r £ 3
ï
D ar = í4 ; r= 4
If a r = 5 r , r ³ 0 then E [a r ] = a r + 1 = 5r + 1
ï1 ; r³ 5
î
E 2 [a r ] = a r + 2 = 5 r + 2 , E - n [a r ] = a r - n = 5 r - n
III) Backward difference of a numeric function
Example 2 :
The backward difference of a numeric function a r is denoted by Ñ a r and defined as
ìr 3 - 2r + 5 ; 0 £ r £ 6
If ar = í Ñ a r = a r – a r -1 ; r ³ 1
î 3 ; r³ 7

ì 0 , 0£ r< 2 Examples
ï
then E 3 [a r ] = í(r + 3) 3 - 2 (r + 3) + 5 , 3 £ r £ 9 Example 1 :
ï 3 , r ³ 10
î Let a r = 6r ; r ³ 0
ì(r - 2) 3 - 2(r - 2) + 5 ; 0 £ r £ 4
and E - 2 [a r = í a r - 1 = 6 r - 1 ; r – 1 ³ 0 i.e. r ³ 1
î 3 ; r³ 5
\ Ñ a r = a r – a r - 1 = 6r - 6r - 1 ; r ³ 1
II) Forward difference of a numeric function = (6 – 1) 6 r - 1 ; r ³ 1
The forward difference of a numeric function a r is denoted by D a r and defined as Ñ a r = 5 × 6r - 1 ; r ³ 1
D a r = a r +1 – a r ; r ³ 0
Example 2 :
Example 1 : ì1 ; 0£ r£ 2
If ar = í r
If a r = 3 r ; r ³ 0 then î3 ; r³ 3

D ar = ar+ – a r = 3r + 1
- 3r ì 1 ; 0£ r- 1£ 2
1 ar- 1 = í r- 1
î3 ; r- 1³ 3
= (3 – 1) 3 r = 2.3 r
ì 1 ; 1£ r£ 3
Example 2 : = í r- 1
î3 ; r ³4
ì 3 ; 0£ r£ 4
If ar = í ì 0 ; r= 0
îr + 2 ; r³ 5 ï 1 ; r= 1
ïï
then D ar = ar+ 1 - ar Ñ ar = ar – ar- 1 =í 0 ; r= 2
; 0£ r+ 1£ 4 ï 26 ; r= 3
ì 3
ï
\ ar+ = í
1 ïî2 ´ 3 r - 1 ; r³ 4
î(r + 1) + 2 ; r+ 1³ 5

ì 3 ; 0£ r£ 3 Example 5.6.1 Determine a * b for the following numeric functions


ar+ 1 = í
îr + 3 ; r³ 4 ì1 ; 0 £ r £ 2 ìr + 1 ; 0 £ r £ 2
ar = í and b r = í
î0 ; r³3 î 0 ; r³3
ì 0 ; 0£ r£ 3
ï Solution : By the definition of a r , the numeric function of a is given by
\ D ar = í - 3 + (4 + 3) ; r= 4
ï(r + 3) - (r + 2) ; r³ 5 a = {1, 1, 1, 0, 0, 0, …} = {a 0 , a 1 , a 2 , a 3 , …}
î
b = {1, 2, 3, 0, 0, 0 …} = {b 0 , b1 , b 2 , b 3 , …}

TECHNICAL PUBLICATIONS® - An up thrust for knowledge TECHNICAL PUBLICATIONS® - An up thrust for knowledge
Discrete Mathematics 5 - 27 Functions Discrete Mathematics 5 - 28 Functions

Notes
The convolution of a and b is a numeric function c such that
r
c = a*b where cr = åan br - n
n= 0

\ c r = a 0 br + a1 br -1 + a 2 br - 2 + … + a r b0
\ c 0 = a 0 b0 = 1 ´ 1 = 1
c 1 = a 0 b1 + a 1 b 0 = 1.2 + 1.1 = 3
c 2 = a 0 b 2 + a 1 b1 + a 2 b 0 = 1.3 + 1.2 + 1.1 = 6
c 3 = a 0 b 3 + a 1 b 2 + a 2 b1 + a 3 b 0
= 1.0 + 1.3 + 1.2 + 1.0 = 5
c 4 = a 0 b 4 + a 1 b 3 + a 2 b 2 + a 3 b1 + a 4 b 0
c 4 = 0 + 0 + 1.3 + 0 + 0 = 3
c 5 = 0, c 6 = 0, c r = 0 ; r ³ 5

Thus the numeric sequence of c is given by


c = {1, 3, 6, 5, 3, 0, 0, 0, …}
ì1 ; r= 0
ï3 ; r= 1
ï
ï6 ; r= 2
\ cr = í
ï5 ; r= 3
ï3 ; r = 4
ï
î0 ; r ³ 5
qqq

TECHNICAL PUBLICATIONS® - An up thrust for knowledge TECHNICAL PUBLICATIONS® - An up thrust for knowledge

SUBJECT CODE : 210241


Discrete Mathematics
As per Revised Syllabus of (For END SEM Exam - 70 Marks)
Savitribai Phule Pune University
Choice Based Credit System (CBCS) Subject Code : 210241
S.E. (Computer) Semester - I
S.E. (Computer) Semester - I

Discrete Mathematics
ã Copyright with Authors
(For END SEM Exam - 70 Marks) All publishing rights (printed and ebook version) reserved with Technical Publications. No part of this book
should be reproduced in any form, Electronic, Mechanical, Photocopy or any information storage and
Dr. H. R. Bhapkar retrieval system without prior permission in writing, from Technical Publications, Pune.

M.Sc. SET, Ph.D. (Mathematics)


MIT Art, Design and Technology University’s
MIT School of Engineering, Lonikalbhor, Pune
Email : drhrbhapkar@gmail.com Published by :
® ®
Amit Residency, Office No.1, 412, Shaniwar Peth,
Mobile : 9011227141 TECHNICAL Pune - 411030, M.S. INDIA, Ph.: +91-020-24495496/97
PUBLICATIONS
SINCE 1993 An Up-Thrust for Knowledge Email : sales@technicalpublications.org Website : www.technicalpublications.org
Dr. Rajesh Nandkumar Phursule
Ph.D. (Computer Science and Engineering)
Associate Professor,
Pimpri Chinchwad College of Engineering Printer :
Nigdi, Pune. Yogiraj Printers & Binders
Sr.No. 10/1A,
Ghule Industrial Estate, Nanded Village Road,
Tal. - Haveli, Dist. - Pune - 411041.

ISBN 978-93-332-2155-9
® ®
TECHNICAL
PUBLICATIONS
SINCE 1993 An Up-Thrust for Knowledge

9 789333 221559 SPPU 19

(i) 9789333221559 [1] (ii)



S A  B

E1 n1 E2
n2 E1 E2 n1  n 2
E1 E2

 E1
E2
 E1 E2 E1 E2


E1
 E1

E2

TM

TECHNICAL PUBLICATIONS - An up thrust for knowledge

E2

E2 E1 E2  
3
P2

r th
A B n
Pr n  (n  1)  (n  2) (n  (r  1))
n  (n  1) (n  2) (n  r  1)  (n  r) !
A B
(n  r) !

n!
n Pr  
E1 n1 (n  r) !
E2 n2 n1  n 2
E1 E2 n n! n!
Pn
(n  n) ! 0!

n n! n (n  1) !
P1
(n  1) ! (n  1) !

n n! n (n  1) [(n  2) !]
P2
(n  2) ! (n  2) !
 n P3

       28

 A
6 6! 6! 6 5 4 !
P2
(6  2) ! 4! 4!
6 6! 6 5 4 3 !
P3
(6  3) ! 3!


6 6! 6!
n P4
Pr (6  2) ! 4!
6! 6!   
6
P5     
(6  5) ! 1!
6 6! 6!
P6
(6  6) ! 0!

= (5 x 4 x 3) x 2

9 9! 9 8 7 6 5 !
P4
(9  4) ! 5!
TH H Ten Unit place

= 120 ways

  

TH UnitPlace
1 1
3 4 3 2

  

 
6
P4

6!

2!


M1 M2

M1 M1 M1 M1 M1 M1
or or or or or or
M2 M2 M2 M2 M2 M2

6
P2
  
 
  6
P2 5!  6  5


M1 M2 M3

M3
M1 M2
M3 
   
M3
M3

 

 

M1 M2 M1
M2

M1 M2   
 

(n  p)
Pr
11 !

(n  p) 4! 4! 2! 1!
Pr  p  r Pp

n
Pr 
A B

a1 a 2 ar b1 b 2 b 3 br bn

f
A B
a1 f(a1) has n choices b1
13 !
a2 f(a2) has n – 1 choices b2 
a3 f(a3) has n – 2 choices
3! 2! 2! 4! 1! 1!
br

ar f(ar) has n – (r – 1) choices bn

n
Pr 

r1
r2
r3
7!

3!  2!  1!  1!
rk r1  r 2  rk

r1 r 2 rk
n!
r1 ! r 2 ! r 3 ! rk !
nr

n!
 
n

   44

3 53 

    65 E
S
E

S S
6 6
5
E E

  S S
 E
       28 E
S

 24

nC
r

n!
nC 
r
r ! (n  r) !

 16
C5
n Pr
n!
nC 
r
r!(n  r) ! r!
16 C 16! 16 15  14  13  12  11!
nC n! 5
5!  (16  5) ! 5  4  3  2  1  11 !
n
n! (n  n) !

n n! n!
C0
0! (n  0) ! n!

n n! n n! n (n  1)
C1 C2
1! (n  1) ! 2! (n  2) ! 2
n (n  1) (n  2) n (n  1) (n  2) (n  3)  30 C
10
nC nC
2 4
3! 4! 20
C15
nC 25 25
n 2 Cn  2 C 2n  1 5
C5

n! 30! 20! 30!


n
C n 2  30 c  20 c 15  5 c 5  1
(n  2) ! (n  (n  2)) ! 10
10! (20!) 15!  5! 10! 15! 5!
n (n  1) (n  2) !

(n  2) ! (2 !)


5C
n 2  n  20 1
20
 C15
25
C6

nC nC
5
C1  20 C15  25 C 6
r n 2

25 25
C n 2 C n 2



10 10!
C5
5! 5!
26
C1
10  9  8  7  6
5 4 3 2 1
25
C1

9C
4
10
C1
9! 9 87 5
 9
C4
4! 5! 4  3 2 9
C1
8C
1

9
C5
   
9!
 9
C5
4! 5!
9C
1

9
C1
8
C1

7C
3    
4
C2
7
C3  4 C2
7C 4C
4 1

 7c
4  4 c1 
7
C5 9! 9 8 7 6
1 9 C4
4! 5! 4 3 2
 7
C5

12 C
3

5
C3
 4C
3  36 C 2
12 C  5C 12!

5! 4! 36! 36  35
3 3
3!  9! 3! 2!  4
3!  1! 2! 34! 2
12  11  10 5  4

3 2 2 40
C5
36
C5

7
8 6
40
C 5  36 C 5
9 5

12 12 ! 12  11 10 4
C2
2! 10! 2 1
11 3

12 2 (n  r  1) (n  r 1)
1 Cr Cn  1

x1  x 2   xn

8
C2

  a1   a 2  an


40
C5 9! 9  8  7  6  5!
40 ! 40  39  38  37  36 4! 5! 4  3  2  1  5!
40 C
5
5!  (40  5) ! 5 4 3 2 1
  a1   a 2   a 3   a 4

36
C5
36 ! 36  35  34  33  32 
 36
C5
5!  (31) ! 5 4 3 2 1 6! 6  5  4  3!
4
C3 3! 3! 3  2  3!
36 C
2

x1  x 2  x 3  x 4 a1  a 2  a 3   a n a 1  b1
a 2  b2 a 3  b3 a n  bn b1 b2 b3 bn
r  b1  b 2  b 3   bn

23! 23  22  21
20!  3! 3 2

10  9  8  7  6!
4  3  2  1  6!

x1 x2 x3 x4 x5
xi 
xi yi  2 yi 
x1  x 2  x 3  x 4  x 5
yi  2 y2  2 y3  2 y4  2 y5  2
y1  y 2  y 3  y 4  y 5 a1 a 2 a 3 an

 am am 1
y1  y 2  y 3  y 4  y 5
 ak ak am

10! 10  9  8  7  6!

6!  4! 4  3  2  1  6!
a1 a 2 a 3 am 1 
x1 x2 x3 x4 x5
x1  x2  x3  x4  x5 
x1 y1  2 x 2 y2  3 x3 y3  4 x4 y4  2 x5 y5  0

 x1  x 2  x 3  x 4  x 5
 y1  2  y 2  3  y 3  4  y 4  2  y 5 (a 1 , a 2 , , an )

y1  y 2  y 3  y 4  y 5

23  22  21  20 ai ak
4 3 2 1
ak ai ai ai
ak 




ar as 



i th

a1 a 2 a 3 ak a1 a2 a3
am am 1 ak
 ak
 ak 1

 ak ak am ai

am 


aj aj a j 1 aj
1254 3 6
 ai
  a1

125 4 63 a 1, a 2 , a 3 , a r

 a1 a 2 a3 ar


ai

ai a2  1

aj ai  j i
n
(x  y) n  (n) x n  r y r
r
r 0
a1 a 2 a 3 a 4
 n  n  n  n 1 y n  x n 2 y 2   n 
a1 a2 a3 a4 a4 a3 a2 a1
 0  x  1  x 2   n  1  xy n  1   n  y n
        n

 n  x n y 0   n  1 x n  1 y   n  2  x n  2 y 2  n n


a4 n n
        1  xy n  1   0  x 0 y n
a4 n        
n  n  r n r
  rx y
  n cr n cn  r
r 0  
a3
a3
 1  1
 ( x  y) 1 x1  y 1  0 x 1  y
a3    
   
 5 4
  ( x  y) 2 x 2  2xy  y 2  2  x 2   2  xy   2  y 2
0  1 2
a2 a2      
 3 6 5 4 ( x  y) 3 x3  3x 2 y  3xy 2 y3
   
 3  x 3   3  x 2 y   3  xy 2   3  y 3
a1 a1 0  1 2  3
       
 2 6 5
    3  x 3   3  x 2 y   3  xy 2   3  y 3
  2   0
 3    1  

 ( x  y) 4  4  x 4   4  x 3 y   4  x 2 y 2   4  xy 3   4  x 0 y 4
0  1 2  3 4
         
x 4  4 x 3 y  6x 2 y 2  4 xy 3  1y 4

 x 13 y 7 ( x  y) 20

xn r yr (x  y) n
n
nc
r  
r


  20 ! 20  19  18  17  16  15  14  13!
 20 
 7  7 !  13 ! 7  6  5  4  2  13!

(x  y) n x 12 y 13 ( x  y) 25
 25   (25 !) 200
   
 12  12! 13! (2x  3y) 200   200  (2x) ( 3y)
 r 
r n r
r 0
x y 8 12
( 2x  3y) 20

x 12 y 8  x 101 y 99

 200  2101 ( 3) 99


 
20  101 
(2x  3y) 20   20r  (2x) 20  r (  3y) r
r  0
  200  2101 (3) 99
 101 

 x 8 y 12 1
x 10 ( x  ) 100
x
 20  (2) 8 ( 3) 12 (2) 8 ( 3) 12
 12 
 
100 100  r
1 100  100  x r  1 
x 99 ( x  1) 101 (x 
x
)     
 r  x
r 0
 101 
  100
 99   100  x 2r  100
 
 r 

r  10
x 98 ( x  1) 101
 

 x 10  100 
101  
(x  1) 101
  101  x r (1) n  r  55 
r 0  r 
1
xk ( x 2  ) 100
 x
 101  ( 1) 3 x51
  
 98 

x 99 ( 2  x) 19 1 100
 100  (x 2 ) r (  1 ) 100  r
(x 2  ) 100
x  
 r 

x
r 0

100 100  r
19  100  x 2r   1 
(2  x) 19   19  (2) r (  x) 19  r
   
 r 
 
 x

r 0  r  r 0
100
 100  (  1) 100  r (x) 3r  100
 
 r 

   10 r 0
 19  2 ( 1)  92378  210
9
 10  1

3
x 101 y 99 ( 2x  3y) 200

 100  1 
1 200   100  (k  100)
 (x 2  ) 100
x   1 (k  100  (  1) 3 (x) k
k   100 3  n n
  o 1
   
1
xk (x 2  ) 100
x
   
   n2   nn 
 100  100 
1
(k  100)    
 1 (k  100  (  1) 3

3 
 
1     n   n 
x 20 x 21 ( x 2  ) 100 2n  n0    n1    2  
x       n
n
x 20 n
2 n
  
r
r 0
n
  100  ( 1) 100  40
 
 100 
   (  1 )r  nr 
 40   40  r 0

x 21
n n
   
(0) n (1  ( 1)) n   
r n r
 nr  ( 1) 1   nr  ( 1)
 
r
r 0 r 0

 n 1 
  r  n  n r  1 
r  

79
  
3

n
n
 r 
 
2n
r 0
n
 
r

(x  y) n  n  x n  n  x n 1 y n  x n 2 y 2    n  xº y n r  n 
0 1 2
      n r

 n  n  n   n 1
(1  1) n 2n 0 1 2   n   r 1 
      n  
n
n  n 1
2n  r
 
 n r  1 
 
r 0
n  n 1
2n   n r  1 
r  
n
 m  n
 2 r (n)
r
3n  r k  k 
  
r 0


n n
 n  1n  r 2 r 2 r  n 
(1  2) n  r  r
  n
r 0   r 0 r (m  n)   rm
 k   k 
n
n
r k 0  
 2r  
r
3n
r 0 n 2
( 2n)   nk 
 n k  0

 n  1  n  n
 r  r 1  r
      2
 2n 
n
 n  n
n
n   n    n 
 n 
 
  n k  k 
  
 k
 
  n k   k 
   
k 0 k 0 

 n  1
 
 r 

 

 n 
 
 r 1

n
r
 
 n  1  n  n
 r   r 1  r 
     
   
r  
( m  n)   r m k   nk 
r k  0 

A B
 A B A  B
  m n 
 r 
 

(n)
k
 
 rm 
  k

    
 


 



 
n

 



 

nC
2

20 ! 30 !
20 C
3  30 C 4 
3 ! 17 ! 4 ! 26 !

20  19  18 30  29  28  27

3 2 4  3 21
14
C1
8C
1
8 4
14 7
14 14 ! 14  13
C2
2! 12! 2  
 
 
6C
1
8
C1
8 6 48
91 91

35555 354

 4  3  2  24

13 C 13  12  11  10 !
10
3 ! 10 !

13 13  12  11 !
C 11
2 ! 11 !

11 !

3! 3! 2! 2! 1!

7 765
C3
3!

6 65 4!
C2
2 4!

26 C
1
26 C
1
7
C4  6C1

10  10  10 – 1
7 C5  6 C 0

26  26  999
1  1  999

10 10  9  8  7  6 !
14 14  13  12  11 ! P4
4 ! 6 !
C 11
11 ! 3 !
1 2 3 4 5 6 7 8
1

6 2

8
C1
6 2 9!

 
    
  
3
3!
 3
( 2)
0
( 3  ) ! !

3! 3 0 3! 3 1  2 1
 20 
( 3  0) ! 0 ! ( 3  1) ! 1 !
6 6
)6    
   

3! 3 2
 22 
3! 3 3
2 3
      ( 3  2) ! 2 ! ( 3  3) ! 3 !

6   3   2 2    22   0 2 3

  
6 6 65 4  3! 65 4
 
  ( 6  3) ! 3 ! 3!  3! 3 2

10 P 10 ! 10 ! 10  9  8  7  6 !
4 6C 6! 65
(10 – 4) ! 6! 6! 4
4 !  2! 2

5 5!
C4
4!
 

n n
(x  y) n    x n– r y r
r= 0 r
n n
(a  b) n   r  a n– r b r
r  0 
8 th 4
4!
 (3x) 4– r (– 4) r
r 0
(4 – r)!r!
8 th
 13  13–7 7 4! 4!
  x y 1716 x 6 y 7 (3x) 4– 0  (– 4) 0  (3x) 4– 1  (– 4)
7  (4 – 0)!0! (4 – 1)!1!

4! 4! 4!
 (3x) 4– 2  (– 4) 2  (3x) 4– 3  (– 4) 3  (3x) 4– 4 (– 4) 4
(4 – 2)!2! (4 – 3)!3! (4 – 4)!4!

1  (3x) 4 (– 4) 0  4  (3x) 3  (– 4)  6  (3x) 2  (– 4) 2 + 4(3x) (– 4) 3  3  (3x) 0  (– 4) 4

81 x 4 – 432 x 3 + 864 x 2 – 768 x+ 256

6 6! 65
C2
2!  4 ! 2

5C 5! 5 4
2
2!  3! 2


TM

TECHNICAL PUBLICATIONS - An up thrust for knowledge

4 3 1 2

5 3
e1 e2
1 2 e1 e2 e1 e2
6 4

V(G1 ) V(G 2 )
E (G1 ) E (G 2 )

6 5 4
d

b
a c
1 2 3

V (G 3 ) V (G 2 )
E (G 3 ) E (G 4 )

e d V(G 5 )
E (G 5 )

b a b c
c
G1 G2 G3
a

e3 e2 V (G 6 )
G2
E (G 6 ) e1 e 2 e 3 G3
a e1 b

Nn

v3

v1 v1 v2 v1 v2 v1 v2 v3 v4
N1 N2 N3 N4
G2

c

 

a b

c
5 8 v 1, v 2 , v 3 , vn di vi
d 1, d 2 , d 3 , dn
a 3 b

v6 v5
v4
G  (v) d  (v) d (v 1 ) d (v 2 ) d (v 3 ) d (v 5 ) v3
d (v 6 ) d (v 4 )
v1 v2
G  (v) d  (v) 

d c b c
e

a b a d
f
 d (v)
v V
G1 G2

   d (v)

G1
G1

G1 V (G 1 )
G1
G1 

 d (x) v4 v3 v4
x V
v1 v3
G1 v5 v6

G1 v2 v1 v2

d G (u) d G (u)  1 G1 G2
1
d G 1 (v) d G (v)  1
v v2 v v 4v5 v 6
v1 1 3
v1 v2 v 3v4 0 1 1 0 0 0
 d (x)  d G 1 (u)  d G 1 (v)
v2 
v 1 0 1 0 1 1 1 1 0 0 0
x V v3  
x  u, v A (G1 ) v 2 1 0 1 1 A (G 2 ) 1 1 0 1 0 0
  v4  
v 3 0 1 0 1
 d (x)  d G 1 (u)  1 + d G (v) – 1
0 0 1 0 1 0
  v5 
x V v 4 1 1 1 0 4  4 0 0 0 1 0 0
x  u, v v6  
0 0 0 0 0 0 6  6
  d(x)
x V   a ij n  n
a ij vi vj

 d (v)
v V v1 v2 v 3 v 4 v5
v 1 1 1 0 0 0
v 2 1 0 2 0 1 v4
  v5
v 3 0 2 0 1 0
 
v 4 0 0 1 0 2 v3
v2 v1
v 5 0 1 0 2 1 

[x ij ] n  m
[a ij ] n  n
x ij vi
a ij vi vj
vi vj 

d (v i )
e1 e 2 e 3 e 4 e 5 e 6 e 7 v1 e1 v2
v 1 1 0 0 0 0 1 1 e7
v 2 1 1 1 0 0 0 0
e2
e6 v5 e3
  e5
v 3 0 0 1 1 1 0 1
  v4 v3 [x ij ] n  m
v 4 0 0 0 1 1 1 0
e4
v 5 0 1 0 0 0 0 0 5 7 x ij ej vi
ej vi

e1 e2 e3 e4 e5 e6
v 1 – 1 0 0 0 1 1  v6
v2  1 1 0 0 0 0 e6
v5 e4 v4
  e5 e3
v3  0 –1 1 –1 0 0 v1 v3
  e1 e2
v4  0 0 –1 0 0 0
v2
e1 e2 e 3 v5  0 0 0 1 0 – 1
v1 v2 v3  
v 1 1 0 1  v6  0 0 0 0 – 1 0 6 6
e1 e2
v 2 1 1 0 e3
 
v 3 0 1 0 3  3

[a ij ] n n  d (v i )
vi  V
a ij v i to v j

  d (v i )  d(x)   d(y)
vi  V x V y V
even degree odd degree

v1 v2 v 3 v 4 v5 v 6   d (v i ) 2q   d (x)
vi  V x V
v 1 0 1 0 0 0 0 v1 v2 odd degree even degree
v 2 0 0 1 0 0 0
v3
  
v 3 0 1 0 0 0 0
 
v 4 0 0 0 0 0 0 v4
v6 v5
v 5 0 1 0 1 0 1
  n(n  1)
v 6 1 0 0 0 0 0 6  6
2

 d (v) 
v V
G1 G2 G3
   V 
 

n (n  1)
2
n (n  1)
2

 d (v)
v  V(G)

n
 d (v i ) 
i 1

 d (v)


 d (v)
v G
d (v 1 )  d (v 2 )  d (v 3 )  d (v 4 )  d (v 5 )  l (v 6 )

1 2 3 4 5
 n (n  1) 
 
4 5
2 2 

d c
3 2

1 2 3
a b
1
G1 G2 G3

G1 G 2 G 3 End A End B
a b c d M
T MS MT
a 0 1 1 1 -
S A B {S} A B {T}
A (G1 ) b 1 2 1 1 C {M S C}
  {M T C} M MS
c 1 1 0 1
  A B
d 1 1 1 1 4  4

1 0 1 1 0 0
2 1 0 0 1 0
 
A (G 2 ) 3 1 0 0 1 0
  MC
4 0 1 1 0 1 MS
A B {T C}
5 0 0 0 1 0 5  5 {M S} A B {M T S C}
M
1 2 3
1 1 0 0
A (G 3 )
2 0 1 0
 
3 0 0 1  3  3

n (n  1)
2
m
e 
 
 
n (n  1)
mC
e  
2

n (n  1) 4 3
m m m m
 0  1  2    m  2 2
     
       
2m 

  d (v)
v x
K1 K2 K3 K4 K5

v1 v2 v1  v 2
v1  v 2  v1 v2
v1

1
1 2 3 1 2 3
1) 2) 3) b a

i) ii)
3 2
v a b c
c
N3 : 1 - regular graph 1 - regular graph v1 = {v} v1 = {a, b, c} v1 = {1, 2, 3}
v2 = {1, 2, 3} v2 = {1, 2, 3} v2 = {a, b, c}

iii) iv)

v1  v 2 v1  v 2 
Vi v2

2 - regular graphs 3 - regular graph


v1 v2 G (v 1  v 2 , E)
K m, n

1) 2) 3) 4)
Kn

K1,1 K1,2 K2,2 K2,3


Kn

Kn 5)
n (n  1)
Kn
2
K4,2 K4,1
K1, n

G (v 1  v 2 , E)
v1  v 2
E v1  v 2
E

K 10 K5 , 3 K 5 ,7
10  9
K10
2
6 1 K 5, 3 
K 5, 7 
2
4

5 3

G1 (V1 , E 1 ) G 2 (V2 , E 2 ) G1 G2

 V1  V2
 E1  E 2 G1
  G2
  G1 G2
G1  G 2
G1 (V1 , E 1 ) G 2 (V2 , E 2 )

V1 V2 G1 G2
K1, 3 K2, 3 K4, 2
E1 E2 G1 G2
G1 G2
K1, 3 K 2, 3 K 4, 2 G1 G2
K1, 3 K 2, 3 K 4, 2 G1 G2

G1 G2
G1 G2

G1 G2

y1
z
x
x1
y
G1 G2
G1 G2 G3 G4

G1
G2 x1 y1 G1 G2 G3 G4 G5 G6
x1
G1 G2 G1  G 2

G7 G8 G9 G10 G11

a) and
v1 v2 v3 u1 u2 u3 are isomorphic graphs

v3 u3
b)
and u1 are isomorphic graphs
v2
v1
u2

G1 G2 G3 G4 G5 G6

and are isomorphic graphs


u v x y
d c v3 v4

a b v2 v1
G1 G2

G1 G2

  v1 v 2 v 3 v 4
G1 G2
 v1 G1 G2
 v4
 v2
 v3


  E1  E 2
G1 G2
 G1 G2

G1 G2
G1 G2
G1 G2

u1 v1
G1 G2 G1 G2
u2 u6 v2 v6
G1 G2
G1 x
v1 v3 u3 u5 v3 v5
v1
G2 y
u4 v4

G1 G2
v3
G1 G2
G1 G2
 G1  G2
u1  v1
u2  v2

u3  v3
u4  v4 a
3 6
u3  v5 e
h 2 4 5 7
u6  v6 d f b
g
 (u 2 , u 5 )  G1 1 8
c
  u2  u5 (v 2 , v 5 )  G2
 G1 G2
G1 G2

a
1
 G1  G2
6        
5 2 e b
f
7
g 
10 j
h
i  G1 G2
9 8

4 3
d c
G1 G2
G3

G1 G 2 G3
V1 , E 1
V1  E1 



6
 2 3 6
 5 7
1 7 7
 4 5
2
4 5
6
 3
H1 H2 H3
 G


2 4 5
7
 3 4 7

 1 3 6
H4 H5 H6
G1 G3
H1 H 2 H 3 H4 H5 H6 2 4 2 4 5 2 4 5
 H5   H6  3 3
1 3 1 1

8 7 8 7 6 8 7 6
H3 H4 H5 H6

 H1 H 2 H6
 H1 H2
H1 3 H3 H4
6
H5 H6 H5 H6

7
4

H1 3 6
G1

7 K 2n  1 K 2n
4 5

H1 H2
H1 1 1 1

H2 H1  H2 
6 2 6 2 6 2
H1 H2
H1 H2
5 3 5 3
(H1 )  H2  5 3

4 4 4
G 1-factor of G 2-factor of G

1 2 1 2 1 2 1 2
2 4
5
3 2
1 1 3
4 3 4 3 4 3 4 3
6 1 3 G
8 7 8 1 - factor 2 - factor 3 - factor
G H1 H2

V  E  V E 
G
V  E   V E 

i) ii)

G1 G2
K1 K1
G G
G G

OR OR
K2 K2 = N2 (K2)
K3 K 3 = N3 (K 3 ) = N 3

OR

N4 N 4 = K4
G GG

G 

(G)
OR OR
Kn

K1

A B
OR OR OR
E
V E
V E C D
G
V E


 
1

4 3

5
G1 G2 G3
7 6
G

K m, n
Km Kn
1

4 3 i) ii)

6
5
7
K2 and K2 K3 and K3

i) ii)
4 d
e
5 3
G1 G2 G2 G1 c
a K2,2 K3,3
G1 G2 1 2
b
G1 G2

i) ii) iii)
G1 (V1 , E 1 )
G 2 (V2 , E 2 ) V1  V2
E1  E 2 G1 G2 G1  G 2
G1 G2 G3

G1 G2 G1  G 2
G1  G 2 G1 G2
e4
v3 e3 v3
v4
e2 e5 v5
x y a b c a b x y z
e1 e6
v2 v1
v2 e1 v1 G1 G2 G3 = N2 G4 = N3
G1 G2
x y a b

V1 v1 v 2 v 3 v 4 V2 v1 v 2 v 3 v 5 v3
a b c x y z
E1 e1 e 2 e 3 e 4 E2 e1 e 5 e 6
G1 + G2 G3 + G4
G1  G 2 v2 e1 v1

V1  V2 v1 v 2 v 3 E1  E 2 e1 G1  G2
Nm  Nn
K m, n
G1 (V1 , E 1 ) G 2 (V2 , E 2 ) G1
G1 (V1 , E 1 ) G 2 (V2 , E 2 )
G2 G1  G 2
G1 G2 G1  G 2
V1  V2 E1  E 2
V1  V2 x1 x 2 y1 y 2
G1 G2 e4 x1 y1 x2 y2 G2 x2 y2 x1 y1 G1
v3 e3 v4

G1 G2 e5 e2
v5 x1 y1 x2 y2 z2
e6
V1  V2 v1 v 2 v 3 v 4 v 5 v2 e1 v1
G1 G2
V1  V2 e1 e 2 e 3 e 4 e 5 e 6 G1  G2

G1 G2 G1  G 2
G1  G 2
G1 (V1 , E 1 )
G 2 (V2 , E 2 ) G1  G 2 V1  V2 (x1, x2) (y1, x2)
E1 E2
(x1, y2) (y1, y2)
(E 1  E 2 ) (E 1  E 2 )
v3 e3 v4 e4 (x1, z2) (y1, z2)
G1 G2
G1  G 2 e5 e2
v5
v1 v 2 v 3 v 4 v 5 V1  V2 v2 v1
e6

(E 1  E 2 ) (E 1  E 2 ) e2 e3 e4 e5 e6  

G1 (V1 , E 1 )
G 2 (V2 , E 2 ) G1  G 2
G1  G 2 G1 G2
b e b e e

a f a f

c d c d d
G H K

v 1  e 1  v 2  e 2  v 3  e 10  v 5
e d
d
e v 1  e 1  v 2  e 2  v 3  e 10  v 5  e 9  v 2  e 7  v 6 v3
f
a a a
f
c v 3  e 3  v 4  e 4  v 5  e 10  v 3  e 8  v 6 v3
b c b
G Graph after fusion of b and e Graph after fusion of c and d
v 3  e 3  v 4  e 4  v 5  e 10  v 3  e 3  v 4 e3

v 0  e1  e 2  e 3 
vn 1  en  vn

v1  e1  v 2  e 2  v 3
vo vn v1 v 2 vn 1
v1  e1  v 2  e 2  v 3  e 8  e 6  e 7  v 2

v6 e5 v5
e6 e4
e8
v1 e7 e10 v4
e1 e9 e3
v2 e2 v3

v1  e1  v 2  v 3
v 2  e 2  v 3  e 3  v 4  e 4  v 5  e 10  v 3  e 2  v 2
v 6  e 5  v 5  e 10  v 3  e 8  v 6
v1  e 6  v 6

v 1  e 1  v 2  e 2  v 3  e 10  v 5  e 9  v 2  e 1  v 1 e1 G1 G2 G 2
v2 G2 G3
v 1  e 1  v 2  e 2  v 3  e 10  v 5  e 9  v 2  e 7  v 6  e 6  v 1
v2
v1  e1  v 2  e 7  v 6  e 6  v1

e6
e1 e7
e3 e5
G1 G2 e2 e8
e4
G
G1 G2

e4 e5 e6
e1 e 3 e 6
e1 e 2
e1 e 2 e 3 e1 e 2

G1
e2
e1

a b G1

c
G1 G1  {e 1 }  e1
G1 G2 G3

e2  e2

v2 v5 v4 v2 v3 v4 v5 v2 v3 e ij vi v j
v2 v3 
v3 v4

v1 v6
   
v2 v5 


  (G)  
 
  
2e

n 

2
b 2 e

22 4
20 6
16 10
a z
c
7
10
8 3 9

d 6 f

  xT 

 


  
  
  
 

  

   a z

 8 9

d 6 f

2
b

   7
a z

8

d 6 f
  

b 3 e

2 5 1
2

a
1
z 
c
7
2
4 3

d 4 f
  


 x T 

  






  

  
  

b 3 e

2 1

a z

b 3 e

2 1 3
1 2
4 2 f 7
a z
c
1
2 4
5 1
2 
d g

  xT 


 



 



 


 



B A C

2
1
D
a f z

1
1
2
d g

C
River er
Riv
River D e a e1
A Kneiphof e1
River e2
Riv f e5
River er e5 e4 e2
d b
B e3
e4 c e3
G1 G2 G3 G4

G1 e2 e3 e4 e5 e1
G1
G2
G 2
G3 e1  e 2  e 3  e 4  e 4  e 5 G3

G 3
G4


K2, 4

K2

a e1 b
e7 e2
e6
f c
e5 e3
e 3  d  e 4  e  e 5  f  e 6  e  e 2  b  e1  a  e 7  f e e4 d
G
G1 G2 G3

Kn
G1
Kn
G2 G2

K2 G3 G3

K2
K2

K m ,n

K m, n

K2, 2 K4, 4
K m, n
K1, 2 K 4, 4
n
v 
2
1 2 3 c d x a b
v
y
c
z
6 5 4 b a u e
d
G1 G2 G3 G4

G1
K m ,n
G1
 K m ,n
G2
K m, n K n, n
G 2
G3 n
  K n, n
G 3 2

G4
G 4 K n, n
 V1 V2 k m, n v1
v2
K m, n  V1

K m, n  V1
V1
K m, n 

Kn 
Kn
a b 1 Kn 
6 2 
G1 c
 G1 5 3 
n
  (K n )
d e 2
4
G2 G1 G2 Kn Kn


Kn
(n  1) !
Kn
2

K 4, 3
K 4, 3 x y z w

K 4, 3 

 K 4, 3 a b c
K4,3
K 4, 3

1 2
1 2 3

5
b e 3 4
6 5 4
7 6

G1 G2
a c d

G1

a b G1

c G2

e d
G2 

6
G2  G2
2
 
a b 1 2 

c 4 3
a b a b a e d

d e d c b c
G1 G2 G3

v1 v1
G1 

vn vn 1
G1
vn
G2
G2
4
2
v1
G3

Kn

7
u v 14 12
9
b c
10
5
x y 13 8
6

d 11 e

7

b c
a

7 14
d e
b c
 6
5
8
a

d e
7

b c

d e


a
a
7
5 4
b c
2
6 b c
8 5 8
3 3
2 5
d e

d 2 e

a

b c
6
5
8

d e
a a

4 4

b 2 c
b c

d e d 2 e

a
4

2
b c 4

b 2 c
8

2
d e

d 2 e

a
2
b c

2
b c

d e

d 2 e

a 4
2
5 b c

3
b c

3 d 2 e

( j)

d 2 e

2
b c

d 2 e

4 G1 G2 G3 G4
2
b c

d 2 e

G1 G2
G1 G2

e4
e9 e2
R6 R5 e7 R4 R3 e3 R2
e1
R1
e10 e6 e1
e8 e1
e5
G

R1 

R2 e1 e 2 e 3 e1 e 2 e 3
R3 e3 e4 e5 e6 
R4 e6 e7 R6 e 10




2e
 
3

2e

3


K5

K5
e1

x e2
G Insertion of v G3 G4
G2 Insertion of vertices Deletion of x from G
or merging of e1 and e2

K5

K5


 

K5   d (x i ) 
x i  v (G)
K5  

K 3, 3   

 K3, 3

K 3, 3
B
 K 3, 3 i) A C ii)

K5 K 3, 3

D F
E

G G1

K5 K 3, 3
G1 G2

i) E ii)
A C

D F
B
n (n  1) 6 5
a y 1 2
2 2

3 4
x b 

z

c 5 6

(i) (ii) (iii)


x y z

K 3, 3 
 24
a b c 
8

2
K 3, 3
1

4
K 3, 3 K 3, 3
3

5 r
6
  (the number of edges in the i th region)
i 1

 

 

 
K 3, 3

Pn 

K1 P1  
Kn
1 2 1 Pn ( )    
3 2 1 1
4 2
2 2
2 3
3
1
G1 G2 G3


G2 G3

 

H1 H2 H3

W G E

1 2

5
3 4

6 7

4
3
5 6

W W G E
G E 1 2

Attempt 1 Attempt 2
b a b

d
a c
i c
f
e
5 3 d h g

(Graph G1) (Graph G2)


4 2 1
6

7
G1
8 9

G2

k m, n V(k m, n )  V 1  V 2
V 1 and V 2 V(k m, n )

b e
 V1
3
V1 V2
2 1 3
1 2
 k m, n
4 2 f 7
a z
c
1
2 4
5 1
2 1 2 1 2
d g

3 4
5 7

4 3 6 5
(Graph 1) (Graph 2)

Ge v2
V2
Ge

v3
v3
v3
v2
v2
v2
v1
v1
v1

(a) (b) (c)

   A B



 C D

v5

100 20

v4
V v1 10
 V FV 30 50

 v2 60 v3

V  V 
E  E   { v 1 , v 2 , v 3 , v 4, v 5 }
 { v 1}   x T x  v 1
v1 v1
{ v 1} { v 2, v 3, v 4, v 5 }
  {v 2} {old L(v 2 ), L(v 1 )  w (v 1 , v 2 )}
{ , 0  30}
{v 3} { , 0   } 
v 1, v 2 , v 3 v 2
{v 4} { , 0   }  B 3 E
{v 5} { , 0  100}
2 5 1
2
{v 2}
1
A Z
v2 v2 C
7
2
{v 1 , v 2 } {v 3 , v 4 , v 5 } 4 3

L{v 3 } min {old L(v 3 ), L(v 2 ) + w(v 2 , v 3 )} D 4 F


L{v 4 } 
L{v 5 }   
L{v 4 }

v4 v4 v1 v5
{v 1 , v 2 , v 4 } {v 3 , v 5 }
2
{v 3} {  , 40  50}
v2 v4 b 2 e
{v 5} {  , 40  20}
{v 5} 6
22 4

v5 v5 20

v 1 to v 5 a
16 10
z
c
v 1 – v 2 – v 4 – v5
3
8 7
10 9

d 6 f
a 1

b c 2 3

d e 4 5

f 6

u
b

v
c t
a Kn

w
G: d H:

g x a b e f
e z l m

f
y

d g n o
c h

(a) (b) (c)

 
a b e f l m

      

g
  c d h n o

(a) (b) (c)

 
 deg (u) 
u v



 deg (u) 
u v
  E1  E2
 deg (u)
u v

(i) (ii) (iii)

(i) (ii) (iii) (iv)

TM

TECHNICAL PUBLICATIONS - An up thrust for knowledge


(e 1  e 2 )  1 n 1 – 1  n 2 – 1+1
b c
a n1  n 2 –1
d
f
n1  n 2
e

n2

G – e e   E (G)
e
 G1 G2
G1 G2 G1 G2
G1 n1 m1 G2 n2 m2
 d(x)  1 x  V (G)

m1 n1 – 1 m2 n2 –1
 d(x) 2( n –1)

x  V (G) mn – m – n + 1

2n   d(x) 

x  V (G)
2n  2n – 2  n  n –1 K1, n K m, 1

 d(v)
VV(T)

K m, n

 ( m + n –1)
K m, n m n
11 15

2 2
v  V(G)

max{d(v, v i ) v i  v(G)}

 c
b
a g
d

n –2 n f
 –1
2 2
G
n n
 n– 1 1 
2 2
n n
1 –1
2 2 b d g
i
n a
2 c f h

n
1
2
n
–1
2

T2 , T4 , T6 , T8 ... T2n
n G
2
G
G K2 K1 K 2 K1

e f b
a
h
d e c
b g j
d
c i g h
k f

G i k

j
Rooted tree

a (root)
level 0

b c d level 1

e f g h i level 2

j k l m level 3

n o p level 4

q r level 5
Directed tree Non directed tree or tree
G

v - Father

u - Son

a - Father

v u - Sons
{a, v 1 , v 2 , v 3 , .... v n –1 , b} a (root)

c c (root)
b

e f f (root)
a (root) d d e f

g h i j i j g h i j
b c level 1

g k l k l k l
d e f h level 2
Subtree with Subtree with
j Tree
i k level 3 root at f root at c

level 4
l m n

3 - ary tree 2 - ary tree Full 2 - ary tree

x  V (T)

1+2 +2 2  .... 2 h


S h +1 1+2 +2 2  .... 2 h

2S h +1 2 +2 2 +2 3  .... 2 h  2 h +1

 

S h +1 2 h +1 – 1

[ Log 2 (n  1) –1]
n –1
2

a(root)
i) ii) iii) iv)
u v
1 Root

u1 u2 v1 v2  1 2  3

u4 2 3
u3 v3 v4 L R
Left Right
u5 u6 subtree subtree
Binary tree Full binary tree Binary tree Not binary tree
1 Root
231

2 3
u L - subtree R - subtree

u1 u2 v 1 Root
213
v1 v2 u4
u3 u4
2 3
u5 u6 v3 v4 u5 u5 u6 L - subtree R - subtree

Left subtree of a Right subtree of a Left subtree Right subtree


of u4 of u2
A

B E

C D F

A B C D E F : ABCDEF
+

Root Left subtree Right subtree

* –

Starting a – d 
C D B F E A : CDBFEA point

b c e f Ending point
Left subtree Right subtree Root
Binary tree

C B D A E F : CBDAEF
[a ( b – c) ]

Left subtree Root Right subtree [a + b)  c]

 

A B E C D E : ABECDE  c d e Ending point

Root Starting a b
Level 1 Level 2
point

C E

A D F

G
B E F

H
C D G H I

Resulting binary tree

A
B E F

B C D E
C D G H I

F G H I J K L M

N O P Q

B E F

C D G H I
B C D E

A  E, B  D, A  F, F  H, F  I F G H I J K L M

N O P Q
A

B

F C

G H
D  –
I

K E
 4 b 
L
M

N
a 5 5 9
O
P
Q

Binary tree

((( x + y)  z) 3)  (19  ( x  x))


* *

2  7 

/ 

5   2

 3 19 

10 5 12 4

 z x x

( 2  ((5)  (10 5))) ((7)  (((12 4) – ( 2))))


2  (5  2) – (7  ( 3 – 1))
x y

(( a  5)  4)  (b – (5  9))

m 2h
(2 h +1 – 1) – 2 h

2 h ( 2 – 1) – 1
h t
2h – 1

d k p w

1+3 +3 2  3 3  ....  3 h
b f v 3 h +1 –1 3 h +1 –1
3 –1 2
3 h +1 –1
2
m h d b f k t p w v


b f d k h p v w t m


b d f h k m p t v w

2h

19
3

2 h +1 – 1
n+1
2

[log 2 (n +1) –1] log 2 (g +1) –1


 2(n –1)

n 1
2

n 1
2

n –1 13 –1
2 2

 W(T2 )

W(T1 ) W(T2 )

T1

Left side a (root) Right side

0 1
b f
W 1 , W 2 , W 3 , .... W t
0 1 0 1
c k g h
0 1 0 1
d e i j

W1 W2
W1  W2

W 1 , W 2 , W 3 , .... W t

W 1 , W 2 , W 3 , .... W t

t
 W i li
i =0

li

T1 a x


b e y z
W(T1 ) 8 6

c d p q
6 7 7 8

T1 T2

T2
    
5 6 6 11 20
k 1 , k 2 , .... k n  
k 1  k 2  k 3  ....  k n

k1 k i+1 11
 , .

ki
 
5 6
ki x > ki x < ki

  + 
k 1 , k 2 , .... k n 
6 11 11 20
k 1 , k 2 , .... k n
k 0 , k 1 , k 2 , .... k n ki
kj  
kj j i  5 6

ki
ki
x < ki 17
x > ki
 
kj kj k j+1 . 6 11
k0 x < k1
+ + 
kn x > kn 20
11 17



    
5 6
 6 11


28

11 17

   
5 6 6 11

 C D A D F E B
20 28 5 5 10 15 15 20 30
   

10 20

48 C G A
10
5 5 10
0 1


28 C G
20
5 5
0 1
D F F
30
15, 15, 20, 20, 30
11 17
 
0 1 0 1 D F
E B 15 15
   
5 6 6 11 20, 20, 30, 30
 

40
    

E 20
20

A 10
10

C G
5 5
B
30, 30, 40
  17

60 0 1

8 9
B 30
30

12, 14 , 16, 17 , 19
D F
15 15  

40, 60 26
100
  0 1
0 1
40 60
12 14
0 1 0 1

E B
20 30 16, 17 , 19, 26,
20 30
0 1 0 1  
A D F
10
10 15 15 33
0 1
0 1
C G
5 5
16 17
A  3 B 2 C  4 D  3 E  2 F  3 G  4
0
 1

8 9

19, 26, 33,


 

45

0 1

8, 9, 12, 14 , 16, 19 19 26

  0 1

12 14

33, 45 78
44 , 51, 51, 59, 67 , 75, 76, 85
   

0 1
95

0 1
33 45

0 1 44 51
0 1

19 26 0 1 0 1
16 17

0 0 1 20 24 25 26
1

9 12 14
8 51, 59, 67 , 75, 76, 85, 95
 

110

0 1

51 59

0 1

29 30

19 20 20 24 25 26 29 30 32 35 37 51 75 85
  67 , 75, 76, 85, 95, 100
39 44 51 59 67  
0 1 0 1 0 1 0 1 0 1
142
19 20 20 24 25 26 29 30 32 35 37 51 75 85
0 1
37 , 39, 44 , 51, 51, 59, 67 , 75, 85
  67 75

76
0 1
0 1
32 35

37 39
76, 85, 95, 110, 142,
0 1  

19 20
161 303

1 0 1
0

76 85 142 161

0 1 1
0 1 0

37 39 67 75 76 85

0 1 0 1
0 1

19 20 32 35 39
37

0 1
95, 110, 142, 161,
  19 20

205 205, 303,


 
0 1

95 110 508

1 0 1
0
0 1
44 51 51 59

0 1 0 1 0 1 205 303

0 1 0 1
20 24 25 26 29 30

95 110 142 161

142, 161, 205 0 1 0 1 0 1 1


0
  44 51 51 59 67 75 76 85

0 1 0 1 0 1 0 1 0 1

20 24 25 26 29 30 32 35 37 39

0 1

19 20

7 , 9, 10, 13
 
16

0 1

7 9

10, 13, 16,


 

23

0 1

10 13

0 1

5 5

0 1

2 3

2, 3, 5, 7 , 9, 13 16, 23
   
5 39

1 0 1
0

2 3
16 23
5, 5, 7 , 9, 13, 1 1
0 0
 
10 7 9 10 13

0 1 0 1

5 5 5 5

0 1 0 1

2 3 2 3
38

1
0

17 21

0 1 0 1

8 9 10 11

8, 9, 10, 11, 13, 15, 22 22, 28, 38


   

17 50

0 1 1
0

8 9 22 28

0 1
10, 11, 13, 15, 17 , 22
  13 15

21 38, 50
0 1  

10 11 88

0 1
13, 15, 17 , 21, 22,
  38 50

0 1 1
28 0

0 1 17 21 22 28

0 1 0 1 0 1
13 15

8 9 10 11 13 15
17 , 21, 22, 28,
 

T1 T2
T3
T4 g  V(T4 ), T4

0 1

0 1

 0

G(V, E)

vo
{v o , }

ei (v o , v i ) v o T

 {{v 0 , v 1}, {e i }}

ek (v k , v j ) v k T
b d f
b d f
b d f v j T ek vj
a a a ek
e g e g
c c e g c
G T1 T2
b d b d
f f
a a

c e g c e
T3 T4
b
8 5

a d
5
f
 {{a, b, d, f }{e 1 , e 2 , e 3 }}

ei W(e i ) b
8 5
a d
5
e 1 , e 2 , .... e n e n +1 f 5 g
e n +1  e i
e 1 , e 2 , .... e n , e n +1  {{a, b, d, f, g}{e 1 , e 2 , e 3 , e 4 }}
b
W(e n +1 )
8 5

a d e
5 6
f 5 g
 {{a, b, d, f, g, e}{e 1 , e 2 , e 3 , e 4 , e 5 }}
b 8 c c
b
8 7 6 8 5 6
5
9 d e a d e
a 8
6
10 5 6 5 6

f 5 g  {{a, b, d, f, g, e, c}{e 1 , e 2 , e 3 , e 4 , e 5 , e 6 }} f 5 g

a G
a
 {{a},  }
b A 2 B
8 4 3 2
1
H
 {{a, b}, {e 1 }} a C
3
G
5
1 3
b
D 4 E 2 F
8 5

a d
 {{a, b, d}, {e 1 , e 2 }}

1 11
9 b
6 f
{{A},  } A
3 2 8
{{A, C}, e 1 } A 10
e c
1 7
4 5
C
d
{{A, C, B}, {e 1 , e 2 }} A
2
B
1
C
a
{{A, C, B, G}, e 1 , e 2 , e 3 } 2 {{a},  }
A B
1 2 {{a, f}, e 1 } a
1
C G f

{{A, C, B, G, H}, e 1 , e 2 , e 3 , e 4 } {{a, f, d}, {e 1 , e 2 }}


2 B a
A
1
1 2 f
3 2
C H G

{{A, C, B, G, H, E}, {e 1 , e 2 , e 3 , e 4 , e 5 }} 2
A B d
1 2 {{a, f, d, e}, {e 1 , e 2 , e 3 } a
3 1
C H G f
1 3 2
E e

{{A, C, B, G, H, E, F}, {e 1 , e 2 , e 3 , e 4 , e 5 , e 6 }} 2 d
A B
{{a, f, d, e, c}, {e 1 , e 2 , e 3 , e 4 }} a
1 2 1
3 f
C H G
3 2
1 c
e
E F 5
2
d
{{a, f, d, e, c, b}, {e 1 , e 2 , e 3 , e 4 , e 5 } 1 b
f
3 2
7 c
e
5
d
v1 5 v2 c
4 2
6 v4 2 2
v3 2 2
2
1 a d
3
v5 4 v6

c
v4 2
2
1
a
{v 4 , v 5 } v5
d
v1 v2 4

2 v4 2 f
{v 5 , v 1 }, {v 4 , v 6 }, {v 6 , v 2 } 1 2
v5 v6
c

v1 v2
{v 5 , v 3 } v3
2 2

2 v4
2 a d
3
1 2
v5 v6 4 4

e
f

b 5 c

10 2
2
b 5 c

9 a 5 d 4 2 2
6
4 4 a d

f 10 e
4 4

f e

e
b 12 e 1 2
7
15 2
10 1 c 4
7 f h
a 4 11 3
c f
14 h
8 9 g
3 d
13 5

8 g
d
b
e

10 1
2
7
e
c
1 4 f h
3

c
d g

e
b
1 2

h 10
c
a 1 e
c
e 2
3 7
1 13
2 4
c f h
h d
3
g

1
2
c
4 h
3

d
g
V (T1 ) V (T2 )
T1 T2

b e4 e b e4 e b e4 e
e1 e9 e1 e9 e1
e5 e5
a e3 e7 f a f a e3 f
e2 e8 e2 e8 n –1
c e6 d c e6 c d
n –1
d

G T1 T2
b e4 c b e6 c b e6 c
e1 e7 e1 e7 e1
e5 e3 e8 d
a e3 e8 d a d a
{e 1 , e 2 , e 3 , e 4 , e 5 , e 6 , e 7 , e 8 , e 9 }
e2 e2 e9
e9 e
T1 f
f e4 e f e4 e
T1 e 1, e 2 , e 4 , e 6 , e 9 G T1 T2
T1 e3, e5, e4, e8
T1
T1 e 1, e 3 , e 6 , e 7 , e 8

e3 {e1 , e2 , e3}
T1
e5 {e1 , e2 , e6 , e5}
e1 {e1 , e2}
e7 {e1 , e2 , e4 , e6 , e7}
e8 e3 {e3 , e2 , e4 }
{e1 , e2 , e4 , e6 , e9 , e8}
e6 {e6 , e5 , e4 }
T2
e7 {e7 , e9}
T2 e 1, e 3 , e 5 , e 4 , e 8
e8 {e8 , e4 , e9 , e5}
T2 e2, e4, e6, e7, e9

e2 {e1 , e3 , e2} v1 v8 v1 v8
e6 {e3 , e5 , e6} e1 e6 e9
e4 e12 e4 e12
e7 {e4 , e5 , e7} e2 v3 e 7 e2 v3
v2 v7 v2 v7
e9 {e4 , e5 , e8 , e9} v5 e10 e7 v5 e10
e3 e5 e13 e5 e13
e8 e11

v4 v6 v4 v6
T1 T2
V (T1 )  V(T2 ) G T

{e 2 , e 4 , e 5 , e 7 , e 10 , e 12 , e 13 }

e3 {e1 , e2 , e7 , e8 , e3}
e4 {e4 , e7 , e8}
e2 {e2 , e1 , e3} e5 {e5 , e1 , e8}
e4 {e4 , e1 , e6} e6 {e6 , e1 , e2 , e8}
e5 {e5 , e3 , e8}
e7 {e7 , e6 , e8}
e10 {e10 , e6 , e9 , e11}
e12 {e12 , e9}
e13 {e13 , e11}

v2 v2

e1 e5 e2 e1 e2

v5 v5
v1 v3 v1 v3
e8 e6 e8

e4 e7 e7
e3

v4 v4

G T
(i, j) W(i, j)
e 1, e 2 , e 7 , e 8
  ( i, j)
(i, j)
 (i, j)  W(i, j) (i, j)
e1 {e1 , e5 , e6 , e3}
e2 {e2 , e6 , e3}
e7 {e7 , e3 , e4 }
e8 {e8 , e4 , e5 , e6 , e3}
  (i, j)   (i, j)
all i all k

e3, e4, e5, e6

v   (a, i) –   ( k , z)
all i all k
(i, j)  (i, j) W(i, j)
 (i, j) W(i, j)

b
2
3
3 d

1 9
a
source 6 c z
(P, P) 4 sink
7 8
P P 5 f
5
w (P, P) g
P
w(P, P)  w (i, j)
i  p and j  P

w(p, p)
b 8 e
2 12

a 7 z
source 9 sink
5 5

c 10 d

P P

(P, P)
w(P, P)
P
P

(P, P)
P
P (–, )

( a + , b) b
w(a, b) –  (a, b)
b
w(a, b) >  (a, b)
b w(a, b) –  (a, b) [if w(a, b) >  (a, b)] 2
7 2
 (a, b)
a c z
9

2
(b , q)
+
3 5

q min{b, [w(b, q) –  (b, q)]}


d
w(b, q) >  (b, q)
(–, )
 ( b, q)
( b, q) b

q min[ b,  (q, b) ]  (q, b) > 0


2,0 2,0
7,0

a c z
(–, ) 9,0

2, 0
3,0 5,0
(y + , z) (y – , z)

(q , y)
+
(q , y)

d

(q + , y) q +
 (q, y) z (q – , y)
 (q, y) – y  (a, b)
b
z c d
+
b (a , 2)

2,0 2,0

P. a c (a+, 9) z
9,0

2, 0
P. ( P, P) 3,0 5,0

+
d (a , 3)
b
(d + , 5)
w(d, z) –  (d, z)
z min{d, {w(d, z) –  (d, z)} 2 7 6
8

a c z
b 9

4
2,0 7,0 2,0 3 5

a c z
(–, ) 9,0
d
3,3 2, 0
5,3
(–, )

b
+
b (a , 2) b

2,0 8,0
2,0 2,0 7,0 6,0
2,0 2,0
7,0 7,0

+ +
a c (a , 9) z (d , 2) a c z a c z
(–, ) 9,0 9,2
(–,) 9,0
2,0 2, 2
3,3 5,3 3,3 5,5
4,0 5,0
3,0

+
d (c , 2) d

+ d
b (a , 2) b

2,0 2,0 2,2 2,2


7,0 7,0 w(a, b) –  (a, b)
b
+ +
a c (a , 7) z (b , 2) a c z
(–, ) 9,2

2,2 2, 2
c d
3,3 5,5 3,3 5,5

d d

+
b (a , 2)
+
b (a , 2) b

2,0 7,0 6,0 8,0


2,0 7,0 8,0 2,2 7,0 8,2
6,0 6,0

a c z
9,0 + +
a c (a ,9) z (b , 2) a c z
(–,) 9,0 9,0

3,0 4,0 5,0


3,3 4,0 5,3 4,0 5,3
3,3

+ +
d (a , 9) d (c , 4) d
d

+
b (c , 6) b
(d + , 5)
w(d, z) –  (d, z)
2,2 7,0 2,2
z min{d, w(d, z) –  (d, z)} min {3, 5} 6,0 8,2 7,0 6,0 8,2

+ +
a c (a ,9) z (d , 2) a c z
(–,) 9,0 9,2
b

5,3 3,3 4,2 5,5


3,3

2,0 7,0 8,0


6,0
+
d (c ,4) d
a c z
(–,) +
b (c , 6) b

4,0 5,3
3,3

2,2 7,0 6,0 8,2 2,2 7,0 6,6 8,8

d
+ +
a c (a ,7) z (b , 6) a c z
(–,) 9,2 9,8

4,2 5,5 4,2 5,5


3,3 3,3

d d
(c + , 4)
b 5 d
(e + , z)
3 8
1 z
a 2 z
2
4
3 (e + , 3)
7
c 4 e
+ +
b (a ,3) (5,0) d (b ,3)
(3,0)
(–, ) (1,0)
(8,0)

a (2,0) (2,0) +
z (e , 3)
(–,)
b d (4,0) (3,0)
(5,0) (7,0)
(3,0) c (a ,7)
+
(4,0) +
e (c ,4)
(8, 0)
(1,0)
a (2,0) (2,0) z
(–,) (a, c)
(4,0) (3,0)
(7,0)
b (5,0) d
c (4,0) e
(3,0) (8,0)
(1,0)
(a + , b) (a + , c) a (2,0) (2,0) z
(–,)
b ((a, b) – f(a, b)
(4,0) (3,3)
(7,3)
c ((a, b) – f(a, c)
c (4,3) e
 (a + , 3) (a + , 7)

(a + , 3) (a + , 7)
(b + ,  d) (c + ,  e) b (a ,3)
+
(5,0) +
d (b ,3)

d min (b, c(b, d) – f(b, d)) (3,0)


(1,0)
min (3, 5, – 0) (2,0) +
z (d ,3)
a (2,0)
(–,)
(4,0)
e min (c, c(c, e) – f(c, e)) (7,3)
+ +
c(a ,4) (4,3) e (c ,1)
min ( 7, 4, – 0)

 (b + , 3) (c + , 4)

P
(5,3)
P {b, d, z}
b d

(3,3)
P P {( a, b), (c, d), (e, z)} c ( a, b)  c(c, d)  c(e, z)
(8,3)
(1,0)
a (2,0) z
(2,0)
(–,)
(3,3)
(7,3) (4,0)

c (4,3) e
a e

1 1
+
b (5,3) d (c ,1) 3
5
(3,3)
(8,3)
(1,0) 4 3 d 3
(2,0) + s t
a (2,0) z (d ,1)
(–,) b

(4,0) (3,3)
(7,3) 5
+
4
+ 1
c (a ,4) (4,3) e (c ,1) 1

c 4 f

b (5,3) d

(3,3) (–, )
(8,4)
(1,1)
a (2,0) z a e
(2,0)

(3,3) 1,0 1,0


(7,4) (4,0)
3,0
e 5,0
c (4,3)

3,0 d 3,0
s t
b (5,3) d (–,) b
(3,3)
(8,1) 5,0
(1,1)
(2,0) 4,0 1,0 1,0
a (2,0) z
(–,)
(3,3)
(7,4) (4,0) c 4,0 f
+ +
c (a ,3) (4,3) e (c ,1)

( P, P)
+
a (s ,2) e
a (s+ ,3)
(1,0) (1,0) 1,1 1,0
3,0
3,0 5,0
5,0
+ +
(s ,3) b d (b ,3) +
+
(s+ ,4) b 3,0 d (a ,1) s t (d ,2)
s t (f ',4) (–,) 4,1 3,0
(–, ) 4,0 3,0
5,5
5,0
4,4 1,0 1,1
4,0 1,0 1,0

c 4,4 f
+
c (s+ ,4) 4,0 f (c ,4)
+ +
a (s ,2) e (b ,1)

1,1 1,0
+
a (s+ ,3) e (b ,1) 3,1
5,0
1,0 1,0 + +
(s ,1) b 3,2 d (b ,1) +
3,0 s t (e ,1)
5,0
(–,) 4,3 3,3
+
(s+ ,4) b 3,0 d (a ,1)
s t (f ' ,4) 5,5
(–,) 3,0 4,4 1,0 1,1

5,4
c 4,4 f
4,4 1,0 1,0
+
a (s ,2) e
+
c 4,4 f (b ,4)
1,1 1,1
3,1
5,1
+ +
a (s ,3) e (b ,1) +
b 3,2 d (b ,1)
s t
1,0 1,0 (–,) 4,4 3,3
3,0
5,0
4,4 5,5
+ 1,0 1,1
+
(s ,3) b 3,0 d (a ,1) +
s t (d ,1)
(–,) 4,1 3,0 c 4,4 f

5,5

4,4 1,0 1,1


P
c 4,4 f

Tic Tac Toe


b 5 d
3 8
1
a 2 z
2
4
3
7
c 4 e

b 4 d
5 5

5 6 2
a z

7 5 6
B 10
g C
h

15

20
A D
16 15
12
20 12
12
E
10
G 14 F

A 2 B

1 4 3 2
3
C G
5
6 1 3
E
B 10 B 10
D 4 E 2 F C C 10
F

b 8
c

10 B 10
B C C 8 8 7

D 9 d
A A a e
5 8
12 6
10 5 6
12 E 12 E
10 10 g
F f 5
G G F

B 10
C
B 10
C

 a
12 e1
12 A D
A D
b
12
12
12 E
12 8
E 10
10 G 14 F
G F a
Weight of MS = 10+10+10+12+12+14 = 68

e1 e2

8 7

a e

e1 e2 e 3

8 7

a d e
5
e1 e2 e 3 e4
b

8 7

a d e
5
5

e 1 e 2 e 3 e 4 e 4 e5

b
a
8 7

d e b c
a
5

5
d e f g
f 5 g

h i j

e 1 e 2 e 3 e 4 e 4 e5 e 6

b c

8 7 8

a d e
5
5

b 4 d
f 5 g
5 5
6 z
a 5 2

7 6
g 5 h

+
(a ,5)
+
(b ,4) PP P
b (4,0) d b (4,0) d
(5,0) (5,0) (5,0) (5,0)
(6,0) (6,0) +
P
(–,00) a (5,0) (2,0) z a (5,0) (2,0) (h ,5)
z
PP
(7,0) (6,0) (7,0) (6,0)
g (5,0) h g h
+ +
(a ,7) (g ,5)

+ +
(a ,5) (b ,4)
b (4,0) d b (4,0) d
(5,0) (5,0) (5,0) (5,0)
(6,0) (6,1) +
a (5,0) (2,0) z a (5,0) (2,0) (h ,1)
z
(7,5) (6,5) (7,5) (6,5) A
g (5,5) h g (5,5) h 5
+ + 6
(a ,2) (b ,5)
1
B D
+ + 5
(a ,4) (b ,4) 5 C
b (4,0) d b (4,0) d
3 2
(5,1) (5,0) (5,1) (5,0) 4
a (5,0)
(6,1)
(2,0) z a (5,0)
(6,1)
(2,0)
+
(d ,4)  E
6
F
(–, ) z
(7,5) (6,0) (7,5) (6,6) { A}, 
g (5,5) h A
g (5,5) h
+ +
(a ,2) (b ,4)
1
+ +
(g ,2) (h ,2)
b (4,4) d b (4,4) d
(5,5) (5,4)
 C
(5,5) (5,4)
(6,1) (6,1) +
a (5,0) (2,0) z a (5,0) (2,0) (d ,1) A
(–, ) z
(7,5) (6,6) (7,5) (6,6)
g (5,5) h g (5,5) h
1
+ + C
(a ,2) (b ,2) 4
F
+ +
(g ,1) (h ,1)
b (4,4) d b (4,4) d A
(5,5) (5,5) (5,5) (5,5)
 {A, C, F} {(A, C), (C, F)} 
1
(6,2) (6,2) +
a (5,1) (2,1) z a (5,1) (2,1) (d ,1)
(–, ) z C D
(7,6) (6,6) (7,5) (6,6) 4
g g
2
(5,5) h (5,5) h
+ +
(a ,1) (b ,1) F
v1 5 v2
A
4 2
 {A, C, F, D}, {(A, C), (C, F), (F, D)}  1 6 v4
5 v3 2 2
D 2
B C
4 3 1
2
v5 4 v6
F


 A

{A, C, F, D, B}, {(A, C), (C, F), (F, D), (C, B)}  5
1
B C D
3 4
2
E
F

{A, C, F, D, B, E,}, {(A, C), (C, F), (F, D), (C, B), (B, E)} 

12
a b

16 20

9
s 4 7 t

4
b d
13 4
9 5 c d
14

4
a 2 3 z
( , )
6
8 (12, 0)
a b
c e
5 (16, 0) (20, 0)

s (4, 0) (9, 0) t
(7, 0)
(–, )

(13, 0) (4, 0)
c d
(14, 0)

(12, –12)
  a b
 
(16, –12) (20, –12)

   , s (4, –0) (9, –0) (7, –0) t


(–, )
   ,
  (13, –4) (4, –4)
c d
  (14, –4)
   

  
 +
(c , 7)
(12, –12)
a b
(16, –12)
 (20, –12)

 
s (4, –0) (9, –0) (7, –7) t +
(d , 4)
(–, )

  (13, –11) (4, –4)


 c d
(14, –11)
+ +
(s , 11) (c , 11)
 

+ + +
(s , 16) (a , 12) (s , 12)
(12, –6) (12, –12)
a b a b
(16, 0) (20, 0) (16, –12) (20, –19)

(9, 0) + (9, –0)


s (4, 0) (7, 0) t (d , 4) s (4, 0) (7, –7) t
(–, ) (–, )

(13, 0) (4, 0) (13, –11) (4, –4)


c d c d
(14, 0) (14, –11)
+ + + +
(s , 13) (c , 13) (s , 11) (c , 11)
 a

a
     b
9

a
9
b

a
15
12 2
9

b 4 c
d
11 10
2 3
5 a
g
9
2 14 b
d
e
8 6
2
7

f
d
2

e
a
9
b
12 15 a
4
9
c b
2
4
3 10
2 c
d 8 11
5 g
2 d
14
7 2
e
6
f e

a
9
b
4
c
2

d
2 J

J
e R
6
f

D R
D R

a
9 G
b
4
c
2
10 J

d
g J
2
D R

D R
e
6 G W
f
G W

    
J J

D R D R
i) ii)

G M W G M W
0 1

E e
E H
0 1 0 1

e t
J J
0 1 0 1

a
D R D R
0 1 0 1 0 1

G M W s t
A G M W n a s

1
E H P
E H P
i
Binary tree Binary tree

J
J

D R
V1 V2 V3
D R 12 5

A G M W 14
A G M W 1 3 8

E H P E H P V5 4
V4 V6

F 6 11
F Q 2 9

This is binary search tree

V7 V9
13 V8 10

(V 2 - V 5 )
V1 V2 V3
12 5 V1 V2

14 3
1 8
1 3

V5 4
V4 V6 V5
V4

6 11
2 9 2

V7 V9
13 V8 10 V7

(V 5 - V 6 )

(V 1 - V 4 ) V1 V2

V1
1 3

1 V5 4
V4 V6

V4
2

(V 5 - V 7 )
V7
V1
(V 2 - V 3 )

V1 V2 V3
1 5

V5
V4 1 3

2 V5 4
V4 V6

V7 2

V7
(V 4 - V 7 )

V1 V2 V3
5

1 3

V5 4 b 5 d
V4 V6
1
6
2 3
8

V7 a z

(V 3 - V 6 ) 2
7 2
(V 6 - V 9 ) 3
4
V1 V2 V3
5
c 4 e

1 3

(–, )
V5 4
V4 V6
b (5,0) d
6
2 9 (3,0)
(8, 0)
(1,0)
a (2,0) (2,0) z
V7 V9 (–,)
(4,0) (3,0)
(V 8 - V 9 ) (7,0)
V3 c (4,0) e
V1 V2 5

1 3 (a + , b) (a + , c)

V5 4 b ((a, b) – f(a, b)
V4 V6
c ((a, b) – f(a, c)
6
2 9  (a + , 3) (a + , 7)

(a + , 3) (a + , 7)
V7 V9
V8 10
(b + ,  d) (c + ,  e)

+ +
d min (b, c(b, d) – f(b, d)) b (a ,3) (5,0) d (b ,3)
(3,0)
min (3, 5, – 0)
(1,0)
+
a (2,0) (2,0) z (d ,3)
(–,)
e min (c, c(c, e) – f(c, e)) (4,0)
(7,3)
min ( 7, 4, – 0) +
c(a ,4)
+
e (c ,1)
(4,3)

 (b + , 3) (c + , 4) b (5,3) d

(3,3)
(8,3)
(1,0)
a (2,0) z
(2,0)
(c + , 4) (–,)
(3,3)
(7,3) (4,0)
(e + , z)
c (4,3) e
z

+
b (5,3) d (c ,1)
(e + , 3)
(3,3)
+ + (8,3)
b (a ,3) (5,0) d (b ,3) (1,0)
(2,0) +
a (2,0) z (d ,1)
(3,0) (–,)
(8,0)
(1,0) (3,3)
(4,0)
a (2,0) (2,0) +
z (e , 3) (7,3)
(–,) +
c (a ,4) (4,3)
+
e (c ,1)
(4,0) (3,0)
(7,0)
+ +
c (a ,7) (4,0) e (c ,4)
b (5,3) d

(3,3) (8,4)
(1,1)
a (2,0) z
(a, c) (2,0)

(3,3)
(7,4) (4,0)

b (5,0) d c (4,3) e

(3,0) (8,0) b (5,3) d


(1,0)
a (2,0) (2,0) z (3,3)
(8,1)
(–,) (1,1)
(4,0) (3,3) a (2,0) z
(7,3) (2,0)
(–,)
c (4,3) e (3,3)
(7,4) (4,0)
+ +
c (a ,3) (4,3) e (c ,1)
T1 e 1 e 2 e 4 e 6 e7

( P, P) e1 e1 e 3 e5
P
e2 e2 e 3 e5
P {b, d, z}
e4 e4 e5
P P {( a, b), (c, d), (e, z)} c ( a, b)  c(c, d)  c(e, z)
e4 e4 e5

e7 e5 e7

G1
T1

b b

e1 e2 e1 e2
e3
a c a c

e7 e4 e7 e4

f d f d

e6 e5 e6

e e

G1 T1

e1 e2 e4 e6 e7
T1 e 3 e5

e3 e1 e2 e 3

e5 e1 e2 e4 e5 e6 e7

A A A

' ' a b A a,b A a b

A A A ... A (n times) A
An A
( a 1 , a 2 , ... a n ) A a2 A

x y x y (or C) x, y (or C)
x, y N x y N

2
–3 N N
3

TM

TECHNICAL PUBLICATIONS - An up thrust for knowledge


z+ z
min {a, b} R
a | b| R
1
2 (–1) 2 –1 z
2
' ' a b b a a, b A
(A , )
x, y
a, b A a b

a ( b c) a b a ( a b) c
' ' a ( b c) (a b) c
a c
a, b, c A

a b b a a, b A
a a
' ' a a a A

c)
a b (a b) c a (b
z a b
z a b
a b
a b a |b|
a b ab

R2 R ab R

(a, b) z z a b a z c d d c b d d b
b
a ( b c) (a b) c
(1, 2) z z , z+

c d d c
b d d b a
b c a ( b c) a b
a b (a b) c c c
b d d b
( C, ), ( C, ), ( , ), ( , ), (Q , )
a ( b c) ( a b) c

0 N

' '
a, b G, a b G

a+b A a, b, A

a, b, c A

' '
a A 0 A
a (b c) (a b) c a, b, c G

' '

' ' (G, )

a (b c) ( a b) c a, b, c G
a G, e G

(G, ) a e e a
(G, )
a (b c) (a b) c a, b, c G
e G e a a e a G a G, b G
a b b a
a+b a, b
(G, )
a –1

(G, )
a b b a a, b G

(G, )

2 2

2 2

Q
2

2 2 2 2

2 2 2

2 2

2 2 2

2 2 2

2 2

2
a a
1 a a 1

1
a
b
(a, b)|a 0
a

1 b a 0 b 0
,
a a 0 0 0 0

ab 0
0 0

1 0
0 0

1
a
a 0
a is non zero real number 1
0 0 0
a
0 0
a 0 b 0 1 0
0 0 0 0 0 0

ab 0
0 0
a 0 b 0 ab 0
0 0 0 0 0 0

b 0 a 0 ba 0
0 0 0 0 0 0
1 0
0 0

a 0 1 0 a 0
0 0 0 0 0 0 x x
x is non zero real number
x x
1 0 a 0 a 0
0 0 0 0 0 0

1 0
x x y y
0 0
x x y y

x x y y 2xy 2xy
a 0 b 0 x x y y 2xy 2xy
0 0 0 0
e e
e e 2xy 2xy
2xy 2xy

x x e e e e
x x e e e e
cos sin
is non zero real number
2xe 2xe e e sin cos
2xe 2xe e e

1 cos sin cos sin


2 sin cos sin cos
1 1 cos ( ) sin ( )
2 2 sin ( ) cos ( )
1 1
2 2

x x
cos 0 sin 0 1 0
x x
sin 0 cos 0 0 1
y y
y y
1 0
1 1
0 1
2xy 2xy 2 2
2xy 2xy 1 1
2 2
cos sin
1 sin cos
2
cos sin
1 sin cos
4
1 1 0
4x 0 1

1 1
4x 4x
1 1
4x 4x

ab
2

ab
2

bc a(bc) abc
a*
2 4 4

ab (ab)c abc
*c
2 4 4

ae
2

ab
2

4
a
4
a

ab
2
ba ab
2 2
x S x x

x, y T,
( x y) ( x y) (( x y) x) y

( y ( x x)) y
(y x) y
(x y) y a a e e a

x ( y y)
x y

x y T x, y T
' '
x, y, z T S a b
x ( y z) (x y) z a b

– a –2
e e e T a
( , )

a b

a b a, b b a a, b

a b a, b ( , )

a, b a + b +1 a b
a+ mb a+ mb 0 r m
a, b, c
(a b) c (a b 1) c 5 39

15 5 25
a (b c) a (b c 1)

a mb

a mb 0 r m
3 45 a, b, c G

9 64 a 8 ( b + 8 c) (a 8 b) 8c

a G 0 G

a 80 0 8a

' 8'

a
0 80

1 87
5| (12 – 2)|
7 81
5| (12 – 2)|
8 | (30 – 6)| 2 86

5| (9 – 9)| 6 82

3 85

5 83
(G , 8)
4 84
8
a G
+8 0 1 2 3 4 5 6 7
a, b G
0 0 1 2 3 4 5 6 7
1 1 2 3 4 5 6 7 0 a + 8b b + 8a
2 2 3 4 5 6 7 0 1
3 3 4 5 6 7 0 1 2 ' 8'
4 4 5 6 7 0 1 2 3 (G, + 8 )
5 5 6 7 0 1 2 3 4
6 6 7 0 1 2 3 4 5 (G, 7)
7 7 0 1 2 3 4 5 6

a + 8b G a, b G

x7
a 7b b 7a
1 2 3 4 5 6
1 1 2 3 4 5 6 7
2 2 4 6 1 3 5
(G, 7)
3 3 6 2 5 1 4
4 4 1 5 2 6 3 (G, 6)
5 5 3 1 6 4 2
6 6 5 4 3 2 1 2, 3 G 2 63 0 G

6
(G, 6)
a 7b G a, b G
(G, 6)
7
(z 6 , )
z6
a , b, c G
z6 {[ 0], [1], [ 2], [ 3], [ 4 ], [5]}
a 7 (b 7 c) (a 7 b) 7 c
z6 { 0 , 1 , 2 , 3 , 4 , 5}
7

+ 0 1 2 3 4 5
0 0 1 2 3 4 5
a G 1 G 1 1 2 3 4 5 0
2 2 3 4 5 0 1
a 71 1 7 a
3 3 4 5 0 1 2
4 4 5 0 1 2 1
5 5 0 1 2 3 4

z6

1 a b z6 a, b z6
71

2 z6
74 4 72

3 75 5 73

6 76 a , b, c z6

a G a (b c) (a b) c

z6

a, b G
a, b, c, z4
a ( b c) (a b) c
a z6 , 0 z6
z4
a 0 0 a a
a b b c
0 z6 4l+t
(a b) c r c
a ( b c) a t
0 0 0 0 0
1 5 5 1 0 1 5 5 1 4 a l+a t 4 a l + 4m + k
2 4 4 2 0 2 4 4 2 (4p + r) c
3 3 0 3 3

z6 z6 4 a l + 4m + k 4pc + 4q +s

a, b z6 ( a b) c a ( b c)

a b b a (z 4 , )

z6
(z 6 , )

' ' a b a z4 , 0 z4

(z 4 , ) z4
(z 4 , )
z4

z4
z4
0 1 2 3
0 0 0 0 0 x 3 x 3
1 0 1 2 3
2 0 2 0 2
3 0 3 2 1

4m
a, b z4 a b z4 3
(z 4 , ) z4 z4

0 I f1 f2 f3 f4 f5
4 z4
I I f1 f2 f3 f4 f5
z4
f1 f1 I f3 f2 f5 f4
(z 4 , )
f2 f2 f4 I f5 f1 f3
(z 4 , )
f3 f3 f5 f1 f4 I f2
f4 f4 f2 f5 I f3 f1
S S f5 f5 f3 f4 f1 f2 I

P3
f(a 1 ) b1 f(a 2 ) b2 f(a 3 ) b3 f(a n ) b4
f, g P3 fog P3

a1 a2 a3 .... a n Elements in Domain


b1 b2 b3 .... b n Elements in Co- domain
f, g, h P3
f o (g o h) (fog) h
a1 a2 a3 .... a n
b1 b2 b3 .... b n

a1 a2 a3 .... a n b1 b2 b3 .... b n
f1 f2
b1 b2 b3 .... b n c1 c2 c3 .... c n
f P3 , I P3

a1 a2 a3 .... a n
f1 f 2
c1 c2 c3 .... c n

a1 b1 c1 a1 c1

I oI I I –1

f1 o f1 I f 1–1 f1
P3
(1 2 3) (1 2 3) (1 2 3) (1 2 3) (1 2 3) (1 2 3) f2 o f2 I f 2–1 f2
(1 2 3) (1 3 2) ( 21 3) ( 2 31) ( 31 2) ( 3 21)
P3 f 3 o f4 f4 o f 3 f 3–1 f4 f 4–1 f3

I f1 f2 f3 f4 f5 f5 o f5 I f 5–1 f5
P3 .
( P3 , )
f1 o f 2 f3 f 2 o f1 f4
f1 o f 2 f 2 o f1
(P3 , o )

(P3 , o )

(G, )

H1 H2 H3

(G, )
( H, )

( , )
( , )
( , )
( o, ) ( o, )
({1}, ) (Q o , ), ( o, ), ( , )
b 1 b 1

H2

H1
H2 ( , )
H1 H2

2, 3 H1 H2 5 H1 H2
H1 H2
H1 H2
{1, – 1, i, – i}

(G, )
a G
(–1) 2
H a { h a} h H}
a H {a h h H} ( i) 4

(– i) 4

H a a H ( , +)
(G, ) H a a H
( , ) 5 5n n5 0

1 , H +1
3 , H +3
a G
5 , H +5 x G am

(G, )
a G
an ( i) 1 ( i) 2 ( i) 3 ( i) 4

(– i) 1 (– i) 2 (– i) 3 (– i) 4

o ( a –1 ) a G (G, ) g G
h H
–1 –1
g h g H ghg H

N ( a b) c)
(N ( a b)) ( N c)
[(N a) ( N b)] ( N c)
[(N a) ( N b)] ( N c)

N e GN N a GN
( , ) ( N a) ( N e) N ( a e)
(2 , ) (3 , ) ( , ) N a
N e

(G, ) a G, N a G N

a –1 G N a –1 G N
{N a a G}
( N a) ( N a –1 ) N ( a a –1 ) N e
(G N, )
N a –1 N a
(G N, )
(G, ) (G N, )

{N a a G}

a, b G, N a N b GN
( N a) ( N b) N ( a N) b
N ( N a) b
(N N) ( a b)
N c N N a b c G
hi hj hi hj
( N a) ( N b) G N

a, b, c G

N a N b N c GN
(N a) [( N b) ( N c)] N a [ N ( b c)] Ha 1 Ha 2 ........... Ha k
N a ( b c)
an

n
m n
m

3 2 1
a , a , a , a 0 , a 1, a 2 , a 3

aras ar s
as r
a sa r

a, a 2 , a 3 , ...... , a m

ar as ara s
a sa s
ar s
a0 ar s
e a 1

ar s

am
as

a, a 2 , a 3 , ...... , a m a0

a mp q

a mp a q
p
am aq aq

aq a, a 2 , ...... , a m a0

1 0 1 0 1 0 1 0
0 1 0 1 0 1 0 1

1 0 1 0 1 0
0 1 0 1 0 1

1 0 1 0 1 0
0 1 0 1 0 1

1 0 1 0 1 0
0 1 0 1 0 1

am

(a m ) q H

a mq H
mq
a H

at (a m )

am
1
b

b 1

1
g

g 1

1
g

w2

a b c d (e 1 ) (e 2 )
a b c d
f (a –1 ) [f (a)] –1
a b c d
G1 G1 e2 G2
b a c d
f(a) o e 2
a b c d
a e1
a b d c
f (a) o e 2 f (a) o f (e 1 )
a b c d f (e 1 ) e2
b a d c
a G1 then a –1 G

e2 f (e 1 )
f (a a –1 )

e2 o f (a –1 )

f (a –1 ) [f (a)] –1

(G1 , ) (G 2 , o )
(G1 , ) (G 2 , o)
G1 G2

1 1 1 1
f1 f2 f3 f4 G1 G2 G1 and G 2
G1 G 2

(G1 , ) (G 2 , o)
a G
(G1 , ) (G 2 , o )
G G
(a b) = f (a) o f(b) G1
a b G1 f(a) o f(b) G2

e e

G1 G2 (G1 , ) and (G 2 o)
(R o , o) (R o , o)

y y y
m m m

y y
m m
ex

e x1 e x2

x1 x2
e x1 x2

e x1 x2

a R, o R s. t.

a R, –a R

a 0
b 0 a b

a b a, b R
a, b R, a b R
2 ( 4, , ) 2 2 = 4 = 0

(M 2 2 ) 1
a ( b c) (a b) c
1 0 0 0 0 0
a, b, c R A B A 0 and B 0
0 0 0 1 0 0
a (b +c) a b+a c
(a + b) c a c+b c ( , , ) a b

b R, a b = b a
( , , ), ( , +, ), ( , +, )
( 4 , +, )
a R, , 1 R a 1 1 a

n n

( , , ), ( , +, ), (Q, , )

( , +, )

a 0 0 a (R, +, ) (S, + , )
a (–b) (–a) b – (a b)
(–a) (–b) a b
(a + b) (a) + (b) ( a + b 2) (c +d 2 2
(a b) (a) (b)

a R| (x) = 0 a b 2 c d 2
( )

a b a b
: a, b z
b a b a

a+b 2

c +d 2
a b c d a +c b +d
b a d c b +d a +c a 0

(a +c) – (b +d) acd +2bd 2

(a – b ) + (c – d)
a b c d
f
b a d c

a b c d ac + bd ad + bc – bc 2 2bd 2
f
b a d c bc + ad bd + ac
b (2d 2 – c 2 )
(ac + bd) – (ad + bc) c2 2d 2 2
(ac – bc) + (bd – ad)
c 2 2d 2
(a – b) (c – d)
c2 2d 2
a b c d
f f 2 2
b a d c

a b a a 2 2
:a b 0 ; a z
b a a a 2 2 c +d 2

a + b 2 ; a, b z 1
2

( a + b 2) (c +d 2 2

(A 2 2 B2 2) C2 2 A2 2 (B 2 2 C2 2)

0 0
0 0 2 2

0 0
0 0
2 0 0 0
0 0 0 3
7
0 0
0 0

z 22 z 22

+7

z2

(z 8 , 7)
z8 z8 +7
z8
a + 7 ( b + 7 c) (a 7 b) + 7 c

z8

z8

f (x) aixi
i =0

ai 4x + 4x 2

[2 2 (2 4 4 2)x 4 4x 2 ]
aixi bi x i
i =0 i =0 x2
ai bi 2
x

0 1
c0x c1 x .....

Ci x i
i =0

ci ai bi

d 0x 0 d 1 x 1 ..... d n x n

ci d0 x0 a 1 b n –1 a 2 b n – 2 .... a n b 0

di ai bj
i j=n

[f(x) g(x)], f (x) g(x) h(x) h(x) h(x)


f (x) = 2x +4x 2
g(x) = 2+6x +4x 2 h(x) = 2 + 4x

( 2x + 4x 2 ) [2 +6x + 4x 2 ]

8x 2
2( mod 8)

( 2x + 4x 2 ) (2 +6x + 4x 2 )

0 (2 2 0 6) + (2 4 +6 2 + 4 0) x 2 ( 2 4 4 6) x 3 ( 4 4) x 4

4 x +20x 2 32 x 3 16x 4

4x + 4x 2 0x 3 0x 4
x i and y i
(1 0 1 1 0 1)

( 0 1 1 1 1 0)

x y
w(x y)

Sn w(x y)

Sn Sn (S n , )
Sn (x 1 , x 2 .... x n ) ( y 1 , y 2 , ... y n )
x y = (x 1 y 1, x 2 y 2 , ..... x n yn)

(S n ,

x y

Sn

( x 1 , x 2 , .... x n ) ( y 1 , y 2 , .... y n ) Sn ,

k n

2m 2n – k

x Ht

{ 000, 101, 110}

{001, 010, 100, 111}


B2 B5

k n

2k

2k k 1

2m 2n – k

(x 1 , x 2 , x 3 , xn)
x Ht
Ht
1 1 0 1 0 0
0 1 1 0 1 0
1 0 1 0 0 1

1 1 0 1 0 0
0 1 1 0 1 0
1 0 1 0 0 1 3 6

3 6

1 1 0 1 0 0
0 1 1 0 1 0
1 0 1 0 0 1 2m
x Ht
1 0 1
1 1 0
0 1 1
[x 1 x 2 x 3 x 4 x 5 , x 6 ] [ 0 0 0]
1 1 0 1 0 0
h1 0 h2 1 h3 1 0 1 0
1 0 1 0 0 1

x1 x 2 x4
0 x2 x3 x5
h1 h2 h3 0 x1 x3 x6
0 x4 x1 x2
x5 x2 x3
x6 x1 x3

x 1 , x 2, x 3,

x1 x2 x3 x4 x5 x6
2m

2n m

a 1 , a 2 , ... , a r 1 a1
a1 a1
a1 a1

a1 Sn
a1 C a1 a1 x C

Sn a2
a2

Sn

x1 xn

a1 a1 a1 x1 a1 xn
Sn , (S n , ) a2 a2 a2 x1 a2 xn

Sn
(i 1) th
ai ai (i 1) th

Sn
Sn Sn
Sn
c 1 , c 2 ... , c n c n , c 1 , .... , c n 1) Fn
x2 x 1
x2
x2 x 1 x3 x2 x x3 1 x4 x3 x2 x4 x3 x 1, x 4 x, x 4 x2 1

xi
b j xj
(a 0 a 1 a 2 ... a n 2 an 1)
(a n 1 a 0 a 1 a 2 ... a n 2)
Fn

a 0 a 1 a 2 ... a n 1
xn 1
a0 a 1 x ... a n 1

(Fn [x], , xn

(Fn [x] , ,

n 1
a0 a1 ... a n 1 X

a 0 x a 1 x2 ... a n 1 xn
1 xn

a 0 x a 1 x2 ... a n 1 an 1 (1 xn )
1 xn

an 1 a 0 x a 1 x2 ... a n 2 xn 1
Z 12
Z 12 12

Z 12

Z4
Z4

 0 1 2 3

0 0 1 2 3

1 1 2 3 0

2 2 3 0 1
3 3 0 1 2

Z4 Z4

Z4
0 1

–1 –1
(1) ( 2) ( 3) – 1

Z4 Z4

a 10 (b 10 c) (a 10 b) 10 c

(A, 10 )
(A, 10 , 10 )
10

10

10
(A, + 10 )

10

a 10 (b 10 c) (a 10 b) 10 c

a 10 b b 10 a

10 10

(A, + 10 )
(A, + 10 )

10
u1 v1

u2 u6 v2 v6

u3 u5 v3 v5

u4 v4

G1 G2

2
b 2 e

22 4
20 6
16 10
a z
c
7
10
8 3 9

d 6 f
(1  x) 6 x3

1 2

5
3 4

6 7

8th ( x  y) 13
( 3x  4) 4

b 2 e
b 8 c

6 8 7 6
5
22 4 9
a d 8 e
20
6
10 5 6
16 10
a z f 5 g
c
3
8 7
10 9

v2 v2
d 6 f

e1 e5 e2 e1 e2

v5 v5
v1 v3 v1 v3
e8 e6 e8

e4 e7 e7
e3
a b e f l m
v4 v4

G T

d g n o
c h

(a) (b) (c)

b 5 d
Kn , Kmn
3 8
Kmn 1
a 2 z
2
4
3
7
c 4 e
1 1 0 1 0 0
0 1 1 0 1 0
b 5 c  
1 0 1 0 0 1 
10 2
2

9 a 5 d 4
6
4 4

f 10 e

b c

d e f g

h i j

     

( a  b 2 , , x) 

 

 

You might also like